230
เอกสารประกอบการสอน รายวิชา ภาษาอังกฤษเพื่อการสื่อสาร 2 รหัส 2000-1202 หลักสูตรระดับประกาศนียบัตรวิชาชีพ (ปวช.) พุทธศักราช 2545 (ปรับปรุง 2546) นางปฏิมา มเหศวร ครู คศ.2 กลุมวิชาภาษาอังกฤษ แผนกสามัญสัมพันธ วิทยาลัยเทคนิคนครนายก สํานักงานคณะกรรมการการอาชีวศึกษา

เอกสารประกอบการสอน¸ าษา...ก ค าน า เอกสารประกอบการสอนรายว ชาภาษาอ งกฤษเพ

  • Upload
    others

  • View
    2

  • Download
    0

Embed Size (px)

Citation preview

เอกสารประกอบการสอน รายวิชา ภาษาอังกฤษเพื่อการสื่อสาร 2 รหัส 2000-1202

หลักสูตรระดับประกาศนียบัตรวิชาชีพ (ปวช.) พุทธศักราช 2545 (ปรับปรงุ 2546)

นางปฏิมา มเหศวร ครู คศ.2

กลุมวิชาภาษาอังกฤษ

แผนกสามัญสัมพนัธ วิทยาลัยเทคนิคนครนายก สํานักงานคณะกรรมการการอาชีวศกึษา

ก คํานํา

เอกสารประกอบการสอนรายวิชาภาษาอังกฤษเพื่อการสือ่สาร 2 (English for Communication 2) รหัสวิชา 2000 – 1202 มีเนื้อหาตรงตามจุดประสงครายวิชา มาตรฐานรายวิชา และคําอธิบายรายวิชา หลักสูตรประกาศนียบัตรวิชาชีพ (ปวช.) พทุธศักราช 2545 (ปรับปรุง พ.ศ. 2546) กระทรวงศึกษาธิการ เอกสารประกอบการสอนรายวิชาภาษาอังกฤษเพื่อการสือ่สาร 2 แบงเนื้อหาออกเปน 8 หนวย แตละหนวยจะเกีย่วของกับกิจกรรมในชีวติประจําวนัของผูเรียนจากสิ่งที่ใกลตัว ไปสูสังคมภายนอกที่ไกลตัวออกไป มีการบูรณการอยางเบ็ดเสร็จในทักษะทัง้ 4 คือ การฟง การพูด การอาน และ การเขียนที่มีความสัมพันธกันอยางตอเนื่องที่มีเนื้อหาเนนความรู ดานคําศัพท สํานวน และ ประโยคทางภาษาที่ประกอบดวยแบบฝกหัดทีเ่นนผูเรียนสามารถเรียนรู และใชภาษาอังกฤษเพื่อการสื่อสารไดอยางมีประสิทธิภาพ เนื่องจากการจัดกิจกรรมการเรียนการสอนเปนแบบกระบวนการทางภาษาที่มุงเนนที่ตัวผูเรียนไดมกีารฝกทักษะทั้ง 4 เพื่อการสื่อสารอยางแทจริง รวมทั้งมีการเนนการบูรณาการแบบสอดแทรกตรงตามเนื้อหาทกุหนวยการเรยีนโดยมีแบบประเมิน เกณฑประเมินที่ชัดเจนเพื่อรองรับนโยบายของคณุธรรมนําความรูอันเปนการสงเสริมพัฒนาทักษะชวีิตผูเรียนใหเกดิความรูดวยบทบาทของความเกง ความดี ความสุข ขอขอบคุณคณะผูบริหาร คณะครู และผูมีสวนเกีย่วของทุกทานที่ไดใหขอเสนอแนะสําหรับการปรับปรุงแกไขเพื่อการพฒันาใหเอกสารประกอบการสอนรายวิชาภาษาอังกฤษเพือ่การสื่อสาร 2 นี้มีความสมบูรณเกดิประโยชนแกครูผูสอนและตัวผูเรียน ที่หวังจะพฒันาภาษาอังกฤษใหกับตนเองจนเกิดความมั่นใจในการนําไปใชส่ือสารในชีวิตประจําวันทีถู่กตองตามวฒันธรรมของภาษาในสากลนิยม

นางปฏิมา มเหศวร กลุมวิชาภาษาอังกฤษ แผนกสามัญสัมพันธ

วิทยาลัยเทคนคินครนายก

ข สารบัญ

หนา คํานํา ก สารบัญ ข สารบัญรูปภาพ ค จุดประสงครายวิชา ง มาตรฐานรายวิชา ง คําอธิบายรายวิชา ง กําหนดหนวยการเรียนรู จ หนวยที่ 1 Introduction สาระสําคัญ จุดประสงคการเรียนรู เนื้อหาสาระ เนื้อหา ใบความรูที่ 1 ใบงานที่ 1 – ใบงานที่ 5 แบบทดสอบประจําหนวยที่ 1 เฉลยใบงานที่ 2 – ใบงานที่ 5 เฉลยแบบทดสอบประจําหนวยที่ 1 แบบประเมินการทําแบบฝกหัดรายบุคคล แบบประเมินทักษะการเขียนบทสนทนา แบบประเมินพฤตกรรมดานมนุษยสัมพนัธ กิจกรรมการเรยีนการสอน ส่ือการเรียนการสอน การวัดผลประเมินผล บันทึกหลังสอน หนวยที ่

ง ภาษาอังกฤษเพื่อการสื่อสาร 2

(English for Communication 2) รหัสวิชา 2000 – 1202 หนวยกิต 2 หลักสูตรประกาศนียบัตรวิชาชีพ (ปวช.) พุทธศกัราช 2545 (ปรับปรุง 2546) เวลา 2 ชั่วโมง/สัปดาห รวม 36 ชั่วโมง

---------------------------- จุดประสงครายวิชา

1. เพื่อใหมีความรูความเขาใจเกี่ยวกับโครงสราง ศัพท สํานวนภาษาอังกฤษที่ใชส่ือสารใน สถานการณตาง ๆ ในชีวิตประจําวันและการประกอบอาชีพ

2. เพื่อใหสามารถใชภาษาอังกฤษในการตดิตอส่ือสารกับชาวตางชาต ิ

มาตรฐานรายวิชา

1. ศึกษาโครงสราง ศัพท สํานวนภาษาอังกฤษที่ใชในการสื่อสารในชีวิตประจําวันและการ ประกอบอาชพี

2. ใชศัพท สํานวน โครงสรางประโยคภาษาอังกฤษที่เกีย่วของในชีวิตประจําวันเพื่อส่ือสาร กับชาวตางชาติไดอยางมีประสิทธิภาพ คําอธิบายรายวิชา ศึกษา ฝกปฏบิัติการฟง การพูด การอาน การเขียน เร่ืองราว เนื้อหาจากสื่อส่ิงพิมพประเภทตาง ๆ คําบรรยาย ลักษณะบคุคลและสิ่งของ คําแนะนํา สุขภาพ บทสนทนาเกีย่วของกับชีวิตประจาํวัน ขาว เร่ืองทั่วไป สถานที่ การบอกทิศทาง เหตุการณสําคัญตาง ๆ เหตุการณในอดตี อนาคต เขาใจน้ําเสียง ความรูสึกของผูพูด ความเหมือนและความแตกตางระหวางภาษาอังกฤษและภาษาไทย ใชภาษาและทาทางไดถูกตองตามมารยาทสังคมเหมาะสมกบักาลเทศะ การใชคํา สํานวน วลี ถายโอนขอมูลจากเรื่องที่การอานและการฟง มีความเขาใจเกีย่วกับวันสําคัญและประเพณีของเจาของภาษา วางแผนการเรียนโดยใชส่ือที่มอียูในศนูยการเรียนรูในสถานศึกษา

จ กําหนดหนวยการเรียนรู

วิชา ภาษาอังกฤษเพื่อการสื่อสาร 2 รหัส 2000 – 1202 หลักสูตรประกาศนียบัตรวิชาชีพ (ปวช.) พุทธศกัราช 2545 จํานวน 2 ชั่วโมง

สัปดาห ชั่วโมง แผนการเรียนรู หนวยท่ี ชื่อหัวขอ ชั่วโมง1 1 – 2 1 1 Introductions 2 2 3 – 4 2 2 Number, Days and Dates 2 3 5 – 6 3 3 Giving Advices and Making Suggestions 2 4 7 - 8 4 4 Health 2 5 9 – 10 5 5 Likes and Dislikes 2 6 11 - 12 6 6 Asking and Giving Opinion 2 7 13 - 14 7 7 Holiday 2 8 15 - 16 8 8 Offering, Accepting and Refusal 2

9 17 - 18 - - Evaluation 1 (Unit 1-8) 2 10 19 - 20 9 9 Giving Directions 2 11 21 - 22 10 10 Tell Me about Yourself 2 12 23 - 24 11 11 Reading Advertisements 2 13 25 - 26 12 12 Making a Phone Call 2 14 27 - 28 13 13 Telephone Messages 2 15 29 - 30 14 14 Signs and Symbols 2 16 31 – 32 15 15 Instructions and Warnings 2

17 33 – 34 - - Evaluation 2 (Unit 9 – 15) 2

18 35 – 36 - - Evaluation 3 (Unit 1 – 15) 2

รวม 36

Unit 1 Introductions

รูปภาพ 1.1: Business Venture 1 by Roger Barnard & Jeff Cady หนา 9

Unit 1

Introduction

สาระสําคัญ

การทักทายเปนวัฒนธรรมในการเขาสังคม ถือเปนสิ่งจําเปนในการเขาสังคมของทุกชาติทุกภาษา สําหรับการพูดทักทายในภาษาอังกฤษมีแบบฉบับของการสื่อสารในชีวิตประจําวันของสังคมที่มีมาชานานที่นํามาปฏิบัติกันมาจนถึงปจจุบัน การทักทาย การแนะนําใหรูจักกัน รวมทั้งการอําลาจากกันนั้นมีรูปแบบ 2 ประเภทใหญ รูปแบบที่เปนทางการ (Formal Form) รูปแบบที่ไมเปนทางการ (Informal Form) เปนพื้นฐานการพูดภาษาอังกฤษที่นําสูการประกอบอาชีพ การปฏิบัติงาน การศึกษาตอ

จุดประสงคการเรียนรู ความรู

1. รูและเขาใจศพัท สํานวน ประโยคเกีย่วกับการทักทายและการอําลา ทั้งรูปแบบที่เปนทางการ และไมเปนทางการ

2. รูและบอกประเภทของการใชคําศัพท สํานวน ประโยคเกีย่วกับการแนะนําตนเองและการ แนะนําผูอ่ืนใหรูจักกนั

3. เขาใจและใชคาํศัพท สํานวน ประโยคเกีย่วกับประโยคบอกเลา ประโยคปฏิเสธ และประโยค คําถาม ทักษะ

4. ฟงและปฏิบัตกิารพูดคําศัพท สํานวน ประโยคเกีย่วกับการทักทายและการอําลาทั้งรูปแบบที่ เปนทางการและไมเปนทางการไดถูกตอง

5. พูดและใชประเภทของคําศพัท สํานวน ประโยคเกีย่วกบัการแนะนําตนเองและการแนะนําผูอ่ืน ใหรูจักกันไดอยางถูกตองตามรูปแบบ

6. บอกและปฏิบตัิการเขียนคําศัพท สํานวน ประโยคเกีย่วกับประโยคบอกเลา ประโยคปฏิเสธ และประโยคคาํถามไดถูกตองตามหลักไวยากรณ คุณธรรม

7. สรางทัศนคติที่ดีตอการพูดสื่อสารภาษาอังกฤษโดยการเสริมสรางมนุษยสัมพันธใหเกิดแกตวั ผูเรียนสําหรับการปฏิบัติดวยการบูรณาการ 4 ทักษะ เกีย่วกับการทักทาย การแนะนํา และการลาจากกันเปนภาษาอังกฤษ

เนื้อหาสาระ 1. Language Focus

1.1 Greetings and Responses 1.2 Self – Introductions and Responses 1.3 Saying Goodbye and Responses 1.4 Third – Party Introductions and Responses 1.5 Greeting around the World 1.6 Present Simple Tense 1.6.1 Affirmative Sentences 1.6.2 Negative Sentences 1.6.3 Question Sentences

2. Worksheet 2.1 ใบงานที่ 1.1: Warm up: Listening and speaking อานออกเสียงตามประโยคที่กําหนดใน Exercise 1 2.2 ใบงานที่ 1.2: Reading: Greeting around the World อานและสรุปเกี่ยวกับ Greeting ใน Exercise 2 2.3 ใบงานที่ 1.3: Reading: อานบทสนทนาและเติมดวยคําหรือขอความที่กําหนดใหใน Exercise 3 - 4 2.4 ใบงานที่ 1.4: Writing: เลือกคําที่กําหนดเติมคําในชองวางใหถูกตองตาม Tense ใน Exercise 5 – 7 2.5ใบงานที1่.5:Writing: ฝกเขียนบทสนทนาเกีย่วกับการทักทายกันโดยพิจารณาจากประโยคเพื่อหาประโยคของการสนทนามใหสอดคลองและสัมพันธกันใน Exercise ที่ 8 ในใบงานที่ 1.5 และ Exerciseที่ 9 ในใบงานที่ 1.5 2.6 ใบงานที่ 1.6 แบบทดสอบประจําหนวยที่ 1 เพื่อทดสอบองคความรูของหนวยที่ 1

Greeting and Response

Greeting Response - How are you? - How are you doing? - Are you doing okay? - How’s it going? - How’s everything? - And you? - How are things going? - Are you OK?

- I’m doing very well.

- Very well. - Great. - Fantastic!

- I’m fine. - Fine - Oh, so – so. - Not bad. - Pretty good. - Okay (OK) - All right. - They are all fine.

-Awful! - Pretty bad! - Terrible! - Not so good. - Not very well.

- How do you do? - My pleasure.

Practice Conversation 1 A: Good morning, Ann. B: They‘re all fine. A: I’m doing very well, thank you, and you? B: Very well, thank you.

Self – Introductions

Self – Introductions Response - Let me introduce myself. I’m........................... - May I introduce myself? My name’s .............. - How do you do? My name ‘s .......................... - Good to meet you. I’m .................................... - Hello. I don’t think we know each other. My name‘s....................................................

- It’s a pleasure to meet you. My name’s ............. - It’s nice to meet you. I’m .................................. - Glad to meet you. I’m ....................................... - It’s nice meeting you. My name’s .................... - It was very good to meet you. - Please to meet you. - How do you do? -It’s been great seeing you. My name‘s............

Saying Goodbye and Responses

Self – Introductions Response - It was nice talking to you. - I’ve really enjoyed talking to you. - I’ve enjoyed talking to you. - I’ve enjoyed seeing you. - I’d better be going. - Well, it’s getting pretty late. - It’s been wonderful seeing you again. - Goodbye. - Goodbye for now. - See you around.

- Let’s get together again. - Please drop me a line (Please write me a letter.) - I hope we’ll meet again. - Take care. - Have a nice day. - Have a pleasant weekend. - Have a nice working day. - Have fun. - Have a good time. - All right. Bye now.

- So long. Conversation 2 A: May I introduce myself? My name‘s Susan. I’m a teacher. B: It’s nice to meet you. I’m Dr. Somchai. I work at Nakhonnayok hospital. A: I’ve enjoyed talking to you. B: Let’s go together again.

Third – Party Introductions

Third – Party Introductions Responses Response - I’d like you to meet ........................ - I’d like to introduce you to ........... - Let me introduce you to ............. - This is ........................................ (Give appropriate background information, your relationship to the two people.)

- How do you do? - It’s pleasure to meet you. - I’ m pleased to meet you. - Nice to meet you. - Hello. Good to meet you.

- It’s nice to meet you too. - It’s a pleasure to meet you. - Nice meeting you, too. - My pleasure.

Conversation 3 A: Hey, Tony. B: Oh, Hi, Max. A: This is Wasana and Jurai. They are new students in my class. They come from Ongkarak. B: I’m please to meet you all. Students: Please to meet you too.

Language Points: Affirmative Sentence: Present Simple (I do, work, like etc.) She works in the office. They talk about their project.

Subject Verb

I / We / You /They work talk use play watch do He / She / It works talks uses plays watches does

Example: I live in Nakonnayok but my sister lives in Prachinburi. We use the present simple for things that are true in general, or for things that happen sometimes or

all the times: - I walk to school. - The museum open at 09.00 a.m. and closes at 05.30 p.m. - Suda goes to work in the morning. - He does everything that she asks. We use the present simple with always/ never/ often / usually / sometimes: - We always travel by train. - I often try to fix my computer. - He washes his car every weekend. - She usually carries her note book computer to work.

Negative Sentences: Pattern: I don’t + verb (present simple negative) Example: Santa doesn’t come. The racing car doesn’t slow.

Subject Auxiliary Verb + not Verb I / You / We / They don’ t work

He / She / It doesn’t work - I drink coffee but I don’t drink tea. - The children don’t play in the park. - Mary doesn’t go to the library. - She drinks tea but she doesn’t drink coffee.

Question Sentence: Pattern: Do you..... (Present simple question)....?

Example: Do they work? Does she work?

Auxiliary Verb Subject Verb +?

Do I /we / you / they work?

Does He / she/ it work?

- Do you smoke? No, I don’t. - Do they speak English? Yes, they do. - Does she work hard? Yes, she does - Does your brother live in Bangkok? No, he does.

รูปภาพ 1.2 – The Oxford Picture Dictionary Page 8 ใบความรูท่ี 1

ใบงานที่ 1.1 Warm up: Listening and Speaking Exercise 1: Listen and repeat./ Greeting

a) Informal Greeting: Speaking to friends or to people we know well.

b) Formal Greeting: Speaking to people we don’t know or who are not close friends.

a) Informal Greeting b) Formal Greeting 1.1 A: Hi, Tom. How’s a thing? 1.1 A: Good morning. Miss Elizabeth. How are you today? B: Hi, Fine, thanks B: Good morning. Very well, thank you. 1.2 A: Hi, Elizabeth! 1.2 A: Good afternoon. Mr. Tom. How are you this afternoon? B: Oh, hello Tom. How are you? B: Good afternoon. Very well, thank you. A: Fine, thanks. And you? B: Good, thanks.

Introductions a) Informal Greeting: Meeting someone for the first time. A friendly greeting.

b) Formal Greeting: Meeting someone for the first time. A formal greeting.

a) Informal Greeting b) Formal Greeting 1.1 A: Tony, this is Anna. 1.2 A: How do you do? This is Mr. Robinson.

B: Hi, Anna. How are you? B: How do you do. My name is Miss Rose. C: Hi. Fine, thanks. It’s very nice to meet you. C: It’s very nice to meet you too.

ใบงานที่ 1.2 Reading Exercise 2: Read about the story and summarize the following questions.

Meeting and Greeting Customs People usually shake hands when they meet for the first time. When two women first meet they sometimes give one kiss on the cheek. They usually “kiss the air” Women also greet both male and female friends with a kiss. Chilean men give their friends warm abrazos (hugs) or sometimes kiss women on the cheek. The everyday greeting for friends is a handshake for both men and women. Men sometimes pat each other on the back. Men bow slightly and shake hands to greet each other. Women do not usually shake hands. To address someone with his or her full name, the family name comes first, then the first name. Finns greet each other with a firm handshake. Hugs and kisses are only for close friends and family. People shake hands when they are first introduced. Friends and family members often hug or kiss on the cheek when they see each other. In these situations, men often kiss women but not other men.

Exercise 2: Read and summary.

a) Which greetings are typical in your country?

................................................................................................................................................................... .....................................................................................................................................................................

b) Can you name a country for each greeting? No. Country Greeting 1 ................................................................ ............................................................................ 2 ................................................................ ........................................................................... 3 ................................................................ .......................................................................... 4 ................................................................ .......................................................................... 5 ................................................................ .........................................................................

ใบงานที่ 1.3 Reading Exercise 3: Complete the dialog with the expressions given.

Conversation 4: At the party.

1) Where are you from? 2) Nice to meet you. 3) I don’t think we’ve met. 4) How do you do? 5) I’m from America You: Excuse me (1)........................................................... My name is Karl Green. Guest: (2) .......................................................................... I’m Paul Jamison. You: My name is Tony Carls (3) .................................... Guest: I’m from Brazil and you? You: (4) ............................................................................... Guest: (5) .............................................................................. You: Nice to meet you too. Conversation 5: At the office. 1) How is she? 2) My name is Gerald Whitman. 3) Are you Mr. Hardy? 4) Who is that? 5) Please sit down. Visitor: Good morning (1) ....................................................... You: Yes, that’s right. Visitor: (2) .................................................................................... You: Good morning, Mr Simson. (3)..................................................................................... Visitor: I’ve a friend in London who told me to get in touch with you. You: Really? (4) ................................................................................................................... Visitor: Alexandra Green. You: Ah! Yes, of course (5) ....................................................................................................

Exercise 4: Study the conversation and then answer the questions.

Conversation 6 A: Hello. I’m Suwit. B: Hello. My name is Ratana. A: Nice to meet you. How do you do? B: How do you do? Nice to meet you too. How long have you been here? A: I’ll go home tomorrow. Oh! Is that Arkom over there? B: I’m not sure. A: Excuse me, are you Arkom? C: Yes, I am. B: My name is Ratana. Good to meet you. C: Good to meet you too. Suda told me about you. B: Oh1 She is my good friend. She told me about you too. Questions

1. How many people are there in this conversation? .................................................................................................................................................................. 2. What will Suwit do tomorrow? .................................................................................................................................................................... 3. Have Ratana and Arkom met before? ................................................................................................................................................................... 4. Does Suwit know Arkom? .................................................................................................................................................................... 5. Where do you think this conversation take place? ....................................................................................................................................................................

ใบงานที่ 1.4 Writing : Simple present Tense Exercise 5: Complete the text using words from the box.

get up go have sleep start finish have go have go

My name‘s Margaret Beech. I’m a journalist. I (1) ...............early, at quarter to six, and (2) ........................... To work at about half past six. I (3)..................... breakfast in a café near my office, and (4) .............. work at half past seven. I don’t (5)....................... a big lunch, just a sandwich and a coffee, I usually (6) ......................... work at about six o’clock. I (7) ......................... home at seven, then I (8) ............... dinner with husband. He loves cooking! After dinner we usually watch TV, then I (9) ................................ to bed early. About ten o’clock and (10)............................. for seven or eight house.

Exercise 6: Complete the sentences with the words given. All of them are negative.

cost drive go know play see sell smoke wash wear 1. He likes basketball but he ...................................... very often. 2. Suda is married but she ......................................... a ring. 3. “Have a cigarette.” “ No, thanks. I ............................................” 4. They ............................................ newspapers in 7 – 11 shop. 5. I like films but I .......................... to the cinema very often. 6. She lives near my house but I .....................................her very often. 7. He has a car but he ......................................................very often. 8. He smells because he ...................................................very often. 9. I ............................... much about politics. 10. It’s a cheap room. It ................................................. much to stay here.

Exercise 7: Use the verbs in the list to make questions.

do like do smoke cost speak go rain play have 1. A: (he) ..................... often ....................... basketball?

B: Yes, he’s a very good player. 2. A: (you) Excuse me, .......................................Chinese?

B: Yes, a little. 3. A: (you) What .............................................................?

B: I’m a secretary. 4. A: (your sister) What ...................................................?

B: She works in a shop. 5. A: (it) How often ............................................in summer?

B: Not often. It’s usually dry. 6. A: (she) ...........................................................................?

B: Yes, 20 cigarettes a day. 7. A: (you) ................................................................dancing?

B: Yes, I love it. 8. A: (they) What time .................usually ..................to bed?

B: 10 o’clock. 9. A: (you) What ..........................usually .................for breakfast?

B: Bread and coffee. 10. A: (it) How much .................................to stay at this hotel?

B: 2,000 baht a night.

ใบงานที่ 1.5 Writing Exercise 8: Complete this conversation by your own sentences.

Conversation 7 Wichai: Hi! My name is Wicahi Srisuk. Wanna: (1) ................................................................................................................. Wichai: I’m pleased to meet you. Wanna: .(2) ............................................................................................................... Wichai: I ‘m a technical student. My major is welding. I come from Ayudthaya. Wanna: .(3) ................................................................................................................. Wicahi: See you later, goodbye. Wanna: .(4) .................................................................................................................

Exercise 9: Match the questions in column A with the statements in column B by drawing a line,

then write a complete conversation.

Column A Column B 1. Could you meet on Friday, the 19 th and work on it together? 2. When is our report due, Yai. 3. What is the date today?

a) At first they said Monday, the 22 nd, but now I hear it is on Thursday. The 25 th

b) Yes, I will mark it down in my appointment book. c) It is Tuesday, the 16 th

Conversation 8 Lek: (5) .............................................................................................. Yai: (6) ............................................................................................... Lek: (7) ............................................................................................. Yai: (8) ................................................................................................. Lek: (9) ................................................................................................ Yai :(10) ……………………………………………………………..

ใบงานที่ 1.6 แบบทดสอบประจําหนวยท่ี 1 Choose the best answer. 1. Thank you very much for all your trouble. a) No matter b) Don’t mention it. c) It’s not worth it. d) Good-bye. 2. My name is Sue. How do you do? a) How do you do? b) I’m fine, thank you. c) All right. d) I’m sorry. 3. I’d like you to meet my husband. a) All right. b) I’m pleased to meet you. c) Hello. d) Thank you. 4. See you tomorrow. a) There you are. b) Sure enough. c) Yes, see you. d) You will. 5. A: Good night. B: Yes, ....................................... a) sleep tight. b) sleep it off. c) sleep on it. d) sleep over. 6. A: Bye, then. Thanks for the meal. B: Yes. ......................................... a) All the way. b) All too soon. c) All the best. d) All over. 7. A: Hello. How are you doing? B: ............................................... a) I’m fine. And you? b) Thank you. c) How are you too? d) I don’t know. 8. A: Bye. Have a good evening. B: ........................................... a) Bye. Have a nice day. b) Good night. c) Thank you. You, too. d) Happy, too. 9. A: It’s my birthday today! B: ............................................. a) Congratulations b) Thanks a lot. c) Help yourself. d) Well done. 10. You can ............................. that he is angry. Look at the expression. a) say b) speak c) tell d) talk

Answer Exercise 1 ฝกการฟงจากผูสอนอานและผูเรียนอานตาม Exercise 2 แนวคําตอบ a) Greeting are typical in my country by wai and say ‘Sawasdee Kha” หรือ ‘Sawas Krab” b) 1. America – Shaking hand 2. Finland - Hugging 3. England – Kissing 4. Japan – bowing slightly 5. New Zealand – Nose to Nose Exercise 3: Conversation 1 1. How do you do? 2. I don’t think se’ve met. 3. Where are you from? 4. I’m from America. 5. Nice to meet you. Exercise 3: Conversation 2 1. My name is Gerald Whitman. 2. Are you Mr. Hardy? 3. Please sit down 4. Who is that? 5. Please sit down. Exercise 4: Conversation 3 แนวคําตอบ 1. three 2. He will go home tomorrow. 3. No. 4. Yes. 5. in an office Exercise 5: 1. Get 2. go 3. have 4. start 5. have 6. finish 7. go 8. have 9. go 10. sleep Exercise 6: 1. can’t play 2. wears 3. smoke 4. sell 5. go 6. don’t see 7. doesn’t 8. washes 9. know 10. costs Exercise 7 1. How often do you play basketball? 2. How do you like Chinese?

3. What do you do? 4. Where does she work? 5. How often do you go in summer? 6. How many cigarettes do you smoke? 7. How do you like dancing? 8. What time do you usually go to bed? 9. What do you usually have for breakfast? 10. How much does it cost to stay at this hotel? Exercise 8: 1. My name is Wanna Jaingam. 2. Please to meet you too. 3. I’m a commercial student. My major is accounting. 4. See you, goodbye. Exercise 9: 5. What is the date today? 6. It is Tuesday, the 16th 7. When is our report due, Yai. 8. At first they said Mandy, the 22nd, but now I hear it is on Thursday, the 25 th

9. Could you meet on Friday, the 19th and work on it together? 10. Yes, I will mark it down in my appointment book. แบบทดสอบ 1. = b 2. = b 3. = b 4. = c 5. = a 6. = c 7. = a 8. = c 9. = a 10.= c

Unit 2

Numbers, Days and Dates

รูปภาพที่ 2.1: Ready to Go 1 (สํานักพิมพ Longman) Page 60

Unit 2 Numbers, Dates and Times

สาระสําคัญ การเรียนรูเกี่ยวกับสํานวนการเรียกวนั, เวลาในชวงตาง ๆ ของวัน ยังมีการใชวนั, เวลาในการถาม การตอบของบทสนทนาเปนสิ่งที่มักจะพบเหน็บอยในชีวิตประจําวัน นับวาเปนสิ่งสําคัญที่ผูเรียนตองศึกษาใหเขาใจสําหรับใชในการสื่อสารไดถูกตองตามหลักไวยากรณ จุดประสงคการเรียนรู ความรู

1. ฟงและเขาใจคาํศัพท สํานวน ประโยคเกีย่วกับวนั เวลาตาง ๆ ได 2. ฟงและตอบเกีย่วกับคําศัพท สํานวน ประโยคเกีย่วกับวนั เวลาตาง ๆ ได 3. ฟงและใชคําศพัท สํานวน ประโยคเกีย่วกบัวัน เวลาตาง ๆ ในการเขียน Timetable ของตนเอง

ทักษะ 4. เขาใจและบอกคําศัพท สํานวน ประโยคเกีย่วกับวนั เวลาตาง ๆ 5. เขาใจและใชคาํศัพท สํานวน ประโยคเกีย่วกันวนั เวลาตาง ๆ ในการพูดบทสนทนา 6. บอกและใชคําศัพท สํานวน ประโยคเกีย่วกับวัน เวลาตาง ๆ ในการเขียน Timetable ของตนเอง

คุณธรรม 7 สรางจิตสํานึกของการตรงตอเวลาในการฝกทักษะการฟงเกี่ยวกับ Days and Dates นําสูการ

บูรณการเกีย่วกับความมวีินยัของการพูดและเขียนเวลาไดตรงตามที่กําหนด สาระการเรียนรู 1. Language Focus 1.1 Telling Dates: 1.1.1 We use ordinal numbers to tell dates 1.1.2 We use cardinal numbers for years by dividing the figures of year into two parts. The first two figures go together as the second one. 1.2 Tell the times: 1.2.1 Saying the time 1.2.2 Asking the time 1.3 Preposition of Time: 1.3.1 at / on / in

1.3.2 We use at in these expressions. 1.3.3 We do not use /at / on / in / before last, next, this, every 1.3.4 We use in with in a few minutes / in tow months / in two months’ time 1.3.5 Asking the dates and times 2. Worksheet 2.1 ใบงานที่ 1 Warm up จับคูเวลากับนาฬิกาทีก่ําหนดให 2.2 ใบงานที่ 2 Listening ฟงบทสนทนาแลวเติมคําลงในชองวางจากคําที่กําหนดให 2.3 ใบงานที่ 3 Speaking จับคูฝกพูดบทสนทนาตอบคําถามเกี่ยวกับเวลา 2.4 ใบงานที่ 4 Reading ฝกการอานบทสนทนาและตอบคําถามใหถูกตอง 2.5 ใบงานที่ 5 Writing เลือกคําบุพบทใหถูกตองตามวลีที่กําหนดให เนื้อหาสาระ Practice Conversation 1 1.1 A: Are you going to have a party for your birthday?

B: No, I’m going to go out with a friend. 1.2 A: Are you going to go to a restaurant?

B: Yes. We’re going to go to Nick’s Café. 1.3 A: When’s your birthday?

B: October 9th. Conversation 2 2.1 A: What time is it? B: It’s 3:15. A: 3:15? Uh-oh. I’m late. Bye. B: Bye. See you later. 2.2 A: What time does the post office open?

B: At 8:30 a.m. A: And when does it close? B: I’m not sure. At noon, I think. A: At noon? That’s great!

2.3 A: When does school start? B: In October. A: And when does it end? B: In March.

Conversation 3 Manager: Good morning. Oh, it’s 9:00. You‘re right on time. That’s great. Worker: Thank you. Manager: Well, let me tell you a little about the job. Your shift starts at 6:00. Worker: 6:00 a.m.? Manager: Well, actually no. At 6:00 p.m. Worker: Good. No problem. Language Points: 1.1 Telling Dates:

1.1.1 We use ordinal numbers to tell dates by using “the” 1st = first 11th = eleventh

2nd = second 12 th = twelfth 3rd = third 13th = thirteenth 4th = fourth 14th = fourteenth 5th = fifth 20th = twentieth 6th = sixth 21st = twenty – first 7th = seventh 22nd = twenty – second 8th = eighth 23rd = twenty – third

9th = ninth 30th = thirtieth 10th = tenth 31st = thirty - first Note: We say “of” between date and month. Do not write - st / - nd / - rd / - th / after dates. 1 January = the first of January 22 May = the twenty – second of May 16 November = the sixteenth of November

1.1.2 We use cardinal numbers for years by dividing the figures of year into two parts. The first two figures go together as the second one. 1895 = eighteen ninety – five 1975 = nineteen seventy – five 1900 = nineteen hundred 1990 = nineteen ninety

2000 = two thousand 2001 = two thousand and one 1.2 Telling the timers 1.2.1 Saying the time

1.2.1.1 We use past, after (from 1 minute to 29 minutes) and to (from 31 minutes to 59 minutes) in conversation.

09:20 = twenty past nine / twenty after nine 09:50 = ten to ten

Note: For 30 minutes, we say half and for 15 and 45 minutes, we use a quarter. 10:30 = half past ten 10:15 = a quarter past ten 10:45 = a quarter to eleven 1.2.1.2 We say only figures (without past, after, to) to express more formally. 10:30 = ten thirty 10:10 = ten ten 10:45 = ten forty – five 1.2.1.3 We use a.m. or A.M. for morning and we use p.m. or P.M. for afternoon. We use the 24 hour clock for itineraries and timetable. 10:00 a.m. = ten a.m. 10:00 p.m. = ten p.m. 10:45 = ten forty – five 13:15 = thirteen fifteen 14:05 = fourteen oh five 21:45 = twenty – one forty - five 1.2.2 Asking the time * At the moment of speaking

Asking Answering

- What‘s the time? - What time is it? - Have you got the time? - Do you have the time, please? - Sorry, could you tell me the time, please? - Do you know what the time is?

- It’s nine o’clock. - It’s five twenty by my watch.

* Asking about the train/ bus / flight schedules Asking Answering

- When does the train from Yala arrive? - Could you tell me when the flight from Phuket arrives?

- It arrives at 6 o’clock in the morning. - It arrives at two o’clock.

1.3 Preposition of Time 1.3.1 at / on / in - They will arrive at 10 o’clock. - They will arrive on Sunday. - They will arrive in June. 1.3.2 We use at in these expressions. at night at the weekend at Christmas at the moment at present at the same time 1.3.3 We do not use at/on / in/ before last, next, this, every. - I’ll see the best next Monday. - He’ll go to the office this morning. 1.3.4 We use in with expressions: /in a few minutes/in two months / in two months’ time - The manager has gone away. He’ll be back in a week. - Susan will be here in a moment. - The class will start in a few minutes. 1.3.5 Asking the dates and times. We use “What time / date.............?” for the specific time and “When ...................?” for unspecific time in questions.

- What time are you going out this evening? - At 6 o’clock. - When is the next party? - It’s in July.

รูปภาพที่ 2.2: THE OXFORD PICTURE DICTIONARY – Page 18

ใบความรูท่ี 2 Wrap up 1. The Time 8:00 = eight o’clock 10:05 = ten – oh – five 2:15 = two fifteen 11:30 = eleven thirty 6:45 = six forty – five 2. Months of the year January February March April May June July August September October November December 3. Days of the week Sunday Monday Tuesday Wednesday Thursday Friday Saturday 4. Time expressions a year today tomorrow tonight next week next month next summer 5. It’s for days, dates, and times

Days A: Is it Tuesday or Wednesday? B: It’s Wednesday.

Dates It’s September 21, 2004.

Times A: Is it 5:00? B: No, it isn’t. It’s 4:45.

6. in, on, at, from, and to for telling time

in + month - School starts in September. - It ends in June.

on + day - We have class on Monday. - We don’t have class on July 4.

at + time - The post office opens at 9:00. - It closes at 5:00.

from ______ to _________ (months, days, times)

- School is open from September to June. - He works from Monday to Friday. - We have class from 9:00 to 10:30.

7. Information questions about time

What time is it? It’s 2:15.

What time/ When does the restaurant open? At 11:00.

ใบงานที่ 2.1 Warm up: What time is it? Exercise 1: Match these clocks with the time expression.

รูปภาพที่ 2.3 :

ใบงานที่ 2.2 Listening Exercise 2: Listen to the conversation and complete the sentences with the given words. What is take a taxi Is that what’s the date on Sunday Tuesday the seventh quarter past two what time half past four my watch

รูปภาพที่ 2.4: English for Communication 1 Page 165

Exercise 3: Listen and choose the best answer. Days and Dates 1. The date today is Friday the twenty – first ............. April. a) in b) of c) on d) by 2. My friends and I went to a shopping mall ................ Saturday. a) in b) of c) on d) at 3. I am going back to my country ............. I only have six more weeks in the U.K. a) two months after b) the month after next c) two months later d) the month before last 4. We went on holiday to Malaysia, last year ............. June. a) in b) at c) on d) for 5. I don’t remember his birthday. I think it’s .............. the middle of October. a) in b) at c) on d) for 6. We usually have a party .......... the end of December for Christmas. a) in b) for c) on d) at 7. Last year my birthday was ................................ a Friday. a) in b) on c) at d) for 8. Mother’s day is usually on ............................... in March. a) the first Sunday b) Sunday number one c) Sunday the first d) number one Sunday 9. Today is the sixteenth so the fourteenth was................... a) two days after b) yesterday yesterday c) before two days d) the day before yesterday 10. Easter is usually between the middle of March and the end ......... April. a) in b) on c) of d) for

ใบงานที่ 2.3 Speaking Exercise 4: You and your partner are going to have conversations from the situation given below. A: You are meeting guest of the company at the airport / railway station. B: You are the officer at the airport/railway station.

ใบงานที่ 2.4 Reading Exercise 5: Read the passage and tell the time of each situation. I get up every morning during the week at the same time. The alarm clock rings at about seven – thirty and I get up at once. I take a shower, get dresses and ready for breakfast at about eight o’clock. At eight – thirty, I leave home and I always walk to school. If the weather is bad I will take a bus. I arrive at school about ten minutes to nine. The school begins at nine o’clock. At a quarter to eleven we have a short intermission. We have lunch from one o’clock to two o’clock and the schools ends at three – thirty. I go home at once and arrive home at about five or ten minutes to four. At six – thirty every night we have dinner. I study my lessons, read a book, play games, listen to the music on the radio, and watch television after dinner. I go to bed at about half past ten.

รูปภาพที่ 2.5: English for Communication 1 Page 170

ใบงานที่ 2.5 Writing Exercise 6: Complete the sentences by using at / on / in + one of the expressions given. Exercise 7: Choose words and write the words on the line. 1. A: What time _________ (is / does) it? B: 4:30. 2. A: What time _________ (is / does) the post office close? B: At 5:00. 3. A: _________ (When / What) is that store open? B: From 9:00 a.m. to 9:00 p.m.

4. A: _________ (When / What) time does your class start? B: At 8:00 a.m. 5. A: When ____________ (is / does) your English class? B: On Monday and Wednesday Exercise 8: Put the words in order to make a question. 6. the / is / open? / when / supermarket ................................................................................................................................................................... 7. when / the / supermarket / open? / does ................................................................................................................................................................... 8. close? / does / the / what time / bank ................................................................................................................................................................... 9. when / the / bank / open ?/ is ................................................................................................................................................................... 10. your / is / when / birthday? ...................................................................................................................................................................

รูปภาพที่ 2.6: NEW INTERCHANGE INTRO Page 28

ใบงานที่ 2.6 แบบทดสอบประจําหนวยท่ี 2- Pre test

1. Saturday and Sunday are .......................................... a) weekday b) weekend c) holiday d) vocation 2. See you ......................................... Wednesday. a) in b) on c) at d) to 3. The work starts ..................................... 9.00. a) in b) on c) at d) to 4. What is the personal time? a) at office hour b) at lunch time c) on weekend d) on company time 5. We will meet again next month. This month is the sixth so next month is ............................... a) August b) May c) October d) July 6. What time is it? a) It ‘s seven point fifty – five b) It’s five to eight. c) It’s five by eight. d) It’s seven point five five. 7. A: ........................................................................... B: It’s twelve midnight. a) What is it? b) What it is? c) What time is it? d) What is its time? 8. 3/5/07 (Write in the British Style) a) the third of May two thousand seven b) March the fift6h two thousand seven c) third May two thousand seven d) March fifth two thousand seven 9. Harry Potter will show at 1.00 p.m. a) one o’clock in the afternoon. b) one o’clock in the morning. c) one o’clock in the evening. d) one o’clock at night. 10. I went home at half past six in the evening yesterday. Can you tell me what time is it? a) 6.15 a.m. b) 6.00 p.m. c) 6.45 a.m. d) 6.30 p.m.

แบบทดสอบประจําหนวยท่ี 2- Post test 1. ............................. is the seventh month of the year. a) July b) June c) September d) May 2. I’ll have my class .................................. May. a) in b) on c) at d) to 3. It’ a quarter past six. What time is it? a) 6:45 b) 7:45 c) 6:15 d) 7:15 4. What are the hours? a) On Sundays and holidays. b) On weekend. c) Eight to five and an hour for lunch. d) On festival day. 5. My teacher expects me to come to study ......... time. a) in b) on c) by d) at 6. A: ..................... time is it? B: It’s noon. a) What b) When c) Where d) Who 7. 3/5/07 (Write in the American Style) a) the third of May two thousand seven b) March the fift6h two thousand seven c) third May two thousand seven d) March fifth two thousand seven 8. My birthday is on April in the second nineteen fifty – eight. (Write in the number) a) 02/05/1958 b) 03/05/1958 c) 02/04/1958 d) 03/04/1958 9. A quarter to seven. What time is it? a) 6.45 b) 7.45 c) 6.15 d) 7.15 10. I was born ....................................18 March 1985. a) in b) on c) at d) to

Answer Exercise 1: ใบงานที่ 2.1 (Warm up) a. = b. = c. = d. = e. = f. = g. = h. = i. = j. = Exercise 2: ใบงานที่ 2.2 (Listening) 1. 2. 3. 4. 5. 6. 7. 8. 9. 10. Exercise 3: ใบงานที่ 2.2 (Listening) Tape script 1. The date today is Friday the twenty – first of April. 2. My friends and I went to a shopping mall on Saturday. 3. I am going back to my country the month after next. I only have six more weeks in the U.K. 4. We went on holiday to Malaysia, last year in June. 5. I don’t remember his birthday. I think it’s in the middle of October. 6. We usually have a party at the end of December for Christmas. 7. Last year my birthday was on a Friday. 8. Mother’s day is usually on the first Sunday in March. 9. Today is the sixteenth so the fourteenth was the day before yesterday. 10. Easter is usually between the middle of March and the end of April. Exercise 4: ใบงานที่ 2.3 (Speaking) 1. 2. 3. 4. 5.

Exercise 5: ใบงานที่ 2.4 (Reading) Situation 1: Situation 2: Situation 3: Situation 4: Situation 5: Situation 6: Situation 7: Situation 8: Situation 9: Situation 10: Exercise 6: ใบงานที่ 2.5 (Writing) 1. 2. 3. 4. 5. 6. 7. 8. 9. 10. Exercise 7: ใบงานที่ 2.5 (Writing) 1. is 2. does 3. When 4. What 5. is Exercise 8: ใบงานที่ 2.5 (Writing)

6. When is the supermarket open? 7. When does the supermarket open? 8. What time does the bank close? 9. When is the bank open? 10. When is your birthday? แบบทดสอบ- ใบงานที่ 2.6 (Pre - test) 1. b 2. b 3. c 4. c 5. d 6. b 7. c 8. a 9. c 10. d แบบทดสอบ- ใบงานที่ 2.6 (Post - test) 1. b 2 a 3. a 4. c 5. b 6. a 7. c 8. c 9. b 10. a

Unit 3 Advices and Making Suggestions

รูปภาพที่ 3.1 : Workplace plus 4 page 58

Unit 3 Asking and Making Suggestions

ในชีวิตประจําวันอาจพบปญหา ส่ิงที่ตัดสินใจไมได อาจตองขอความเหน็หรือขอเสนอแนะจากผูอ่ืน ในทางกลับกันอาจตองเปนฝายใหความเห็น คาํแนะนําผูอ่ืน ผูเรียนควรไดมีการศึกษาการพูดขอความเห็น การเสนอแนะอยางเหมาะสม จุดประสงคการเรียนรู ความรู

1. ฟงและเขาใจคาํถามและคําตอบเกี่ยวกับการเสนอแนะ 2. ฟงและจับใจความสําคัญเกี่ยวกับคําถาม คําตอบเกี่ยวกับการเสนอแนะ

ทักษะ 3. ปฏิบัติการพูดบทสนทนาเพือ่สอบถามขอเสนอแนะไดถูกตอง 4. ปฏิบัติการอานและเลือกใชขอความสนทนาแสดงความเห็นวาเหน็ดวยหรือไมเห็นดวยไดอยาง

เหมาะสม คุณธรรม

5. สงเสริมลักษณะนิสัยของความรวมมือทีด่ีในการสอบถามขอมูลหรือขอเสนอแนะจากผูอ่ืนไดอยางถูกตองและเหมาสมตามสถานการณ

เนื้อหาสาระ 1. Language Focus

1.1 Asking and Making Suggestions 1.2 Accepting or Refusing Suggestions

2. Worksheet 2.1 ใบงานที่ 1: Warm up: Speaking ทําแบบฝกหัดจับคูประโยคคําแนะนําที่เหมาะสมกับ

รูปภาพ 2.2 ใบงานที่ 2: Listening: ทําแบบฝกหัดฟงและจับใจความเกี่ยวกบัการตอบรับหรือการปฏิเสธ 2.3 .ใบงานที่ 3: Speaking: ทําแบบฝกหัดอานบทสนทนา จับใจความสําคัญและตอบคําถาม 2.4 ใบงานที่ 4: Writing: ฝกการเขียนประโยคเกี่ยวกับคาํแนะนําที่สอบถามได ขอมูลจากเพื่อนและนํามาเขียนเปนคําตอบ

Asking and Making Suggestions Asking for Suggestion Making suggestions

- Can (Could) you open the door, please? - What‘s the matter? - Will you let me do that job - Is it all right if I leave one hour early? - I wonder if I can smoke here. - Can I hand my report tomorrow? - May I use your phone? - Do you mind if I open the window? - Would it possible for me to do that job? - Do you think you could lend me 100 baht? - May I borrow your pen? - How do I start the computer?

- Yes, sure. - Yes, is that all right? - Go ahead. - Certainly you can. - No, I afraid you can’t. - Yes, but don’t be too late. - I’ m afraid not. - Of course. - I’m sorry ............................................................ - I’m afraid I can’t. - Yes, here you are. - I don’ know. Let’s read the directions.

1. Asking and Making Suggestions

1. ∗ How about eating out this evening?

∗ What about going to see the doctor?

2. ∗ Shall we go shopping?

∗ Shall we take a trip to Hua Hin this weekend?

3. ∗ Why don’t we go shopping?

∗ Why don’t we take a trip to Hua Hin this weekend?

4. ∗ Why don’t you go to see the doctor? ∗ Let’s go shopping?

∗ Let’s take a trip to Hua Hin this weekend.

∗ Let’s go to see the doctor.

5 ∗ Let’s go to the cinema, shall we?

∗ Let’s go to the party, shall we?

6. ∗ Can you open the door, please?

∗ Could you take your umbrella?

∗ Can I have these postcards, please?

2. Accepting and Refusing Suggestions

2.1. Accepting Suggestions

∗ Good idea. ∗ That’s a good idea.

∗ Great. ∗ That’s great.

∗ Yes, that’s marvelous. ∗ O.K. Let’s go.

2.2 Refusing Suggestions

∗ No, I don’t think so.

∗ Well, I don’t really want to.

∗ I’m sorry. I can’t do it.

Practice Conversation 1 A: Excuse me; could you give me some change for the machine? (Asking for suggestions) B: Yes, is that all right. (Making suggestions) A: That‘s fine. Thank you very much.

Conversation 2.

A: Oh, no!

B: What’s wrong? (Asking for suggestions)

A: The cash register is out of order.

B: Let’s call Ms. Rivas. (Making suggestions)

A: Good idea.

Conversation 3 A: We’re free this evening. Let’s have a swim, shall we? = Asking for suggestions B: I’m sorry. I’ve got a headache. = Making suggestions

ท่ีมาของรูปภาพที่ 3.2: Ready To Go 1 (สํานักพิมพ Longman) Page 38

Language Points: Present Continuous Tense

1. Imperatives: Pattern: Verb / Do + not + Verb

- Take a pill every hour. - Don’t work hard.

- Drink lots of juice. - Don’t stay up late.

2. Giving Advices

Activities for actions: Pattern: Verb to be + Verb ---ing

driving running swimming reading dancing studying

playing tennis going to the movies watching TV. shopping walking

Pattern: What + doing

2.1 A: What are you doing?

B: I am sleeping.

2.2 A: What’s Debbie doing right now?

B: She is driving.

Example 2.3:

A: What are James and Anne doing?

B: They are having lunch.

3. Making Suggestions: การใหคําแนะนําหรือขอเสนอแนะมีรูปแบบหลายรูปแบบ เชน

Pattern 1: 1.1 What about + Verb+ ing?

1.2 How about + Verb+ ing?

Pattern 2: Shall / Should / we + Verb ชองที่1?

Pattern 3: Why don’t we/ you + Verb ชองที่ 1?

Pattern 4: Let’s + Verbชองที่ 1

Pattern 5: Let’s + V1 ……, shall we? เรียกวา Invitation คือ การกลาวเชิญหรือชวนใหผูอ่ืนมารวมกิจกรรม

ใบความรูท่ี 3

Warp up

Conversation 4

A: I have a stomachache. (Problem health)

B: Don’t eat any heavy food today. (Giving advice)

* If you use “verbs of activities”, it is about telling the time*

Conversation 5

A: What time is it in Los Angles? (About the time)

B: It’s four o’clock in the morning. (About the time)

A: What are people doing here? (Asking for action)

B: They‘re sleeping. (Answering for action)

Conversation 6 A: We’re late. What shall we do? (Asking for suggestion) B: Let’s hurry! (Making suggestion)

Conversation 7 A: What shall we do this evening? (Asking for suggestion) B: Let’s stay at home. (Making suggestion) A: I don’t think so. I would rather eat out. (Refusing suggestion)

Conversation 8 A: What shall we do this weekend? (Asking for suggestion) B: Shall we take a trip to Hua Hin? (Making suggestion) A: That’s a good idea. (Accepting suggestion) * Answering by using “Sure. / OK. / I’m sorry. /* - Sure. = to express willingness - OK. = to express willingness - I’m sorry = to apologize.

ใบงานที่ 3.1 Warm up. Speaking Exercise 1: Match each picture with the appropriate suggestions.

Suggestions: a) You could read the directions. b) You can call Mr. Tom; please His number is 555-2144. c) You should turn the key. d) You can press the button e) You could unplug the machine, please.

ท่ีมาของรูปภาพที่ 3.3 : Ready To Go 1 (สํานักพิมพ Longman) Page 37

ใบงานที่ 3.2 Listening

Exercise 2: Choose the best accepting or refusing suggestions from listening.

Asking for Suggestions Accepting or Refusing Suggestions 1. What could you do if I go to the canteen? 2. What should you do if I have more detail? 3. May you write me a letter? 4. May you copy it for me? 5. Would you mind if I smoke here? 6. Can I get me a bottle of water? 7. Would you mind lending me your computer? 8. Can I speak slowly? 9. Would you mind going shopping with us? 10. Can you fill in the form?

1.................................................................................... 2................................................................................... 3................................................................................... 4................................................................................... 5.................................................................................... 6.....................................................................................7.................................................................................... 8.................................................................................... 9.................................................................................... 10..................................................................................

ใบงานที่ 3.3 Reading Conversations Exercise 3: Read the following conversation and write 5 things that are suggested.

Conversation 9 A: Excuse me; could you give me some change for the machine? B: Yes, is that all right. A: That’s fine. Thank you very much. Conversation 10 A: Excuse me. B: Yes. A: Can I have a knife, please? B: Oh, sure. Can I get you anything else? A: Can I some water, please? B: Sure. A: Thanks. Conversation 11 A: May I have a bottle of wine, please? B: Red wine or White wine, please? A: I prefer White wine. B: Do you want French wine, German wine or Italian wine? A: French wine, please. B: This is a very good one, sir. A: Thank you. Answer: 1............................2..............................3...............................4............................5..............................

Exercise 4: Read this conversation and answer the questions.

Conversation 12 Henry: Can I have permission to leave the office early today, sir? Boss: Well, it would be rather convenience. Is it something important? Henry: Yes, it’s important. Boss: Do you mind if I ask what is it? Henry: I want to get a good seat for dinner tonight. It is my first date. Boss: I’m sorry but I can’t allow you to leave early. It wouldn’t be fair. (Five minutes later) Henry: Excuse me, sir. I wonder if I could leave early because I’ve been suddenly ill. Boss: ................................................................................................................................................................. Questions

1. What permission does Henry ask? .................................................................................................................................................................. 2. Why does Henry want to leave the office early? .................................................................................................................................................................. 3. What question does the boss ask? .................................................................................................................................................................. 4. Why does Henry come to see the boss again? .................................................................................................................................................................. 5. What do you think the boss will say to Henry at the end of the conversation? .................................................................................................................................................................

ใบงานที่ 3.4 Writing Exercise 5: Make suggestions about what your friends say.

1. You are carrying a lot of things. You want me to open the door for you. What should you say? ..............................................................................................................................................................

2. We are having dinner. You want me to pass the salt. What should you say? ..............................................................................................................................................................

3. You want me to turn off the computer? What should you say? ...............................................................................................................................................................

4. You want to borrow my flash drive. What should you say? ............................................................................................................................................................

5. You are in my house. You want to use my telephone. What should you say? ...........................................................................................................................................................

6. Your friend says “It’s raining hard outside” What should you say? ............................................................................................................................................................

7. Your friend says “I’m hungry.” What should you say? .........................................................................................................................................................

8. Your friend says “I’ve got a headache.” What should you say? ........................................................................................................................................................

9. Your friend says “My hammer is broken.” What should you say? ........................................................................................................................................................

10. Your friend says “Today is my birthday.” What should you say? ........................................................................................................................................................

ใบงานที่ 3.5 แบบทดสอบประจําหนวยท่ี 3 Choose the best answer for situation 1 – 4

a. You’d better take a road map in case you get lost. b. You‘d better stay at home and relax. c. You’d better take my address in case you need something. d. You’d better take food and water in case you can’t find any.

Situation 1. Jennifer, an Australian teacher is traveling by car in Bangkok. She doesn’t know the road very well. / Answer = .......... Situation 2. Your friends are going to the seaside in Pattaya. The weather report says it’s going to have a storm soon. / Answer = ....... Situation 3. On a train to Suratthani, you make friends with a man from New Zealand. Before you depart, you give him your name card. / Answer = Situation 4. Your friends are going camping to Kao Yai which is far from the city. / Answer = ..........

Choose the best answer for the question 5 – 8 a. Pour it into a glass and mix some ice. b. Put it in a microwave oven for 20 minutes. c. Put in a cone. d. Cook it in a saucepan with water and then drain it.

Question 5. What should I do with this ice cream? Answer: ..... Question 6. What should I do with this frozen dinner? Answer....... Question 7. What should I do with this frozen broccoli? Answer....... Question 8. What should I do with this orange juice? Answer.......

Choose the best answer for the question 9 -10 a. How about eating out this evening? b. What about going to see the doctor? c. Let’s hurry. d. I’m sorry. I don’t like it.

Question 9. A: We’re late. What shall we do? B: What should you say?.......................... Question 10. A: Let’s go dancing. B: What should you say? ........................

รูปภาพที่ 3.4 ชุดการเรียนภาษาอังกฤษเพือ่การสื่อสาร 1 ของสํานักมาตรฐานการอาชีวศึกษาและวิชาชพี

What should I do with this ice cream?

What should I do with this frozen dinner?

What should I do with this frozen broccoli?

What should I do with this orange juice?

Answer Exercice 1 1. = e 2 = d 3 = b 4 = a 5 = c Exercise 2 Typescripts Asking for suggestions Accepting or refusal 1. What could you do if I go to the canteen? Yes, of course. 2. What should you do if I have more detail? Yes, of course. 3. May you write me a letter? No, I ‘m sorry. I don’t have it. 4. May you copy it for me? Yes, of course. 5. Would you mind if I smoke here? No, I’m sorry. I don’t have it. 6. Can I get me a bottle of water? Yes, of course. 7. Would you mind lending me your computer? Yes, of course. 8. Can I speak slowly? No, I’m sorry. I don’t have it. 9. Would you mind going shopping with us? Yes, of course. 10. Can you fill in the form? Yes, of course. Exercise 3 1 my change 2. knife 3. water 4. wine 5. French wine Exercise 4 1. To leave the office early. 2. To get a good seat for dinner tonight.

3. I’m sorry but I can’t allow you to leave early. 4. Because he has been suddenly ill 5. Thank you. Exercise 5 แนวคําตอบ 1. Would you mind opening the door, please? 2. Excuse me. Could you pass the salt? 3. Can you turn off the computer? 4. Would you mind borrowing your flash drive? 5. I’m sorry. Can I use your telephone? 6. You should bring the umbrella to go outside. 7. You should eat some soft drink before. 8. You should eat one tablet of aspirin. 9. Can I give your advice? 10. You should say “Happy birthday.” แบบทดสอบ

Situation 1. = a Situation 2. = b Situation 3. = c Situation 4 = d Question 5. = b Question 6. = c Question 7. = d Question 8. = a Question 9. = c Question 10 = d

Unit 4

Health

รูปภาพที่ 4.1 : Workplace Plus 4 page 138

Unit 4 Health

สาระสําคัญ

การบรรยายอาการเจ็บปวยเบื้องตน และการสอบถามเมื่อเห็นผูที่อยูใกลชิดมีลักษณะอาการผิดปกติ การฟงคําถามและการตอบตรงคําถามเพื่อใหไดขอมูลที่ถูกตอง เปนสิ่งจําเปนเพื่อแพทยจะทําการรักษาสําหรับใหคําแนะนํา จุดประสงคการเรียนรู ความรู

1. บอกคําศัพท สํานวน ประโยคที่เกี่ยวกับสวนตาง ๆ ของรางกายได 2. สอบถามและบรรยายอาการของความเจ็บปวยได

ทักษะ

3. พูดบรรยายเกีย่วกับคําศัพท สํานวน ประโยคที่เกีย่วกับสวนตาง ๆ ของรางกายไดถูกตอง 4. พูดและเขียนสอบถามบรรยายอาการของการเจ็บปวยเพื่อการสื่อสารได

คุณธรรม

5. เสริมสรางลักษณะนิสัยของความปลอดภยัแกผูอ่ืนที่มีอาการผิดปกติและใชดําถามและคําตอบเพื่อใหไดขอมูลที่จําเปนสําหรับแพทยทําการรักษา เนื้อหาสาระ 1. Language Focus 1.1 Parts of the Body and Health 1.2 Describing Symptoms and Inquiries about Symptoms 1.3 Past Tense / Have + Noun / Feel + Adjective / Helpful Advice 2. Worksheet

2.1 ใบงานที่ 1 Warm up ฝกการพูดบทสนทนาเกีย่วกับอวัยวะตาง ๆ ของรางกายตามตัวอยาง 2.2 ใบงานที่ 2 Listening ฝกทักษะการฟงและออกเสียงของคําศัพทเกี่ยวกับ Health Problems 2.3 ใบงานที่ 3 Listening and Speaking ฝกทักษะการฟงและการพูดบทสนทนาเกีย่วกบั Health

2.4 ใบงานที่ 4 Reading ฝกทกัษะการอานคาํศัพทและวิเคราะหหาความสัมพันธที่เกี่ยวของ 2.5 ใบงานที่ 5 Reading ฝกทกัษะการอานเนื้อเร่ืองและสรุปใจความสําคัญเพื่อการตอบคําถาม 2.6 ใบงานที่ 6 Speaking ฝกทักษะการพูดเกี่ยวกับวิธีดูแลสุขภาพเปนกลุม 2.7 ใบงานที่ 7 Writing ฝกทักษะการเขียนแนะนําเกีย่วกบัสุขภาพเปนกิจกรรม Pair work

Vocabularies

Words Meaning 1. AIDS (acquired immunodeficiency syndrome) 2 Influenza 3. Hypertension 4. Infectious diseases 5. Dosage 6. Expiration date 7. Treatment 8. A vaccination 9. Be injured / Be hurt 10. Get a check up 11. Extract a tooth 12. Specialist

-a medical condition that results from contracting the HIV virus - flu - high blood pressure - a disease that is spread through air or water - how much medicine you take and how many times a day you take it - the last day the medicine can be used - something you do to get better - an injection that stops a person from getting a serious disease - get injured / get hurt - to go for a medical exam - to pull out a tooth - a doctor who only treats specific medical problems

Expressions - That’s too bad. - I’m sorry to hear that. - I hope you feel better soon. - Take a pill every hour. - Drink lots of juice - Don’t work too hard - Don’t stay up late. - I feel sick - I feel better - I don’t feel well. - Go to bed. - Use some muscle cream. - Don’t drink coffee. - Don’t go to work - Don’t exercise this week. - Take a hot bath. - Stay in bed. - Don’t drink soda. - Eat a lot of vegetables - Don’t go to bed late. - I can’t loose weight. - Staying in bed. - Drinking fluids. - Getting physical.

Talk about health problems - I have a stomachache - I’m homesick. - I have a sore throat. - I have the flu. - I have a backache - I have an earache - I have a headache - I have a toothache - I have a cold - I have a cough - I have a fever - I have a sore eyes - Be in shock - I have a heart attack. - Get an electric shock. - Burn yourself Giving instructions / Helpful advice - Take / Don’t take ............................ - Go to / Don’t go to ......................... - Eat / Don’t eat ................................ - Do / Don’t do ................................ Practice Conversation 1 A: Point to your neck. B: This is my neck. A: These are my feet. Conversation 2 A: What’ wrong? Do you have a headache? B: No, I don’t A: Do you have an earache? B: Yes, I have an earache. A: That’s too bad. Conversation 3 A: How do you feel today? B: I feel fine, thanks. What about you? A: I feel really terrible. I have a headache. B: I’m sorry to hear that. Conversation 4 A: I can’t sleep at night. B: Get up and do some work. A: Don’t drink coffee in the evening.

Language Points: 1. Past tense for problem health * burn - burned * drown - drowned * swallow – swallowed * overdose – overdosed * choke – choked * have - had * get – got * bleed – bled * can’t – couldn’t * break – broke * fall – fell 2. Have + noun; feel + adjective * I have a sore throat. (Affirmative) * I feel sick. (Affirmative) * I don’t feel well. (Negative) 3. Imperatives = Verb 1 +................. (for affirmative/ for negative = Don’t + verb 1) * Don’t stay up late. * Take a pill every four hours. * Drink lots of juice. * Go to bed and sleep. * Get some exercise every day. * Call your family on the phone. * Be something fun every evening. * Don’t eat dessert. * Go out to a restaurant. * Don’t go to school this week.

ใบความรูท่ี 4 Wrap up Health Problems - a backache - an earache - a headache - a stomachache - a toothache - a cold - a cough - a fever - the flu - sore eyes - a sore throat - nasal congestion - sneeze - feel dizzy - blood nose - feel nauseous - insect bite - sprained ankle - sunburn - strep throat Helpful Advice - Go to bed and sleep. – Go out to a restaurant. – Eat some toast and drink some tea - Don’t go to school this week. – Don’t go outside. – Don’t eat dessert. - Don’t eat any heavy food today. – Take two aspirin. – Close your eyes for ten minutes. - Go home and relax. – Do something fun every evening – Get some exercise every day. - Call your family on the phone. – Go to a store and buy some food. – Stay in bed. Ex.1 – I can’t loose weight. I like dessert. Cake is my favorite food! Ex.2 – My job is very stressful. I usually work 10 hours a day an weekends. I have backaches and headache almost every day. Ex. 3 – I can never get up on time in the morning. I ‘m always late for work. I guess I’m not a morning person. Ex.4 I’m new in town, and I don’t know any people here. How can I make some friends? Expressions * A: How are you? * A: What‘s the matter? / What’s wrong? B: I am not so good, actually. B: I have a headache. * A: How do you feel? B: I feel sick. / I don’t feel well. / I feel better already. Expressing Sympathy * That‘s too bad. / I’m sorry to hear that. / I hope you feel better soon. Giving instructions & Advice * Drink eight cups of water every day. * Wear a seat belt when you are driving. * Get enough calcium. * Don’t eat any heavy food today.

ใบงานที่ 4.1 Warm up: Speaking Exercise 1: Point to the body in the picture and ask about the part of the body.

รูปภาพที่ 4.2: NEW INTERCHANGE INTRO / Page 72

ใบงานที่ 4.2 Listening: Health Problems Exercise 2: Study these symptoms and injuries and answer what it is called in English.

รูปภาพที่ 4.3 :Picture Dictionary (OXFORD) Page 78

Answer: 1. ............................................... 2. ............................................... 3. ............................................... 4. ............................................... 5. .............................................. 6. ............................................. 7. .............................................. 8. .............................................. 9. ............................................. 10. ..............................................

ใบงานที่ 4.3

Listening and Speaking Exercise 3: Listen and repeat the conversation. Conversation 5 Jee: Hey, Joo. How are you? Joo: Oh, I’m not so good, actually. Jee: Why? What’s the matter? Joo: Well, I have a headache. And a backache. Jee: Maybe you have the flu. Joo: No, I think I just miss Japan – I feel a little homesick. Jee: That’s too bad. But I think I can help. Let’s have lunch at that new Japanese restaurant. Joo: That’s a great idea. Thanks, Brian. I feel better already! Conversation 6 Santi: Hello, Ms. Sri. How are you today? Sri : I feel terrible. Santi: So, what’s wrong, exactly? Sri : I’m exhausted. Santi: Hmm. Why are you so tired? Sri : I just can’t sleep at night. Santi: OK. Let’s take a look at you. Sri : I’m going to give you some pills. Take one pill every night after dinner. Santi: OK. Sri : And don’t drink coffee, tea or soda. Santi: No soda? Sri : No. And don’t work too hard. Santi: All right. Thanks, Sri.

ใบงานที่ 4.4

Speaking Exercise 4: Practice speak to the vocabulary and find the ‘odd one out’

No. Vocabulary Vocabulary Vocabulary Answer 1 a consultant a surgeon a doctor .............................. 2 to wound to damage to injure ............................. 3 a waiting list an appointment a consultation ............................. 4 to cure to treat to heal .............................. 5 to admit to discharge to transfer .............................. 6 a wheel chair a bed a trolley .............................. 7 a Zimmer - frame a walking stick crutches ............................. 8 homeopathy massage osteopathy ............................. 9 deodorant insect bite bloody nose ............................. 10 toothbrush teenage toothpaste ............................

ใบงานที่ 4.5

Reading Exercise 5: Read the article and then complete the advice. 1. To get exercise, ................................................................................................................................... 2. To help your bones, ............................................................................................................................. 3. To help your muscles,........................................................................................................................... 4. To keep your gums healthy, ................................................................................................................. 5. To have enough energy for the morning, ............................................................................................ 6. To challenge your brain, ..................................................................................................................... 7. To help your body in many ways, ...................................................................................................... 8. To help your skin, ............................................................................................................................... 9. To floss your teeth, .............................................................................................................................. 10. To save thousand lives, ......................................................................................................................

รูปภาพที่ 4.4 : NEW INTERCHANGE INTRO Page 77

ใบงานที่ 4.6 Speaking Exercise 6: Group work / Talk about these questions and get the information. 1. Which of these ten things do you do regularly? ................................................................................................................................................................... 2. What else do you do for your health? ................................................................................................................................................................... 3. Can you think of some ways to improve your health? ................................................................................................................................................................... 4. Do you think how eating breakfast is useful? ................................................................................................................................................................... 5. Do you know how many cups of water you drink every day? ................................................................................................................................................................... 6. How long of a time – out do you take for your relaxation? ................................................................................................................................................................... 7. What do you use to keep your skin? ................................................................................................................................................................... 8. Which activities do it useful for challenging your brain? .................................................................................................................................................................. 9. What is good exercise for good health? .................................................................................................................................................................. 10. What do your bones need? ...................................................................................................................................................................

ใบงานที่ 4.7 Writing: Pair work

Exercise 7: Write about the health problems and give advice by the given phrases.

Health Problems Giving Advice

1. I can’t sleep at night. 2. I have the flu. 3. I can’t lose weight. 4. I’m homesick. 5. My job is very stressful. 6. There’s no food in the house. 7. I have a stomachache. 8. I have a backache. 9. I feel sad. 10. I have a cold.

...............................................................................

................................................................................

................................................................................

...............................................................................

..............................................................................

...............................................................................

...............................................................................

...............................................................................

................................................................................

................................................................................

Close your eyes for ten minutes.

Take two aspirin. Go out to a restaurant.

Call your family on the phone.

Go to a store and buy some food Don’t eat desserts.

Go to bed and sleep.

Don’t go outside. Go home and relax.

Don’t go to school this week

Don’t eat any heavy food today. Get some exercise every day.

Eat some toast and drink some t

ใบงานที่ 4.8 แบบทดสอบประจําหนวยท่ี 9 Choose the best answer. 1. I’ve got a ........................ headache. a) splitting b) banging c) hurting d) crashing 2. I think I’ve got a ................ I feel terribly hot. a) rage b) stroke c) fiver d) temperature 3. You don’t look well. Let me ................. your temperature. a) make b) measure c) do d) take 4. It’s nothing. It’s just a .................. headache. a) weak b) soft c) mild d) gentle 5. You look tried – you probably need a ................... night’s sleep. a) fine b) perfect c) wonderful d) good 6. My grandfather’s ninety, but he’s still ............................ the best of health. a) with b) in c) at d) for 7. I’m feeling a bit under.............. I think perhaps I’ll go and see a doctor tomorrow. a) a storm b) a cloud c) the weather d) the rain 8. I‘ve got a sore throat and a bit of a headache. I think I may be going ...... with something. a) on b) through c) along d) down 9. Before having a heart attack, many patients report feeling under........... for several days. a) the moon b) the weather c) a storm cloud d) to the top 10. Don’t worry; it’s just a flu bug that’s going round. You’ll be back............. in a day or two. a) in the air b) on the form c) on your feet d) to the top.

Answer Exercise1คําศัพทดูจากรูปภาพเกี่ยวกับอวยัวะของรางกาย Exercise 2 1. headache 2. toothache 3. earache 4. stomachache 5. backache 6. sore throat 7. nasal congestion 8. fever / temperature 9. chills 10. rash Exercise 3: ประเมินผลการพูดตามบทสนทนา Exercise 4: 1. go for a walk 2. drink orange juice 3.stretch for five minutes 4. floss your teeth 5. eat breakfast 6. read a new book 7. drink water 8. use sunscreen 9. keeps your gum 10. wear a seat belt Exercise 5: แนวคําตอบ 1. I go for a walk regularly. 2. I swim and exercise by slim shape. 3. I think taking care of your health always. 4. It gives you energy for the morning. 5. There are eight cups of water every day. 6. Take a “time – out” – a break of about 20 minutes. 7. Moisturize your skin and use sunscreen. 8. Do a crossword puzzle or read a new book. 9. Walking is good exercise, and exercise is necessary for good health. 10. Get enough calcium for your bones. Exercise 6 แนวคําตอบตามเนื้อเร่ือง Exercise 7 แนวคําตอบตามคาํแนะนําที่กําหน แบบทดสอบ 1. = a 2. = d 3. = d 4. = b 5. = d 6. = b 7. = c 8. = d 9. = b 10.= c

Unit 5 Like and Dislike

Like: listening to music / working in the yard/ washing my clothes Dislike: renting a video / inviting friends to my house / cooking a meal

รูปภาพที่ 5.1: NEW INTERCHANGE INTRO Page 88

Unit 5 Like and Dislike

สาระสําคัญ การพูดเกีย่วกบัอาหารที่ชอบ อาหารที่ไมชอบ การพูดเกีย่วกับพฤติกรรมการกิน การพูดคุยเกี่ยวกับ กิจกรรมที่เปนงานอดิเรกมกันิยมปฏิบัติกันในชีวิตประจําวันเวลาที่ตองการพักผอน ผูเรียนควรศึกษาความหมาย คําศัพท สํานวน ประโยคภาษาอังกฤษ สําหรับเปนการพูดโตตอบใหขอมูลเกีย่วกับความชอบ ความไมชอบของแตละบุคคลที่เกี่ยวของกับพฤติกรรมการกินโดยทําการสํารวจเกี่ยวกบัความชอบ ความไมชอบสําหรับการหาขอมูล

จุดประสงคการเรียนรู ความรู

1. อานออกเสียงและบอกความหมาย คําศัพทของคํานามที่นับไดและคํานามที่นับไมได 2. ฟงบทสนทนาเพื่อจับใจความจากบทสนทนาเกีย่วกับควาชอบและความไมชอบของ

ของแตละบุคคล ทักษะ

3. พูดสนทนาโตตอบเพื่อการสํารวจหาขอมูลเกี่ยวกับ ความชอบและไมชอบของแตละ บุคคลได

4. เขียนบรรยายพฤติกรรมการกินเพื่อหาขอมูลสําหรับความชอบและความไมชอบของ แตละบุคคลสําหรับเปนขอมลูอางอิงในการสํารวจ คุณธรรม

5. เกิดทัศนคติทีด่ีตอการพูดภาษาอังกฤษเกี่ยวกับการถามความชอบและไมชอบจากผูอ่ืน ทําใหมีความกระตือรือรนและสรางลักษณะนิสัยของความสนใจใฝรูในการใชคําศัพทเกี่ยวกับการพดูชอบและไมชอบของแตละบุคคล เนื้อหาสาระ 1. Language Focus 1.1 Activities Vocabulary: -ing form verb 1.2 Adverb of frequency 1.3 Using ‘some’ and ‘any’ 1.4 Countable Noun and Uncountable Noun 2. Worksheet

2.1 ใบงานที่1 : Warm up: Listening & speaking ทําแบบฝกหัดที่ออกเสียงคําศัพทของกิจกรรมที่เปลี่ยนรูปกรยิาเปนรูป – ing ใหถูตอง

2.2 ใบงานที่ 2: Listening & speaking: ฝกฟงและออกเสยีงเกีย่วกับประโยคภาษาอังกฤษที่ใช Verb / like & dislike/ โดยออกเสียงใหถูกตอง

2.3 ใบงานที่ 3: Reading ทําแบบฝกหัดฝกการอานประโยคที่ใช like & dislike และขีดเสนใตคํากริยาที่อยูในวงเล็บใหถูกตองตามหลักการใช Verb like & dislike 2.4 ใบงานที่ 4: Reading & Writing ทําแบบฝกหัดอานประโยคและพจิารณาคํากริยาที่อยูในวงเล็บโดยนํามาเขียนเปนประโยคใหมใหถูกตองตามหลักการเขียน Verb like & dislike 2.5 ใบงานที่ 5: Writing ทําแบบฝกหัดเขยีนขอมูลเปนรายงานสั้น ๆ ตามตัวอยางทีก่ําหนดใหโดยปรับขอมูลมาเปนความชอบและความไมชอบของตนเอง Vocabulary Study: Pattern: Verb + ing.

Teaching Working Drawing Copying Typing Interviewing Telephoning Speaking Listening Writing Doing Homework Sleeping Graduating Driving Marrying Gardening

รูปภาพที่ 5.2: The Oxford Picture Dictionary (สํานักพิมพ OXFORD)

Key Vocabulary: 1. Nouns 1.1. Dairy foods – cheese - milk - yogurt 1.2. Desserts – cake – cookie - ice cream – pie 1.3. Fat, Oil, Sugar – butter – candy – cream – oil - potato chips 1.4. Fruit – apple (s) – banana(s) – grape (s) – mango (es) – orange (s) – strawberry (-ies) – tangerine 1.5. Meat / Protein – bacon – bean (s) – beef – chicken – egg(s) – fish – hamburger(s) – nut(s) 1.6. Grains – bread – cereal – cracker(s) - noodles – pasta – rice – rolls – toast 1.7. Salads – fruit salad – potato salad 1.8. Vegetable – broccoli – carrot(s) – celery – lettuce – onion(s) – potato (es) – tomato (es) 1.9. Beverage – lemonade – soda – (green) tea 1.10. Other – barbecue – freezer – grocery store – mayonnaise – snacks – soup 2. Pronouns - everyone - something 3. Determiners - any – some 4. Adjectives - awful – delicious - (Japanese) style 5. Verbs - drink – get – hate – try – want 6. Adverbs of frequency always – ever – never – often – seldom – sometimes – usually 7. Prepositions at my desk – for breakfast / for the barbecue / in the salad Expressions 1. Talking about likes and dislikes:

- I love oranges. - Everyone likes potato salad. - I hate onions. - I think.............is/are delicious / awful. - ...........is/are my favorite.

2. Talking about things you need: - Do you need any............? - Yes, we need some............... - What do you need any........? - No, we don’t need any.........

3. Asking eating about eating habits: - What time do you eat breakfast / lunch / dinner? - What do you usually have for breakfast / lunch / dinner?

- Do you ever eat ............... for breakfast/ lunch / dinner? - Do you ever go to a restaurant for breakfast / lunch / dinner? - Do you always drink the same thing in the morning / afternoon / evening? - What is something you never have for breakfast / lunch / dinner?

Practice: Conversation 1 A: What do you like? B: I like some bread and some fish. A: Do you like any fruit? B: Yes, I like some bananas. Conversation 2 A: Do you ever have fish for breakfast? B: Yes, I always do. / Sometimes I do. A: Do you usually eat rice for lunch? B: No, I never do. Conversation 3 A: Do you like to eat anything special on New Year’s Day for good luck? What? B: Yes, I like to eat cake, some fruit, and beverages. And you?

รูปภาพที่ 5.3: New Interchange INTRO page IC-9 Picture 1: She dislikes having breakfast in bed Picture 2: SA likes riding a motorcycle to call.

Picture 3: Elizabeth likes speaking three languages.

Language Points 1. การใช verb / like & dislike: ตองตามดวย Verb + ing:

1. หลักการเปล่ียนรูปใหเปน – ing form 1.1 คํากริยาปกติเติม – ing ตอทายคํา เชน cook = cooking / play = playing / read = reading 1.2 คํากริยาที่ลงทายดวย /e/ ตองตัด /e/ กอนแลวเติม – ing เชน dance = dancing / take = taking 1.3 คํากริยาเสียงสัน้กอนเติม – ing ตองเติมพยญัชนะตัวทายเพิ่มอีกตัวแลวจึงเติม – ing เชน

get = getting / swim = swimming / run = running 2. Question and Answer

Verb – Do/Does Questions Replies Do you like getting up early. Yes, I do. / No, I don’t. Do we like swimming now? Yes, we do. / No, we don’t.

Does she like cooking at home? Yes, she does./ No, she doesn’t. Does he like driving to work? Yes, he does. / No, he doesn’t.

3. การใช Adverb of frequency ไดแก always / often / usually / sometimes / never / hardly ที่บอก เกี่ยวกับความถี่เพื่อมาขยายคํากรยิาสําหรับการถามและการตอบในหัวขอ like และ dislike

4. การวางตําแหนงของ Adverb of frequency สวนมากวางไว หนากริยา, หลังประธาน, หลัง Verb ชวย, หลัง Verb to be, หลัง not และ Adverb บางตัววางไวตนหรือทายประโยค

Example: A: Do you like shopping? B: Yes, I like it. 5. กลุมคํา Adverb บางตัวมักจะวางไวทายประโยคเสมอ ไดแก once / twice / three times / every Example: A: How often do you go shopping? B: Occasionally, about twice a month. Expressing Likes and Dislikes: Here are some sample phrases and sentences for1q expressing likes and dislikes. Answer - What kind? - What kind of sports? - Really? Does she like tea? - Oh. What kind does he like? - Yes, he does - No, he doesn’t -No she doesn’t - Yes, I do - No, I don’t. Asking questions – I like fruit. – Helen likes sports. – Ms. Carmer dislikes coffee.

ใบความรูท่ี 5 Wrap up: Expressing Likes and Dislikes * Here are some sample phrases and sentences for1q expressing likes and dislikes Questions: -What kind? - What kind of sports? - Really? Does she like tea? - Oh. What kind does he like? Answers: - Yes, he does - No, he doesn’t -No she doesn’t - Yes, I do - No, I don’t. Example of using: – I like fruit.

- Helen likes sports. – Ms. Carmer dislikes coffee.

Making suggestion: - All right - Good idea - Oh, yeah. - OK. Specific General Countable I am buying an orange. I like oranges because they are delicious. Uncountable I am buying some broccoli. I like broccoli because it is good for you. Note: Singular = I am buying an orange. : Plural = I am buying some oranges. * I like eating vegetables but I dislike eating dessert. * I like working by a computer but I dislike working by hand. * I like doing the lesson at home but I dislike doing the lesson at my college. * I like swimming for the exercise but I dislike running for the exercise.

ใบงานที่ 5.1 Warm up: Listen and Speaking

Exercise 1: Write the following words into people’ activities for writing ‘like’ and ‘dislike’

รูปภาพที่ 5.4: NEW INTERCHANGE INTRO Page 32 1. He ........................ playing tennis. 2. He ........................ riding a bike. 3. He ........................ running. 4. He .........................swimming. 5. The old man .................................taking a walk. 6. A man and a woman ....................dancing. 7. A woman ................driving. 8. They ...................... going to the movies. 9. He ..........................shopping. 10. She ........................ reading. 11. She ........................ studying. 12. The man .................. watching television.

ใบงานที่ 5.2 Listening and Speaking Exercise 2: Listen and repeat.

1. I like swimming in every evening. 2. She likes working her housework in every weekend. 3. He likes playing golf in every Sunday. 4. Mr. Preecha likes working in the factory. 5. Dang likes working very hare in the night. 6. My sister likes listening and watching TV. 7. My son doesn’t like going shopping but he likes singing the karaoke. 8. My father loves gardening very much. 9. Do you like enjoy eating outside? 10. How often do you like going shopping?

ใบงานที่ 5.3

Reading and Writing Exercise 3: Read the sentences and choose the answer by underlining the correct word.

1. He likes (goes / going) to the gym. 2. Suda can’t (get / getting) up early. 3. Mary like (gardens / gardening) every weekend. 4. They enjoy (go / going) to the cinema most weekends. 5. Elizabeth loves (driving / drives) whenever he can. 6. They like (spending / spend) a lot of time with his children. 7. She prefers (watching / watch) the game show in TV. 8. I don’t like (cooking / cook) at my house. 9. He quiet enjoy (sleep / sleeping) alone. 10. She doesn’t always (works / working) for her students.

ใบงานที่ 5.4 Writing Exercise 4: Rearrange the words in the right order. (Restaurant language) 1. I / Could / please? / for / have / four, / a / table/ ........................................................................................................................................................ 2. two / of / do / wines / these / recommend? / which / you / ......................................................................................................................................................... 3. bill / please? / the / Could / I / have / ......................................................................................................................................................... 4. business. / keen / I’m / very / traveling / on / not / on ......................................................................................................................................................... 5. presentations. / quite / I / giving / like / ......................................................................................................................................................... 6. to / take / can’t / I / have. / having / work / stand / ........................................................................................................................................................ 7. need / bread / I / fish / some / some / and / ......................................................................................................................................................... 8. any / you / fruit / do / need / ? / ......................................................................................................................................................... 9. some / need / hamburger / we / meat. / .......................................................................................................................................................... 10. I / Yes. / some / bananas. / need. .......................................................................................................................................................... Exercise 5: Write the short paragraph about ‘like’ and ‘dislike’ in you own words. Sirida likes reading the novel and listening radio. She doesn’t like going to the beach. She likes swimming in the evening. She dislikes dancing the aerobic but she likes exercises because of being her diet. .......................................................................................................................................... ................................................................................................................................................................... ..................................................................................................................................................................

ใบความรูท่ี 5.5 แบบทดสอบประจําหนวยท่ี 5 Choose the best answers by talking about like and dislike. 1. She.............. gardening in every weekend.

a) likes b) dislike c) does like d) doesn’t dislikes 2. I love ..................... everyday for my children. a) cook b) cooking c) cooks d) cooked 3. We ........................... dancing in every Friday at the nightclub. a) enjoy b) enjoys d) enjoying d) enjoyed 4. really / playing / they / like football. (Rearrange in the right order) a) They like really to play football. b) They really like to play football. c) They really like playing football. d) They really to play like football. 5. He ............... like swimming. a) don’t b)can’t c) doesn’t d) mustn’t 6. The old mother doesn’t ............ doing housework. a) like b) likes c) liked d) liking 7. The students hate ............. the next examination. a) do b) does c) doing d) did 8. We can’t.................... noise. a) stand b) stood c) standing d) stands 9. Do you like.............................? a) shop b) shopped c) shopping d) shops 10. I quite enjoy ........................ alone. a) be b) being c) to be d) have been

Answer Exercise 1 1. He likes playing tennis. 2. He dislikes riding a bike. 3. He likes running. 4. He dislikes swimming. 5. The old man likes taking a walk. 6. A man and a woman like dancing. 7. A woman likes driving. 8. They like going to the movies. 9. He likes shopping 10. She likes shopping. 11. She dislikes studying. 12. The man likes watching television. Exercise 2 / ผูสอนประเมินทักษะการอานตามความสามารถผูเรียน Exercise 3: / 1. going / 2 getting / 3. gardening / 4. going / 5 driving 6. spending / 7. watching / 8. cooking / 9. sleeping / 10. working Exercise 4: Restaurant language / Likes and dislikes/ 1. Could I have a table for four, please? 2. Which two of these wines do you recommend? 3. Could I have the bill, please? 4. I’m not on traveling keen on business. 5. I like giving quite presentations. 6. I can’t take to stand having work home. 7. I need some bread and some fish. 8. Do you need any fruit? 9. We need some hamburger meat. 10. Yes. I need some bananas. Exercise 5 แนวคําตอบตามความคิดเหน็ของผูเรียนแตตองถูกตองตามหลักไวยากรณตามตัวอยาง แบบทดสอบ 1. = a = She likes gardening in every weekend. 2. = b = I love cooking everyday for my children. 3. = a = We enjoy dancing in every Friday at the nightclub. 4. = c = They really like playing football.

5. = c = He doesn’t like swimming. 6. = a = The old mother doesn’t like doing housework. 7. = c = The students hate doing the next examination. 8. = a = We can’t stand noise. 9. = c = Do you like shopping? 10. = b = I quite enjoy being alone.

รายวิชา ภาษาอังกฤษเพื่อการสื่อสาร 2 Unit 6

Unit 6 Asking and Giving Opinion

Unit 6 Asking and Giving Opinion

สาระสําคัญ การทํางานในชีวิตประจําวันบางครั้งอาจ มีการถามหรือแสดงความคิดเห็นซึ่งมีทั้งการตอบรับหรืปฏิเสธิ์ จึงควรรูจักรูปประโยคตางๆ เหลานี้ เพื่อนําไปใชไดอยางถูกตอง จุดประสงคการเรียนรู ความรู

1. ฟงและเขาใจคําศัพท สํานวน ประโยคภาษาอังกฤษที่เกี่ยวกับการแสดงความคิดเหน็ 2. ฟงและพดูออกเสียงเกีย่วกบัการแสดงความคิดเห็นไดชัดเจนและเหมาะสมกับสถานการณ

ทักษะ 3. พูดและเลือกใชคําศพัท สํานวน ประโยคเกี่ยวกับการแสดงความคิดเห็นไดอยางถกูตอง 4. อานและระบุรายละเอียดของการแสดงความคิดเหน็ไดอยางเหมาะสม

คุณธรรม 5. สรางทัศนคติคานิยมเกี่ยวกับ การยอมรบั และการใชคําศัพท สํานวน ประโยคที่เกี่ยวกับการ

แสดงความคิดเห็นไดถูกตองตามสถานการณ เนื้อหาสาระ 1. Language Focus 1.1 Asking for Opinions

1.2 Giving for Opinions 1.3 Agreeing

1.3.1 Positive Agreement 1.3.2 Negative Agreement 1.4 Disagreeing 1.5 Asking for details 1.6 Giving your opinion 1.6.1 Neutrally 1.6.2 Strong point

1.7 Expressions for asking someone’s opinion 1.8 Wh - questions

2. Worksheet 2.1 ใบงานที่ 6.1: Warm up: Listening ฝกทักษะการฟงและจับคูประโยค ของ giving opinion, Agreement and Disagreement ใหสัมพนัธสอดคลองกับหัวขอทีก่ําหนด 2.2 ใบงานที่ 6.2: Reading ฝกทักษะการอานและจับใจความสําคัญเพื่อมาทําแบบฝกหัด 2.3 ใบงานที่ 6.3: Speaking ฝกทักษะการพูดประโยคของ Asking and giving opinion และจบัคูคําถามและคําตอบใหตรงกัน 2.4 ใบงานที่ 6.4: และ 6.5 Writing นําประโยค asking for information กับ giving opinion ที่จับคูตรงกันของใบงานที่ 6.2 มาเขียนเปนบทสนทนาใหสมบูรณและนํามาฝกทักษะการพูด 1. Language Focus

1.1 Asking for Opinions Asking for opinion is a skill in a good manner will mean that you get more information. This section will help students to think about how they can best put questions in a forum like a meeting, class discussion or question time type event. In general, questions need to be:

1. Put politely 2.Clear 3. About one issue 4. Encourage a response: - What do you think? - What’s your opinion? - What are your ideas? - Do you have any thoughts on that? - How do you feel about that?

1.2 Giving Opinions - I think we should get a new car. - I don’t think we need one. - I believe that smoking should be outlawed. - I don’t believe that it should be illegal. - In my opinion, Gone with the Breeze is the best movie ever made. - I feel that it’s the right thing to do. - I don’t feel that it’s such a good idea.

1.3 Agreeing 1.3.1 Positive Agreement

- If you agree with someone about an opinion, you can simply say “I agree” or “You’re right” - The word “so” and “too” is often used in expressions of positive agreement. - Note that “too” usually comes at the end of the sentence, and “so” at the beginning. - Remember to use” question order” with so.

1.3.2 Negative Agreement - For negative agreement, use neither in place of “so” and either in place of “too”. - These expressions can also be used in the second or third person: - Note that neither is used with “positive” verbs and either is used with “negative” verbs - When no verb is used in negative agreements, “neither” is more common:

- Neither do I. - I don’t either. - Me neither. 1.4 Disagreeing

- To express disagreement, you may simply say “I disagree” or “ I don’t think so” - You may also disagree by stating the opposite opinion, sometimes stressing a word for emphasis.

1.5 Giving your opinion - Neutrally : I think / feel / reckon/ guess and in my view/ opinion are used to make opinions and

statements sound less dogmatic. - I think she is lying. - I reckon / guess she just doesn’t love him. - In my view / opinion, it would be better to call the police. - I really feel she is making a mistake.

- Giving a strong opinion: - I’m absolutely convinced that......... - I’m sure that ................................... - I strongly believe that...................... - I have no doubt that .......................

1.6 Expressions for asking someone’s opinion - What do you think? - What’s your view? - How do you see the situation?

รูปภาพ 6.1 www.languageproject.co.uk/learn

1.7 Asking for Details: - WH – Questions can be used to ask for more information:

- What did you do ever the weekend? - Where did you go for Spring Break? - How was your trip? - When did you get back? - What kind of things did you see? - Who did you go with? - How many people were there? - Whose car did you drive?

- When asking for details about a particular item you are considering buying, you can say: - Could you give me some information about this computer? - Can you give more details about that CD player? - Could you tell me about this bookcase? - What can you tell me about these blenders?

Practice Example1 Example2 A: I think we need more time. A: I’m in a hurry. B: I agree. / You’re right. B: So am I. / I am too. / Me too Example3 Example4 A: I need to go to the bathroom. A: I’m not hungry. B: So do I. / I do too./ Me too. B: Neither am I. / I’m not either. Example5 Example6 A: I don’t have enough money anyway. A: They’re from Luxemburg. B: Neither do I. / I don’t either. B: So are we. / We are too. Example7 Example8 A: Todd doesn’t like spaghetti. A: I think John Doe would make a great President. B: Neither does Louise./Louise doesn’t either. B: I disagree. / I don’t think so.

1.8 Simple present Wh - questions 2.1 A: What sport do you like? B: I like basketball and skiing. 2.2 A: When does your team like practicing? B: I like practicing on Saturdays. 2.3 A: Who do you like playing basketball with? B: I like playing basketball with my team. 2.4 A: What time do you like practicing on Saturdays? B: We like practicing at ten o’clock in the morning.

2.5 A: Where do you like practicing? B: I like practicing at my college.

2.6 A: Do you like sports? What sports do you like? B: Yes. I like skiing, but I love swimming.

ใบความรูท่ี 6

Agreeing with feeling and opinions Disagreeing with feeling and opinions 1. So do I. 2. So am I. 3. Neither am I. 4. Neither do I. / Neither can I.

1. Oh, I don’t./Really? I........ 2. Gee, I’m not. 3. I am! 4. Well, I do. Oh, I don’t mind.

Ten Expressions of Asking for approval Ten Expressions of Giving Opinion 1. Do you think it’s all right to do it? 2. What do you think about (me doing that)? 3. Do you think / reckon I ought to (do it)? 4. What would you say if I (did) it? 5. Would you approve of (doing something)? 6. What is your attitude to the idea of.......? 7. Are you in favor of (me doing something)? 8. You are in favor of ........, aren’t you? 9. Do you think anyone would mind if I .............? 10. Do you think it would be really awful if I....?

1. I reckon ........................ 2. I’d say .......................... 3. Personally, I think ......... 4. What I reckon is ............. 5. If you ask me ................. 6. The way I see it .............. 7. As far as I’m concerned..... 8. If you don’t mind me saying ......... 9. I’m utterly convinced that ............. 10. In my humble opinion..................

Ten Expressions of Asking for Information Ten Expressions of Asking somebody’s opinion 1. Can you tell me ...............................? 2. Could you tell me ............................? 3. I’d like to know ............................... 4. Do you know ...................................? 5. (Got / Have you) any idea ...............? 6. Could anyone tell me ......................? 7. (Do / Would) you happen to know ..? 8. I don’t suppose you (would) know...? 9. I wonder if you could tell me ...........? 10. I wonder if someone could tell me..?

1. What do you think of ......................? 2. What do you think about ..................? 3. How do you feel (about..................)? 4. What do you reckon (about...............)? 5. What’s your opinion of ...................? 6. What do think about) that? 7. What are your views on ..................? 8. Where do you stand (on) ...................? 9. What would you say to ...... / if we ....? 10. Are you aware of ..............................?

ใบงานที่ 6.1 Warm up: Listening Exercise 1: Listen and match the opinion, agreement & disagreement with the topic accurately.

Answer Giving Opinion, Agreement, Disagreement Topic .............. .............. .............. .............. ............. .............. .............. .............. .............. ..............

1. Would you ever consider taking out private health insurance? 2. Many people in Britain eat too much sugar. How healthy are eating habits? 3. Have you ever bought something it was fashionable? 4. I ‘m sure that movie is very sad. 5. I strongly believe that there is a ghost truly. 6. I think my class is so happy for my students. 7. Are you for or against self – sufficiency economy? 8. I don’t believe in ghosts. 9. What do you think of this Royal speech of the king? 10. Are the resorts worth visiting?

A. Strong Opinion B. Disagreement C. Health D. Holiday E. Food F. Yes/No Questions G. W-H Questions H. Fashion I. Entertainment J. Agreement

ใบงานที่ 6.2 Reading and Speaking Exercise 2: Match the following Asking for information & giving opinion and speaking with your partner.

Answer Asking for Information Giving opinion .............. .............. .............. ............... ............... .............. ................ ................ ............... ..............

1. How was the movie? 2. How do you like living in Dallas? 3. What does he do? 4. Why not put it off until next week? 5. Thank you very much for all of your opinion. 6. Can you see what I mean? 7. You look upset. What’s the matter? 8. Do you mind my smoking? 9. Will we be in time for the meeting? 10. Do you think he will come?

A. Never heard of him. B. I see. C. No, go head. D. I’m afraid not. E. It’s a nice city. I like it here. F. No, I don’t think he will. G. Don’t mention it. H. Really boring. I. I’m running out of money. J. What a good idea!

ใบงานที่ 6.3 Reading Exercise 3: Read the four paragraphs and write “True” or “False” in each sentence.

รูปภาพที ่ 6.2 New Interchange INTRO หนา 71 ............ 1. To celebrate her birthday, Elena is going to pull on her friends ‘ears. ............ 2. Yan – Ching is going to cook some noodles on her birthday. ............ 3. On his birthday, Mr. Aoki is going to buy something red. ............ 4. Phillippe’s friends are going to take him out to dinner on his birthday. ............ 5. Elena is thirty – first years on her birthday. ............ 6. Phillippe has a family ceremony on his birthday. ............ 7. In Mrs Aoki’s opinion, the color red is a new life. ............. 8. In Yan – Ching Shi’s opinion, she is in lucky by getting the money. ............. 9. Phillippe lives in England. .............10. Mr. and Mrs. Aoki live in Japan.

ใบงานที่ 6.4 Writing and Speaking Exercise 4: Rearrange the following asking and giving opinion from exercise 2, write the complete conversation and practice speaking with your partner. A: .................................................................................. B: .................................................................................. A: .................................................................................. B: .................................................................................. A: .................................................................................. B:................................................................................... A: ................................................................................... B: ..................................................................................... A: .................................................................................... B: .................................................................................... Asking Information and Giving Opinion Exercise 5: Choose the best answer by underlining the word in the blanket.

1. Neither I nor my sister (are / is) going to the dance. 2. Gold as well as silver (have fallen / are falling) in price, he said. 3. The knowledge about science (are much / is much) helpful. 4. One of girls (are / is) ill. 5. Fifty miles on the quiet road (seem / seeming) like two hundred. 6. I think I will (watch / watches) TV tonight. 7. I can’t (open / opens) the door. 8. What do you think they are (do / doing) now? 9. (Take / Taking) an umbrella with you. It looks like rain. 10. I’ve got a sore throat and a bit of a headache. I think I may be going (on / down) with something.

แบบทดสอบประจําหนวยท่ี 6 Choose the best answer 1. I’m afraid I have to go out for a minute. Please, .................. yourself at home. a) make b) treat c) be d) behave 2. I think that our products are better quality................. theirs. a) this b) these c) than d) more 3. How do you feel............................ the new design? a) in b) on c) by d) about 4. I don’t feel that our ads ................... reaching customers. a) is b) am c) are d) be 5. What .......... you think about the new marketing strategy? a) do b) did c) done d) does 6. I agree........... you about selling direct to the customer. a) with b) or c) that d) under 7. Sandra feels the price is too high and I think .......... too. a) to b) of c) so d) as 8. In your opinion, how was the movie? The answer of this opinion is................................................ a) No, go ahead b) No, I don’t think he will c) Never heard of him d) Really boring. 9. you / this / see / how / situation /? / do / (Rearrange the sentence into asking opinion) a) Do you this see how situation? b) How you see do this situation? c) How do you see this situation? d) Do you how see this situation? 10. You look starving. Here, ................... a biscuit. a) enjoy b) eat c) take d) have

Answer Exercise 1 1. C 2 . E 3 .H 4 . I 5. A 6. J 7. F 8. B 9 . G 10 . D Exercise 2 1. False 2 False 3. True 4. True 5. False 6. False 7. True 8. True 9. False 10. True Exercise 3 1. H 2. E 3. A 4. J 5. G 6. I 7. C 8. D 9. B 10. F Exercise 4 A: How was the movie? B: Really boring. A: Can you see what I mean? B: I see. A: Will we be in time for the meeting? B: I’m afraid not. A: Do you mind smoking? B: No, go ahead. A: Do you think he will come? B: No, I don’t think he will. A: Why not put it off until next week? B: What a good idea! A: You look upset. What’s the matter? B: I’m running out of money. A: How do you like living in Dallas? B: It’s nice city. I like it here. A: What does he do? B: Never heard of him.

Exercise 5 1. is 2. have fallen 3. is much 4. is 5. seeming 6. watch 7. open 8. doing 9. Take 10. down เฉลบแบบทดสอบประจําหนวยท่ี 6 1. a 2. c 3. d 4. c 5. a 6. a 7. c 8. d 9. c 10. d

รายวิชา ภาษาอังกฤษเพื่อการสื่อสาร 2 Unit 9

Unit 7 Holidays

Unit 7 Holiday

สาระสําคัญ การสนทนาในแตละครั้งอาจมีการถามเกี่ยวกับการพกัผอนหรือกิจกรรมที่ทําในวนัหยุด ซ่ึงก็มีรูปแบบ

และสํานวนในการถามตางๆ กันไป ในแตละเทศกาล จงึควรไดรับความรูและขอมูลเพื่อเผยแพรใหกับประเทศของเรา และสามารถนําไปสูการสื่อสารไดอยางถูกตอง จุดประสงคการเรียนรู ความรู 1. อานออกเสียงและอธิบายความหมาย ใชคําศัพทเกี่ยวกับสถานที่ทองเที่ยวและกจิกรรมในวนัหยุด 2. อานและจับใจความจากเนือ้เร่ืองเกี่ยวกับสถานที่ที่จะไปในวนัหยุด ทักษะ 3. อานเนื้อเร่ืองเกี่ยวกับสถานที่ที่จะไปในวันหยุดและตอบคําถามที่อานไดถูกตอง 4. สืบคนหาขอมูลแหลงทองเที่ยวและกจิกรรมที่จะทําใหวันหยดุพักผอนไดด ีคุณธรรม 5. สงเสริมการรักการอานที่นาํสูลักษณะนิสัยการวางแผนในการหาขอมลูเกี่ยวกับกจิกรรที่ทําในวันหยุด เนื้อหาสาระ 1. Language Focus 1.1 Key Vocabularies 1.2 Expressions 1.2.1 Asking information about holidays 1.2.2 Giving information about holidays 1.2.3 Asking about holidays 1.2.4 Talk about future plans 1.3 to be going to

1.3.1 The future with be going to: 1.3.2 Wh – questions with be going to 1.3.3 Talk about future plans

2. Worksheet 2.1 ใบงานที่ 7.1 Warm up: ฝกออกเสียงคาํศัพทและวิเคราะหหาความสัมพันธเกี่ยวกบั Holiday

2.2 ใบงานที่ 7.2 Speaking: ฝกการอานออกเสียงการพดูเกี่ยวกับวันที่และเดือนประจาํป 2.3 ใบงานที่ 7.3 Speaking: ฝกการอานบทสนทนาเกีย่วกับกิจกรรมที่ทําในวนัหยุด

2.4 ใบงานที่ 7.4 Reading: ฝกการอานเนื้อเร่ืองและสรุปใจความสําคัญเพื่อการตอบคําถามที่ถูกตอง 2.5ใบงานที่ 7. 5 Writing: ฝกการเขียนคําศัพทเกี่ยวกับกจิกรรมที่ทําในวันหยุดในรูปแบบของบท สนทนา 1. Language Focus 1.1 Key Vocabularies 1.1.1 Vocabularies:

- Holiday (British) - Vacation (American) - Public holiday - Celebration - Religious holiday - Fair - Festival - Feast - Carnival

1.1.2 Activities: - making merit - sightseeing - buying souvenirs - exploring - paying respect to - trying the local food - going to bars and discos - getting fresh air 1.2 Expression: 1.2.1 Asking information about holidays - When is Ramadan? - How long does the celebration last? - Where does the dancing performance take place? - Why do people celebrate Loy Kratong festival? 1.2.2 Giving information about holidays - St. Patrick’s Day is on March 17. - The celebration lasts 3 – 5 days.

- People usually go to church. - It’s day to celebrate / commemorate / preserve. 1.2.3 Asking about holidays

Asking Positive answers Negative Answers - How was your holiday / trip / vacation? -How did you enjoy your holiday / trip / vacation? -What do you think of the celebration there?

- It was wonderful / excellent / marvelous / splendid. - I think going to that festival is wonderful / great.

- It was terrible. - I didn’t (quite) enjoy it.

1.2.4 Talk about future plans Question Answer

- Are you going to do anything exciting this next holiday? - What are your plans? - What are you going to do? - Who’s going to be there? - How are you going to get there?

- Yes, I am. I’m going to……………….. - I’m going to………………………….. - I’m going to go……………………… - ……….is / are going to be there. - We’re going to……………………..

Practice: Conversation 1 A: Hi, James. How was your holiday? B: Hello, Vicky. Oh I’m so glad to be home! A: Why? What happened: B: Everything! It was the worst holiday ever. The flight was delayed by bad weather. A: Did you enjoy shopping there? B: Not at all.

Conversation 2 A: Do you have any kind of celebration on St. Valentine’s Day in Thailand? B: Well, I think we focus more on commercialism. People especially lovers celebrate by giving roses to each other. A: That’s interesting. In the past, young people met to choose the name of a person of

the opposite sex on St. Valentine’s Day. Thus, each gentleman would choose a lady and become her valentine.

Conversation 3 A: What are you going to go for the holiday? B: We are going to go to my parents’ house. A: Who’s going to be there? B: My whole family is going to be there. A: How are you going to get there? B: We’re going to drive. 1.3. to be going to

1.3.1 The future with be going to: Question: Are you going to do anything exciting this weekend? Answer: Yes, I’m going to celebrate my birthday. Answer: No, I’m not. I’m not going to stay home.

1.3.2 Wh – questions with be going to Question: Where are you going to go for the holiday? Answer: We’re going to go to my parent’s house. Answer: No, we’re not. We’re going to stay our home.

1.3.3 Talk about future plans Questions: What are your plans? Any plans? Answer: I’m going to ............................... Question: Who’s going to be there? Answer: - .......... is / are going to be there. Question: When are you going to go? Answer: We’re going to ........................ Question: How are you going to get there? Answer: We’re going to .........................

ใบความรูท่ี 7 Wrap up 1. International Holidays 1.1 April Fools’s Day is a fun observance. It is simply a fun little holiday, but a holiday on which one must remain forever vigilant, for he may be the next April fool. 1.2 Easter is a time of springtime festivals. The celebrations of Easter have many customs and legends that are pagan in origin and have nothing to do with Christianity. 1.3 Halloween is one of the oldest holidays with origins going back thousands of years on October 31. 1.4 Thanksgiving Day is a day of General Thanksgiving to God for the harvest which Canada has been blessed. 1.5 Christmas Day is used the Christmas card in “Merry Christmas and a Happy New Year to you” is the first message on December 25th. 1.6 Valentine’s Day is February 14th of every year. People think about love and friendships because they know that love can’t be beaten and to stand in the way of love. 1.7 New Year’s Day is on January 1st of every year. People will be able to celebrate with their relatives at home or spend a long holiday in a place they like. 1.8 Chinese New Year usually arrives between January 19th and February 21st. It means the date for the Chinese New Year Day. 2. Thai Public Holidays 2.1 Children Day 2.2 Teacher Day 2.3 Makha Bhucha Day 2.4 Chakri Day 2.5 Songkran Day 2.6. National Labor Day 2.7 Coronation Day 2.8 The Royal Ploughing Ceremony Day 2.9 Visakha Puja Day 2.10 Asalha Bhucha Day 2.11 Kho Phansa Day 2.12 H.M. The Queen’s Birthday 2.13 H.M. The King’s Birthday 2.14 Loy Krathong 2.15 Thai Constitutions Day 2.16 Piyamaharaj Day

ใบงานที่ 7.1

Warm up Exercise 1: Read the vocabulary and find the “odd one out”

No. Vocabulary Vocabulary Vocabulary Answer 1 full - board half - board all - inclusive ............................. 2 amenities facilities services ............................. 3 operators competitors rivals ............................. 4 hire rental purchase ............................. 5 travel agents timeshare developments tourist complexes ............................. 6 an itinerary a journey a route ............................. 7 bed & breakfast a hotel a guest house ............................. 8 a youth hostel a tent a caravan ............................. 9 a tour an excursion a sightseeing up ............................. 10 Holiday Vacation Vocation .............................

ใบงานที่ 7.2 Listening Exercise 2: Listen to the song , fill in the blanks and practice singing this song. November full (1)………………………. shines, Loy Krathong, Loy Krathong, and the water\'s high in the (2)……………………… and local klong, Loy Loy Krathong, Loy Loy Krathong, Loy Krathong is (3)…………………. and everybody\'s full of cheer, We\'re together at the klong, Each one with his (4)……………………………… As we push away we pray, We can see a (5)………………………….day.

Exercise 3: Listen to the conversation and choose the given words to fill in the blank. visit show long of course what temple holiday when shoes who A: Hello! Are you going to stay (1)……………………………..in Bangkok? B: Yes, about 2 weeks. A: (2)………………………..are you going to do this long holiday? B: I’m going to do some sight seeing. A: Where do you plan to (3)…………………………… B: I plan to visit some beautiful (4)……………………………in Bangkok, such as Grand Palace and Temple of Emerald Buddha. A: Great. It must be very nice. B: (5)……………………………….I hope it’s going to be interesting. A: (6)……………………………..you are at the temple, you have to take your(7)…………………. Off before entering. And Thai people (8)………………………………the respect to the Buddha Image by “Wai”. B: Yes, I have learned about that. A: (9)…………………………are you going with? B: I’ll go with my sister. A: I hope you enjoying (10)……………………………… B: Thank you very much.

ใบงานที่ 7.3 Speaking Exercise 4: Listen and practice the conversation in pairs. Conversation 1: A: Are you going to do anything exciting this weekend? B: Well, I’m going to celebrate my birthday. A: Fabulous! When is your birthday exactly? B: It’s August ninth – Sunday. A: So what are your plans? B: Well, my friend Joe is going to take me to a restaurant. A: Nice! Is she going to order a cake? B: Yeah, and the waiters are probably going to sing “Happy Birthday” to me. It’s so embarrassing! Conversation 2 A: So, Dennis. What are you going to do for Loy Krathong. B: I’m going to float a Krathong with my family. What about you? Any plans? A: Yeah, I’m going to go to some friend’s house. We’re going to make the Krathongs. B: We always have banana leaves, flowers, candle, and joss sticks in it. A: I know. Every year, my friends and I make the different Krathongs. B: Sounds like fun. Well, have a happy Loy Krathong. A: Thanks. You too.

ใบงานที่ 7.4 Reading Exercise 5: Reading the text and the short answer the questions.

Holidays In Thailand, Tourist Information Centers can arrange shorts stays in Hotels, Guest Houses and Bed & Breakfast establishments. Bookings can be made either by phone or in person on arrival in large towns or cities. Clients can usually specify the part of town where they want to be based, though this may limit their choice of price – range. It is helpful to state whether you are looking for economical, medium – priced or luxury items. A small deposit is usually required which is returnable if you inspect the room and decide you do not want it. If you take the room, the deposit is deducted form the bill. For longer holidays in other countries, it is usual to go to a travel agency. You can collect brochures from rival companies which offer packages in the countries and resorts which you are interested in. Many people go home and study the information. The booking forms on the back of the brochures contain the information which the travel agent needs to know, should you wish to go ahead. You will need to decide the number in your party, the resort, the departure date, the duration, the type of accommodation, the eating arrangements, travel or transportations, car hire, equipment hire and the method of payment. 1. What do you think of package holidays? ............................................................................................. 2. Are the resorts useful for holidays? ............................................................................................. 3. How do you get the information of package holidays? ....................................................................... 4. How can you be booking of package holiday? .................................................................................... 5. Where can you see the booking forms? .............................................................................................. 6. Where do many people study the brochures? ..................................................................................... 7. What can you get from studying the brochures?................................................................................. 8. How do you think about the planning of the holidays?....................................................................... 9. Do you prefer to book holidays through travel agencies or to make your own arrangements? ............................................................................................................................................................. 10. What holiday are you thinking about in Thailand?...........................................................................

ใบงานที่ 7.5 Writing Exercise 6: Write ten questions about doing of these activities this weekend by using some activities from these answers. 1. A: Are you going to ...................................... this weekend? B: Yes, I am. I’m going to see the Harry Porter movie. 2. A: …………………………………………………………………………………………………? B: Yes, I am. I’m going to cook the fried rice at home. 3. A: …………………………………………………………………………………………………? B: Yes, I am. I’m going to eat at a restaurant. 4 A: …………………………………………………………………………………………………..? B: Yes, I am. I’m going to walk for my good exercise. 5. A: ………………………………………………………………………………………………….? B: Yes, I am. I’m going to watch T.V. about “Talk Show Program.” 6. A: What are you going to ..................................... this weekend? B: I’ m going to read some books. 7. A: …………………………………………………………………………………………………..? B: I’m going to do my homework at home. 8. A: Where are you going to …………………………this weedkend? B: I am going to swim at the Health Club near my house. 9. A: What are you going to do on Loy Krathong Day? B: …………………………………………………………………………………………………….. 10. A: What are you going to do on Valentine’s day? B: …………………………………………………………………………………………….............

แบบทดสอบประจําหนวยท่ี 7 Choose the best answers. 1. When is the King’s birthday? a) 5 December b) 5 November c)10 December d) 5 October 2. Why do people float the Krathong on Lay Krathong Day? a) for fun b) to offer thank to the Goddess of water c) to pay respect to the Buddha d) to forgive the elders 3. When is Songkran Day? a) 13 April b) 14 April c) 15 April d) 13-15 April 4. When is the Children day? a) the first Saturday of January b) the second Saturday of January c) the first Sunday of January d) the second Sunday of January 5. When is the Piyamaharaj Day? a) 23 September b)) 23 October c) 23 November d) 23 December 6. When is the Labor Day? a) 1 April b) 1 May c) 1 June d) 1 July 7. What are people doing before Chinese New Year Day? a) to travel. b) to give cash gifts to their employees. c) to pay respect to the elders. d) to cleaning their house to sweep away bad luck. 8. What is the Christmas Day celebration for? a) the King’s birthday b) the Queen’s birthday c) Jesus’ birthday d) Maria’s birthday 9. When is Valentine’s Day? a) 14 January b) 14 February c) 14 March d) 14 April 10. When is the Thai traditional new year? a) New Year Day b) Christmas Day c) Songkran Day d)Loy Krathong Day

Answer Exercise1 1. all – inclusive 2. amenities 3. competitors 4. purchase 5. timeshare developments 6. an itinerary 7. bed & breakfast 8. a youth hostel 9. an excursion 10. Vocation Exercise 2 1. moon 2. river 3. here 4. Krathong 5. better Exercise 3 1. long 2. what 3. visit 4. temple 5. of course 6. when 7. shoes 8. show 9. who 10. holiday Exercise 4: ประเมินผลการพูดตามบทสนทนา Exercise 5: แนวคําตอบ 1. It is very cheap. 2. Yes. 3. from the brochures 4. by phone or in person 5. on the back of the brochures 6. at home 7. the method of payment 8. It is good to plan. 9. เลือกตอบระหวาง through agencies หรือ make my own arrangements (แตหามตอบทั้ง 2 ชนิด) 10. ตอบตามความชอบของผูเรียน Exercise 6: แนวคําตอบ 1. see a movie 2. Are you going to cook this weekend? 3. Are you going to eat outside this weekend? 4. Are you going to exercise this weekend? 5. Are you going to watch TV this weekend? 6. do 7. What are you going to do this weekend? 8. swim 9. I am going to float Krathong in the river. 10. I am going to give a red rose to my friend. เฉลยแบบทดสอบประจําหนวยเรียนท่ี 7 1. C 2. B 3. A 4. B 5. B 6. A 7. D 8. D 9. B 10.

รายวิชา ภาษาอังกฤษเพื่อการสื่อสาร 2 Unit 8

Unit 8 Offering, Accepting

and Refusal

Unit 2 Offering, Accepting and Refusal

สาระสําคัญ การทํางานรวมกันในสังคมมีกิจกรรมทางสังคมและธุรกิจ ที่ตองพึ่งพาอาศัยใหความชวยเหลืระหวางกันจึงตองมกีารขอรอง การขออนุญาต การเสนอใหความชวยเหลือเพื่อชวยใหการทํางานนั้นสําเร็จลุลวงไปดวยดี จึงควรศึกษาศัพท สํานวน ประโยคเพื่อนําไปใชใหเหมาะสมกบัสถานการณและคํานึงถึงความสุภาพในการพูดเชนกัน จุดประสงคการเรียนรู ความรู

1. ฟงและเขาใจคําศัพท สํานวน ประโยคภาษาองักฤษเกีย่วกับการใหความชวยเหลือ การขอรอง การขออนุญาต การตอบรับ และการปฏิเสธ

2. เลือกใชคําศพัท สํานวน ประโยคภาษาองักฤษเกีย่วกับการใหความชวยเหลือ การขอรอง การขอ อนุญาต การตอบรับ และการปฏิเสธในการสนทนาโตตอบที่ถูกตอง ทักษะ 3. ปฏิบัติการพูด การอาน และการเขยีน คําศัพท สํานวน ประโยคภาษาอังกฤษเกี่ยวกับการใหความชวยเหลือ การขอรอง การขออนุญาต การตอบรับ และ การปฏิเสธที่ถูกตอง

4. วิเคราะหการใชคําศัพท สํานวน ประโยคภาษาอังกฤษที่เกีย่วกับการตอบรับและการปฏิเสธที่ สุภาพสําหรับใหความชวยเหลือ การขอรอง การขออนุญาต การตอบรับ และการปฏิเสธอยางเหมาะสมตามสถานการณ คุณธรรม 5. สรางลักษณะนิสัยของ ความสุภาพ ในการพูดเกี่ยวกับการใหความชวยเหลือกัน การตอบรับ และการปฏิเสธอยางสุภาพในชีวิตประจําวนัและการทํางานดวยกจิกรรมทางสังคม เนื้อหาสาระ 1. Language Focus 1.1 Asking for things 1.2 Asking people to do things (to help) 1.3 Asking for permission - Conversation 1 - Conversation 2

1.4 Can you………….? / Could you……….? 2. Worksheet

2.1 ใบงานที่ 8.1 Warm up: ทําแบบฝกหัดเกี่ยวกับคําศัพท สํานวน ประโยค 2.2 ใบงานที่ 8.2 Listening and speaking: ฟงและฝกพดูเกี่ยวกับการเสนอขอความชวยเหลือ การตอบ

รับ หรือการปฏิเสธการเสนอใหความชวยเหลือ และจับคูใหตรงกับภาพที่กําหนดให 2.3 ใบงานที่ 8.3 Speaking ฟงบทสนทนา แลวเลือกเติมคําตอบที่ถูกตอง 2.4 ใบงานที่ 8.4 Reading: อานบทสนทนาและตอบคําถามเกี่ยวกับบทสนทนา 2.5 ใบงานที่ 8.5 Writing: จับคูสํานวนการขอความชวยเหลือและการตอบรับหรือปฏิเสธิ์

1. Language Focus 1.1 Asking for things

Making a request Agreeing Refusing - I would like a table for two, please. - Can I pay by credit card? - May I have the wine list? - Could I have a piece of chocolate cake?

- Certainly. Is this table okay? - Yes, that’s fine. - Yes, of course./ Right away. - Sure (informal) / OK.(informal)

- I’m sorry. All are booked. - I’m sorry but we don’t get credit card here. - Sorry. We don’t serve wine here. - I’m afraid we’re out of it tonight.

1.2 Asking people to do things (to help)

Making a request Agreeing Refusing - Can you send this email message for me please? - Will you make copies of these handouts, please? - Could you prepare this report for the meeting? - Would you send this fax message to ABC Company? - Would you mind working overtime?

- Yes, certainly. - Yes, of course. - Yes, of course. I’ll be happy to. - Yes, that’s no problem. - Oh, sure. I’ll be glad to. - OK. (informal) - Sure. (informal) - No, not at all.

- I’m afraid, I can’t. - Sorry, the machine is out of order. - I’m sorry. I’m preparing for the presentation. - I’d rather not. - Sorry. I’m busy. - I’m sorry. I have an appointment

1.3 Asking for Permission / Agreement

Asking for permission Agreeing Refusing - Can I borrow your talking dictionary, please? - Could I borrow your mobile phone, please? - May I please borrow your note book, please?

- Yes, you can. - Yes, of course. - Yes, certainly. -Certainly. - Sure. (informal) - OK. (informal) - Yes, you may but it must be quick.(partly agree)

- I’m afraid you can’t the battery is low. - I’m sorry but I need to use it myself. - Sorry. I’m using it right now.

- Would you mind if I borrowed your thumb drive?

- No, not at all. - No, of course not.

- I’m sorry but I’m using it.

Practice Conversation 1 A: Yes, What can I do for you? B: Could I have a cup of coffee please? A: Of course. Right away. Could I get you anything else? B: Would you bring some cream too? A: Certainly. B: Thanks. Conversation 2 A: May I borrow your calculator for a minute. I forgot to bring mine to class today. B: I’m sorry, but I’m using it for my math exercises.

1.4 Can you……………….? / Could you…………..? - We use Can you……………….? / Could you…………..? When we ask people to do things. A: Can (Could) you open the door, please? B: Yes, sure

- We use Can I…………….? When we ask if it is okay to do something. A: Tom, can I take your umbrella? B: Yes, of course. - We use Can I have………….? To ask for something. A: Can I have these postcards, please? B: Here you are.

Important 1. Request forms are used with formal and informal forms 2. If you refuse a request, it’s polite to give a reason. 3. The word “mind” means “object to” so to agree to request you have to say “No.” or “No,

not at all.” 4. “Can” is not stressed in polite questions as in requests and permissions, so it is pronounced

/k∋n/. 5. “Can” is stressed in a short answers in both positive and negative. It is pronounced /k&n/

in a positive and /k&nt/ in a negative answer. 6. Can I is less formal than could I or may I. 7. You can use may or can to give permission, and can’t or may not to refuse.

ใบความรูท่ี 8 Expression Requests, Offers, and Permission

Expression Use Example

Asking for help (can, could, will, would)

- เราใช can you, could you, will you หรือ would you เพื่อถามสุภาพ คําถามนั้นเปนการขอความชวยเหลือหรือความรวมมือจากบุคคล คําวา “Can” ใชในความหมายที่เปนทางการนอยกวาคําอืน่

- Can you wait a moment, please? - Could you open the door, please? - Panya, will you do me a fever? - Would you please tell me how to send the email?

Asking for things (can, could, may)

เราใช “Can I have..? หรือ “Could I have...? เพื่อขอสิ่งของที่เราตองการ อาจใช May I have....? ก็ไดแตโดยทัว่ไปไมนิยมใช

- Can I have the wine list? (In a restaurant) - Can I have these shoes in blue, please? (In a shoes shop) - Could I have these copies of handouts? (In an office)

Offering to do things (can ,will)

ในการเสนอทีจ่ะทําสิ่งหนึ่งสิ่งใดใหบุคคลเราอาจใช Can I ......... หรือ I’ll.................

- A: Can I get you a cup of coffee? - B: Yes, that would be nice. - Your box is heavy. I’ll help you.

Offering and inviting (would)

เราใช Would you like.....? เพื่อเสนอสิ่งใดส่ิงหนึ่งหรือในการเชิญ ไมใช “Do you like.....?

- Would you like some more ice-cream? - Would you like to come to dinner with us?

Saying what you want (would)

I’ like ...... เปนสํานวนที่สุภาพในการพูดวา เราตองการสิ่งใด

- I’d like to try this dress on. (in a shop) - I’d like a room for tonight. (at a hotel)

ใบงานที่ 8.1 Warm up: Word power Exercise 1: Match the verbs on the left with the corresponding nouns on the right

Answer Verbs Nouns .............. 1. answer a) email message .............. 2. deposit b) phone message .............. 3. turn on / turn off c) jeans .............. 4. leave d) request ................ 5. take e) appointment ................ 6. fill out f) phone call ............... 7. make h) money ................ 8. send i) form ................ 9. try on j) light ............... 10 refuse k) order

ใบงานที่ 8.2 Listening and Speaking Exercise 2: Practice saying these sentences and analyze the situation from the picture below. A B C

D E

Situation 1: ....................................................... A: Can I have a piece of cake? B: Yes, you can. Situation 2: ....................................................... A: Can I leave the class early today? B: No, you can’t. Situation 3: ....................................................... Can you please type this letter for me? Situation 4: ........................................................ Can you please book a room for 2 nights? Situation 5: ........................................................ Can you please make copies of these handouts?

ใบงานที่ 8.3

Speaking : Pair work Exercise 3: Fill the telephone conversation with the given word. Then practice speaking them. make leave take speak talk help have ask tell connect 1. A: Good morning. RS Entertainment. B: Hello. Is Mr. McIntosh there? A: Yes, he is. B: Can I (1)........................to him? 2. A: Hello. Success Media. B: Hello. May I(2) .............. to Ms. White? A: I’m afraid she’s not in the office at the moment. Can I (3)……...........a message? B: Yes. Please (4)………………….. her Peter called 3. A: Hello. Dr. Bight’s office. B: Can I speak to Dr. Bright? A: May I (5)………………… who’s calling please? B: Sirinya Napaporn. A: Just a moment, Ms. Nappaporn. I’ll (6)…………………… you. 4. A: Hello. May I (7).................. you? B: Hello. Is Mr. Oichi in? A: Sorry. He’s not here right now. B: Oh. Can I (8).................................. a message? 5. A: Good afternoon. Marketing Department. B: Can I(9)………………...... an appointment with Ms. Young, please? A: Is Monday at 2 o’clock all right? B: Yes, that’s fine. A: May I (10)……………........... your name and phone number, please?

ใบงานที่ 8.4

Reading Exercise 4: Read the conversation and answer the questions

The first date Henry: Can I have permission to leave the office early today, sir? Boss: Well, it would be rather convenience. Is it something important? Henry: Yes, it’s important. Boss: Do you mind if I ask what is it? Henry: I want to get a good seat for dinner tonight. It is my first date. Boss: I am sorry but I can’t allow you to leave early. It wouldn’t be fair. Ten minutes later……………………. Henry: Excuse me, sir. I wonder if I could leave early because I’ve been suddenly ill. Boss: .???????? Answer questions.

1. What permission does Henry ask? …………………………………………………………………………………………………… 2. Why does Henry want to leave the office early? …………………………………………………………………………………………………… 3. What question does the boss ask? ……………………………………………………………………………………………………. 4. Why does Henry come to see the boss again? ……………………………………………………………………………………………………. 5. What do you think the boss will say to Henry at the end of conversation? ……………………………………………………………………………………………………..

ใบงานที่ 8.5

Writing Exercise 5: Match the requests and offers with the appropriate responses.

Requests / Offers ……………. 1. Could you work on Saturday? ……………. 2. Would you make some more coffee? ……………. 3. Could you make a copy of this report? ……………. 4. Could you call Ms. Taylor about dinner? ……………. 5. Could you meet Mr. Wason at the airport? ……………. 6. Would you type this letter? It’s urgent. ……………. 7. Would you like a piece of cake? ……………. 8. Would you like a cup of coffee? ………….... 9. Would you like another sandwich? ……………. 10. Would you like some tea?

Responses a) Sure. Black with sugar. b) Of course. What‘s her phone number? c) I’m busy right now. Can I type it tomorrow? d) I’d rather not. It’s my birthday. e) No problem. What time is he arriving? f) Sure no problem. g) No, thank you. I don’t drink tea. h) No, thanks. I’m on a diet. i) Yes, please. Coffee sounds good. j) No, thank you. That‘s enough for me.

แบบทดสอบประจําหนวยท่ี 2 Complete the conversations. Use a polite request with will you, would you, could you or can you with the expressions in the given expression.

a. help me with my term project b. prepare the equipment c. pass me the salt d. sign your name on the deposit slip e. send the order through the website f. turn the lights on g. turn the volume on h. pick him up all the airport i. tell me where the nearest bank is

j. answer the phone for me 1. A: It’s getting dark. B: Would you (1) ...................................? A: Thank you. B: You’re welcome. 2. A: The phone is ringing but my hands are dirty. B: Will you (2) ................................................? A: Yes, of course. B: That’s okay. 3. A: Mr. B, our company’s guest is coming from Chicago tomorrow. B: Could you (3) ............................................................................? A: Certainly, sir. B: Thanks. A: I’m glad to do it. 4. A: Excuse me. I’m a stranger here. B: Can you (4) .............................................? A: Well, thanks any way. I’ll ask someone else.

5. A: I’d like to deposit some money in my saving account. B: Will you (5) ................................................................? A: Thanks. B: Thank you. Here’s your money. 6. A: I’d like to order the book through the Internet. B: Would you (6) ...............................................................? A: Anything else. B: That’s no problem. 7. A: I’d like to listen to the music on the television, but I can’t hear it. B: Could you (7) ..............................................................? A: That’s OK. So I’ll go out to a karaoke. 8. A: I’m going to present our new products to the dealers tomorrow. B: Can you (8) .....................................................................? A: What’s problem? B: I’m sorry. I can’t help you. 9. A: The stake is delicious. B: It’s good. Can you (9) .....................................................? A: Thank you. 10. A: I want you to help me with my term project. B: Can you (10) .................................................? A: Sure.

Can you see how to pour the oil? Will you understand this? Would you interview tomorrow?

Answer Exercice 1 1. F 2. H 3. J 4. B 5. K 6. I 7. E 8. A 9. C 10. D Exercise 2 1. A 2. C 3. B 4. E 5. D Exercise 3: 1. talk 2. speak 3. take 4. tell 5. ask 6. connect 7. help 8. leave 9. make 10. have Exercise 4 : 1. To leave the office early. 2. To get a good seat for dinner tonight. 3. Do you mind if I ask what is it? 4. He’s been suddenly ill. 5. No, I can’t allow. Exercise 5: 1. D 2. I 3. F 4. B 5. E 6. C 7. H 8. A 9. J 10. G เฉลยแบบทดสอบประจําหนวยท่ี 2 1. A 2. B 3. C 4 D 5. B 6. C 7. D 8.A 9. C 10.D

รายวิชา ภาษาอังกฤษเพื่อการสื่อสาร 2 Unit 9

Unit 9 Giving Direction

Unit 9 Giving Direction

สาระสําคัญ ในชีวิตประจําวันเราอาจจําเปนตองเดินทางไปสถานที่ที่เราไมคุนเคย หรือไมรูจักทาง ในบางโอกาส เราอาจพบบุคคลที่เปนชาวตางชาตมิาสอบถามเกี่ยวกับเสนทางไปยังสถานทีต่างๆ ผูเรียนจึงควรเรียนรูวิธีการพดูและการเขียน อธิบายทิศทางจากสถานทีห่นึ่งไปอีกสถานที่หนึ่ง จุดประสงคการเรียนรู ความรู

1. อานออกเสียงความคําศัพท สํานวน และ Imperative Verb ที่ใชในการอธิบายเสนทางหรือวิธีการเดนิทาง 2. เลือกใชศพัท สํานวนและ Imperative Verb ที่ใชในการอธิบายเสนทางหรือวิธีการเดินทางไดถูกตองเหมาะสม ทักษะ

3. พูดสนทนาสอบถามและตอบขอสอบถามเกี่ยวกับเสนทาง วิธีการเดินทางไดถูกตอง 4. วิเคราะหใชศัพท สํานวน ไวยากรณและ Imperative Verb เกี่ยวกับเสนทางหรือวธีิการเดินทางไดอยางเหมาะสม คุณธรรม 5. เสริมสรางลักษณะนิสัยของการอธิบายเสนทาง หรือวิธีเดินทางเปนภาษาอังกฤษดวย ความรับผิดชอบ ในการที่จะตองอธิบายใหเขาใจและถกูตองตามกาลเทศะ เนื้อหาสาระ 1. Language Focus : Expression and sentences in giving direction 1.1 Asking the way and giving direction 1.2 Asking for clarification and useful expression 1.3. Asking , Accepting and Refusing

1.4 Imperatives verb 2. Worksheet 2.1 ใบงานที่ 9.1 Warm up ศึกษาคําศัพทเกี่ยวกับการบอกทิศทางจากภาพที่กําหนด แลวหาคําศัพทที่เกี่ยวกับการบอกทศิทาง 10 คํา

2.2 ใบงานที่ 9.2 Listening ดแูผนที่ แลวฟงขอความจากเทป แลวพิจารณาวาถูกตองตามแผนที่หรือไม

2.3 ใบงานที่ 9.3 Speaking ดูแผนที่แลวจับคูฝกพูดบทสนทนาสอบถามและตอบเกีย่วกับเสนทางไปยังสถานที่ตางๆ ตามแผนที่ที่กาํหนดให 2.4 ใบงานที่ 9.4 Reading เรียงขอความทีก่ําหนดให เปนบทสนทนาเกี่ยวกับการถามทิศทางที่ไดใจความ

2.5 ใบงานที่ 9.5 Writing เขียนบทสนทนาเลียนแบบตวัอยางเกีย่วกับการสอบถามการเดินทาง โดยใชขอมูลที่กําหนดให แลวจับคูพูดบทสนทนา

1. Language Focus: Expression and sentences in giving direction 1.1 Asking the way and giving direction

Asking the way Giving direction

• Excuse me, where’s the hotel?

• Excuse me, can you tell me the way to the library?

• Excuse me. How do I get to the bank from here?

• Which way is the museum?

• Do you know where the post office is?

• Turn right / left at the corner.

• Turn right / left into Rama Road.

• Take the first turning right / left.

• Go up this street and turn right / left.

• Drive down Highway.

• Go east / west / north / south on Highway Road.

• Walk along the road to the traffic light. 1.2 Asking for clarification and useful expression

Asking for clarification Useful expression

• Up this street two blocks and left?

• So I go up the intersection and turn right.

• Do I take the first or the second street on the left?

• Is the bank on the right or on the left?

• So for the bank, I go down two blocks and turn left?

• It’s not very far.

• It’s about ten minutes’ walk.

• It’s about two blocks.

• You can’t miss it.

1.3. Asking , Accepting and Refusing

Asking Accepting and Refusing

where there’s a bank?

Do you know the (shortest) way to………? how to get to……………? where ……………is? How do I get to…………? can there? Can you direct me to………….?

- Yes, there’s one on……….. - Yes, you’ll find it on……….. - Certainly, it’s not far. - I’m sorry. I don’t know. - I’m sorry. I have no idea.

1.4 Imperatives verb Structures: Imperatives Catch train

Take the micro bus from………….. bus no. 17

Keep straight until you see the……… Walk along this road f or……….. Go down till you reach the……….. ahead as far as……………

Imperative verbs look like "bossy" verbs: the verbs in Do this, don't do that, go away are clearly cases of throwing your weight around. Notice, incidentally, that imperative verbs like these are always in present time. It's not possible to issue a command in the past, eg *Didn't do that. And there is no pronoun or noun acting as a subject to the verb. If you needed to put one into the sentence, it would have to be you, as in the kind of disagreements that sound like this:

You go No- you go! Sentences like these, with an imperative verb in them, are called directive sentences. But it would be misleading to say that every directive sentence has a bossy, commanding meaning. Look, for example, at these:

directive sentence imperative verb possible purpose of sentence

Go and see a doctor Go advice

Mind out for the mosquitoes Mind (out) warning

Have a nice holiday Have expression of good wishes

Help me! Help pleading

Come over here please. Come requesting

Come round for a drink tonight. Come invitation

One other thing to notice about the grammar of the imperative verb: in its positive form there is only one verb in the verb phrase (as in the examples above). If we were to turn them into negatives - Don't go, Don't mind out, Don't have etc, then there will be two verbs, and the auxiliary will always be the imperative don't. So, if you come across "bossy" sentences like I'm

telling you now - you will go tomorrow, because I say so, it isn't an imperative verb that's being used. It's a modal verb, making a confident prediction and giving the listener no choices.

The way how to use imperative Example Direct commands Follow me. Request Open the window .(please) Suggestion Take first turning. Warning Watch out! There’s a motorcycle. Direction Go straight for 10 minutes walk. Instruction Use a microwave and boil for 5 minutes. Invitation Come and have birthday party tonight. Prohibition Take off your shoes. Offer Help yourself

Practice Conversation 1 A: Where is the public library, please? B: It is in Broad Street, near the Town Hall. A: How do I get there? B: Well, you could walk. But it’s better to take a bus number 14. It will take you straight there. A: Thanks a lot. Conversation 2 A: Excuse me, could you tell me where the Scala Cinema is , please? B: Let me see. Oh yes, of course, I know. It’s in Lincoln Square, opposite the Asia Hotel. A: Is that near here? B: Oh yes, it’s just a few minutes walk. Turn left at the traffic lights and you’ll see it. A: Thank you very much. Conversation 3 A: Excuse me, please. Could you tell me where the Speedy Car-Hire office is? B: I’m sorry. I have no idea A: I think it’s somewhere off City Avenue. Do you know where that is? B: I’m afraid I don’t. I’d never been here before.

ใบงานที่9.1 Warm up : Vocabulary Study Exercise 1 Study the vocabulary about the direction and find 10 words to fill in the blanks below.

รูปภาพ 9.1 English for Communication 1 ทวี โอมาก หนา 110 1……………………………………………… 6…………………………………………… 2……………………………………………… 7…………………………………………… 3……………………………………………… 8…………………………………………… 4……………………………………………… 9…………………………………………… 5……………………………………………… 10…………………………………………..

ใบงานที่ 9.2 Listening Exercise 2: Study the map before you begin. Listen to the speaker giving directions to different places on the map. Then write ‘True’ or ‘False’

รูปภาพ 9.2 http://www.kr.ac.th/ebook/ben/t03.html .......... 1. The hotel is next to the bank. .......... 2. The zoo is opposite the police station. .......... 3. The library is between the post office and the supermarket. ........... 4. The bowling alley is on the East Street. ............5. The bar is on the corner of West Street and South Street. ............6. City hall is in front of the library. ............7. The hospital is near the bus station. ............8. The zoo is behind the post office. ............ 9. The bowling alley is behind the bookstore. ............10. The school is between the bus station and the police station.

ใบงานที่9.3 Speaking Exercise 3 You are staying with your friend who lives at 46 Calton Drive. Work in pairs. ask your friend how you can get to the places shown on the map. Notice that the road goes under the railway.

รูปภาพ 9.3 Break Through 1 J.C. Richards & M.N. Long หนา 44 1. The hospital. A:…………………………………………………………………………………………. B:…………………………………………………………………………………………. ……………………………………………………………………………………………. 2. The Bangkok bank. A:…………………………………………………………………………………………. B:…………………………………………………………………………………………. ……………………………………………………………………………………………. 3. The restaurant. A:…………………………………………………………………………………………. B:…………………………………………………………………………………………. ……………………………………………………………………………………………. 4. The railway station. A:…………………………………………………………………………………………. B:…………………………………………………………………………………………. …………………………………………………………………………………………….

ใบงานที่9.4 Reading Exercise 4 Rearrange these sentences to be the complete conversations and then practice with your partner. Conversation 1

- The railway station? - Excuse me, how do I get to the station, please? - Yes, that’s right. I want to get to the railway station. - Thanks a lot. - Go straight on. It’s about a kilometer down the road, on your left.

You: …………………………………………………………………………………………… Man: ……………………………………………………………………………………………You: ……………………………………………………………………………………………Man: ………………………………………………………………………………………….. You: …………………………………………………………………………………………… Conversation 2

- It’s about 25 minutes walk - Thank you very much. - Yes. Turn right, and then take the second turning on your left. - Excuse me. Can I get to the Central Department Store this way? - You’re welcome. - Is it far? - And do you know where the National Bank is? - Yes, it’s on this street. Keep walking for two blocks and it’s on the corner, on the right.

You: ………………………………………………………………………………………… Man: ………………………………………………………………………………………… You: ………………………………………………………………………………………… Man: ………………………………………………………………………………………… You: …………………………………………………………………………………………Man: ………………………………………………………………………………………… You: ………………………………………………………………………………………… Man: …………………………………………………………………………………………

ใบงานที่9.5 Writing and Speaking Exercise 5 Make new dialogues with the information given below.

Destination Transportation Period of time Where to catch/park 1. the shopping center The sky train 5 minutes At the cross road 2. the airport The subway 15 minutes Two building block 3. school The bus 30 minutes At the victory monument 4. the golf club My car 1 hour The parking lot of the club

Example: A: How do you get to work?

B: I take the bus. A: How long does it take? B: About forty-five minutes. A: Where do you catch the bus? B: There’s a bus stop near my house. Conversation 1 A: ........................................................................................................................................................ B: ........................................................................................................................................................ A: ........................................................................................................................................................ B: ........................................................................................................................................................ A: ........................................................................................................................................................ B: ........................................................................................................................................................ Conversation 2 A: ........................................................................................................................................................ B: ........................................................................................................................................................ A: ........................................................................................................................................................ B: ........................................................................................................................................................ A: ........................................................................................................................................................ B: ........................................................................................................................................................

Conversation 3 A: ........................................................................................................................................................ B: ........................................................................................................................................................ A: ........................................................................................................................................................ B: ........................................................................................................................................................ A: ........................................................................................................................................................ B: ........................................................................................................................................................ Conversation 4 A: ........................................................................................................................................................ B: ........................................................................................................................................................ A: ........................................................................................................................................................ B: ........................................................................................................................................................

Answer Exercise 1 1. Sorry to 2. where 3. turn 4. on 5. you’re welcome 6. excuse 7. to 8. going 9. until 10. On Exercise 2 1.True 2. Ture 3. False 4. False 5. Ture 6. False 7. True 8. False 9. True 10. False Exercise 3 1. A: Do you know where is the hospital? B: Yes, go along Calton Drive, turn left into Lancaster Street. Then turn righ to Oxford Road pass the railway station. The hospital is on your left. 2. A: Excuse me. How do I get to the Bangkok Bank? B: Go along Calton Drive, turn left into Lancaster Street. Go straight ahead until you see the cross road. The Bangkok Bank is on the opposite corner. 3. A: Can you tell me where is the restaurant, please? B: Go along Calton Drive, turn left into Lancaster Stree. The restaurant is opposite the junction of Lancaster Street and Oxford Road. 4. A: Excuse me. Do you know how to get to the railway station? B: Go along Calton Drive and turn left into Lancaster Street. You’ll see it. It’s not far. Exercise 4 Conversation 1

You: Excuse me, how do I get to the station, please? Man: The railway station? You: Yes, that’s right. I want to get to the railway station. Man: Go straight on. It’s about a kilometer down the road, on your left. You: Thanks a lot.

Conversation 2 You: Excuse me. Can I get to the Central Department Store this way? Man: Yes, Turn right, and then take the second turning on your left. You: Is that far? Man: It’s about 25 minutes walk. You: And do you know where the National Bank is?

Man: Yes, it’s on this street, Keep walking for two blocks and it’s on the corner, on the right. You: Thank you very much. Man: You’re welcome.

Exercise 5 Conversation 1 Conversation 2 A: How do you get to the shopping center? A: How do you get to the air port? B: I take the sky train. B: I take the subway. A: How long does it take? A: How long does it take? B: About 5 minutes. B: About 15 minutes. A: Where do you take the sky train? A: Where do you take the subway? B: There’s the sky train station at the cross road. B: There’s the subway station for two building block. Conversation 3 Conversation 4 A: How do you get to the school? A: How do you get to the golf club? B: I take the bus. B: I drive my car. A: How long does it take? A: How long does it take? B: About 30 minutes. B: About 1 hour. A: Where do you take the bus? A: Where do you take your car? B: There’s the bus stop at the victory monument. B: At the parking lot of the club.

รายวิชา ภาษาอังกฤษเพื่อการสื่อสาร 2 Unit 10

Unit 10 Tell Me about Yourself

Unit 10 Tell me about yourself

สาระสําคัญ การศึกษาเกี่ยวกับการถามและการใหขอมูลสวนตัว ทั้งโดยการฟง การอาน การพูด การเขียน เปนสิ่งที่จําเปนตอการใชภาษาในการสื่อสาร โดยเฉพาะกรณีที่จะตองใหขอมูลในการสัมภาษณงาน จึงเปนสิ่งจําเปนอยางยิ่งที่เจาตวัจะตองสามารถใหขอมูลเกี่ยวกับตนเองไดครบถวน จุดประสงคการเรียนรู ความรู

1. อธิบายการคําถามและคําตอบเกี่ยวกับการใหขอมูลสวนตัวได 2. เขาใจแนวทางเขียนและกรอกเกีย่วขอมูลสวนตวัในแบบฟอรมได

ทักษะ 3. สรุปคําถามและคําตอบเกีย่วกับการใหขอมูลตนเองเพื่อการสัมภาษณที่ถูกตอง 4. วิเคราะหเลือกใชคําศัพท สํานวน ประโยคเกี่ยวกับการใหขอมูลที่ถูกตอง

คุณธรรม 5. สงเสริมความเชื่อมั่นในตนเองแกผูเรียนดวยการปฏิบัติการถามและการตอบการใหขอมูล

ตนเองของการสัมภาษณที่ถูกตองและเหมาะสม เนื้อหาสาระ 1. Language Focus 1.1 Vocabularies : occupations / at works 1.2 Opening the interviews

1.3 Asking about education / qualifications 1.4 Asking about capabilities 1.5 Exchanging information about work. 1.6 Occupations and skills

1.7 Wh- questions with do /Adjectives before nouns 2. Worksheet 2.1 ใบงานที่ 10.1 Warm up: ดูภาพและบอกชื่ออาชีพใหตรงกับภาพ 2.2 ใบงานที่ 10.2 Listening and speaking: ฝกฟงบทสนทนาแลวเติมขอความใหสมบรูณ และเลือกคําตอบที่ถูกตอง

2.3 ใบงานที่ 10.3.Reading: ฝกอานตอบขอมูลสวนตัวของแตละบุคคล 2.4 ใบงานที่ 10.4 Writing: ฝกวิเคราะหขอมูลและเขียนบรรยายลักษณะของแตละอาชพี

1. Language Focus: 1.1 Vocabularies:

- Occupations:

รูปภาพ 10.1 The Basic Oxford Picture Dictionary by Margot F. Garmer หนา 86 1. secretary 2. typist 3. file clerk 4. computer programmer 5. messenger 6. photographer 7. reporter 8. businessman / businesswoman 9. accountant 10.lawyer 11.salesperson 12. baby sister 13. dancer 14. singer 15. actor / actress 16. Artist

- At work:

รูปภาพ 10.2 The Basic Oxford Picture Dictionary by Margot F. Garmer หนา 88 A. fix /repair TVs / appliances B . fix / repair cars C. fix / repair pipes D. cut hair E. cut meat F. cut grass G. sell clothes H. sell vegetables I. sell newspapers J. build house K. build furniture

1.2 Opening the interviews: - Tell me about your free time activities. - Tell me about yourself. - Could you tell me a little about yourself? - Do you have any trouble finding us?

1.3 Asking about education / qualifications

Ask Answer - What are your qualifications? - What qualification do you have?

- I got a certificate / diploma / degree in……… - Besides my Vocational Education Certificate in Electronics I took an evening class in………….. - After graduation. I took a short course in……… - I’m studying English at…………….

1.4 Asking about capabilities

Ask Answer - Can you use a computer? - What kind of computer / program can you use? - Can you speak English / Chinese? - Can you arrange a product / presentation?

- Yes, I can use Microsoft Office / the internet. - Yes, I can speak………… - No, but I’m taking a evening class in…………. - Yes, I can. - No, but I can always learn. - No, I can’t but I can learn quickly.

1.5 Exchanging information about work.

Ask Answer - Where do you work? - When do you start / finish work? - Do you take a break in the afternoon? - What do you do after work? - Where does your brother work?

- I work in a / an……………………. - I start / finish work at………………. - Yes, I do. - I…………………………………… - He works at…………………………

1.6 Occupations and skills

Occupation Skill - a dishwasher - a nurse’s aid - a plumber - a telephone - a truck driver - a painter - a cashier - a hair dresser - a receptionist - a salesperson

- washes dishes - help nurses - fixes toilet and sink - install telephone - drive a truck - paint house - use a cash register - cut hair -greet visitor - sells things

Practice:

A B - What is her job? -She is a doctor. - Who wears a uniform? - A police officer wears a uniform. - Where do you work? - I work in a hotel.

รูปภาพที่ 10.3: NEW INTERCHANGE INTRO หนา 48

1.7 Wh- questions with do - What do you do there? -. What does she do there? Adjectives before nouns

Pattern: be + adjective Adjectives + noun 1. A police officer’s job is dangerous. – A police officer has a dangerous job. 2. A doctor’s job is stressful. – A doctor has a stressful job. 3. An accountant‘s job is difficult. – An account has a difficult job. 4. A gardener’s job is relaxing. – A gardener has a relaxing job. 5. An athlete‘s job is exiting. – An athlete has an exciting job. 6. A teacher’s job is boring. – A teacher has a boring job. Example

A B - Do you have a job? - Where do you work? - What do you do, exactly? - What time do you start work? - When do you finish work? - Do you take a break in the afternoon?

- I am a cashier. - He works in a department store. - I work as a foreman at ABC company. - He starts work at 10:00 A.M., - He finishes at 6:00 P.M. - Yes, I have a break for one hour.

ใบงานที่10.1 Warm up: Listening and Speaking Exercise 1: Look at the picture. Listen and write the sentence about the job used the information given below.

a) cashier b) cook / chef c) doctor d) flight attendant e) judge f) lawyer g) musician h) nurse i) pilot j) police officer k) receptionist l) salesperson m) security guard n) singer o) waiter p) waitress

รูปภาพ 10.3 New Interchange INTRO หนา 46 1. He is a receptionist. 9. …………………………………………….. 2. She is a ……………………. 10………………………………………………. 3. She is a ……………………. 11………………………………………………. 4. He is a …………………….. 12……………………………………………….. 5. She is a ……………………. 13………………………………………………. 6. He is a …………………….. 14………………………………………………. 7. She is a ……………………. 15……………………………………………….. 8. She is a ……………………. 16…………………………………………………

ใบงานที่10.2 Listening Exercise 2: Listen to the conversation and fill in the blanks. A: I’m looking for a (1)………………………………. B: (2)………………………..What skill do you have? A: Well, I can (3)……………………….a computer. B: What (4)……………………..your last job? A: I was a mechanic. B: Do you want to fill out an application? A: Yes, (5)…………………………………. Exercise 3: Listen to the personal information and then choose the correct answer. 6. What is the man’s name?

a) Criag b) Gerg c) Gary d) Gray 7. Where was the man born?

a) Arizona b) Kansas c) Colorado d) Texas 8. Which sentence best describe his university status?

a) He is a third – year student. b) He is majoring in Spanish b) He really enjoys his studies. d) He is originally forming Denver.

9. Where does the man work? a) At a bookstore b) at a restaurant c) at a supermarket d) at university

10. What is one thing the man like to do in his free time? a) reading b) jogging c) camping d) studying

ใบงานที่10.4 Reading Exercise 4: Read Greg’s personal information and choose the given words to fill in the blanks. company attending cashier originally hanging economics majoring junior grocery graduate Hi! My name is Greg and I’m (1)……………………from Denver, Colorado, but my family moved to Arizona when I was about 3. So I grew up in there. I graduated from high school about three years ago, and I am currently (2)………………a university in my city. I’m a(3)…………and I am a (4)………………………in(5)…………………with a minor in Spanish. I also work par time as a (6)……………………..at a (7)…………………store. Life is really busy, but I enjoy hiking, reading and(8)………………out with my friends in my free time. When I (9)……………....... I want to work for a (10)……………………in this area. Exercise 5: Who do you think say this? Write the name of the person and occupation

Name Occupation 1.”Sometime I work a lot, but sometime I don’t.” …………………… ……………………… 2. “This year, I’m helping with the soccer team.” …………………… ……………………….. 3.”I’m not always happy about my decisions.” ……………………. ………………………...

ใบงานที่ 10.4 Writing

Exercise 6: Complete the chart with your opinion by checking in which occupation you agree and write ten occupations in this pattern.

Example: An actor is a difficult job. 1……………………………………………………………………………………………………….. 2……………………………………………………………………………………………………….. 3……………………………………………………………………………………………………….. 4……………………………………………………………………………………………………….. 5……………………………………………………………………………………………………….. 6……………………………………………………………………………………………………….. 7……………………………………………………………………………………………………….. 8……………………………………………………………………………………………………….. 9………………………………………………………………………………………………………... 10……………………………………………………………………………………………………….

ใบความรูท่ี 10 Wrap up: Interviewing Skills An interview is an opportunity for an employer and the candidate to get to know one another and assess each other. As the candidate, your goal is to elaborate on your resume and to obtain firsthand information about the organization and the job. The employer will educate your personality, attitudes and aptitudes in relation to the job and the culture of the organization. The interview is your opportunity to evaluate a prospective employer as well as the prospective employer’s opportunity t evaluates you. Some Phrase: - I’m looking for a job. - What was your last job? - Do you want to fill out an application? - Do you have any experience? - Well, not really. - So, ............ ? (to introduce a questions) - Oh, me? - That’s interesting Occupations and skills 1.a dishwasher - washes dishes 2. a nurse’s aide - helps nurses 3. a plumber - fixes toilet and sinks 4. a telephone - installs telephones 5. a truck driver - drives a truck 6. a painter - paints houses 7. a cashier - uses a cash register 8. a hairdresser - cuts women’s hair 9. a barber - cuts men’s hair 10. a receptionist- greets visitors 11. a salesperson - sells things 12. a dressmaker – sews women ‘s clothes 13. a tailor - sews men ‘s clothes 14. a welder - welds the metal 15. a technician - repair equipments

Answer Exercise 1 1. receptionist 2. doctor 3. nurse 4. pilot 5. flight attendant 6. musician 7. singer 8. judge 9. police officer 10.lawyer 11. cook/chef 12. waiter 13. waitress 14. salesperson 15. cashier 16. security guard Exercise 2 1. job 2. Good 3. use 4. was 5. thanks Exercise 3 6. b 7. c 8. a 9. c 10. a Exercise 4 1. originally 2. attending 3. junior 4. majoring 5. economics 6. cashier 7. grocery 8. hanging 9. graduate 10. company Exercise 5 1. Name: Molly Swift / Carpenter 2. Name: Benjamin Morse / teacher 3. Name: Joseph Todd / Judge Exercise 6 แนวคําตอบ 1. An actor is an exciting job. 2. A DJ is a relaxing job. 3. A fashion design is a selling job. 4. A gardener is an easy job. 5. A librarian is a boring job. 6. A pilot is a dangerous job. 7. A police officer is a difficult job. 8. A receptionist is a stressful job. 9. A teacher is a boring job. 10. A waiter is a selling job. เฉลยแบบทดสอบประจําหนวยท่ี 10 1. B 2. A 3. C 4. D 5. A 6. B 7. D 8. C 9. B 10.C

รายวิชา ภาษาอังกฤษเพื่อการสื่อสาร 2 Unit 11

Unit 11 Reading Advertisement

บทที่ 15 Reading Advertisement

สาระสําคัญ การอานถือวาเปนทักษะที่สําคัญอยางหนึ่งที่ใชบอยในชวีิตประจําวัน ผูอานไดรับความรูเพิ่มขึ้นจากการอาน กลวิธีในการอานภาษาอังกฤษจงึเปนสิ่งจําเปนและสําคัญอยางยิ่งสําหรับผูเรียน โดยเฉพาะอยางยิ่งกับสิ่งที่พบในชีวิตประจําวัน เชน หนงัสือพิมพ โฆษณา ปายประกาศตางๆ เปนตน จุดประสงคการเรียนรู ความรู

1. มีความรูความเขาใจ เกีย่วกับสื่อส่ิงพิมพที่ใชในการโฆษณาทั่วไป และโฆษณารับสมัครงาน 2. พัฒนาทักษะการฟงเกี่ยวกับการหางานในสื่อส่ิงพิมพ 3. พัฒนาทักษะการอานโฆษณาและฝกทักษะการพูดเกีย่วกับคุณสมบัตขิองการประกาศงาน 4. ใชภาษาและอกัษรยอที่มีอยูในประกาศโฆษณาของสื่อส่ิงพิมพ

ทักษะ 5. อานและจําแนกประเภทของสื่อโฆษณาจากการอานสื่อส่ิงพิมพไดถูกตอง 6. ฟงและระบเุทคนิคการโฆษณาในสื่อส่ิงพมิพไดอยางเหมาะสมแตละประเภทของสือ่ส่ิงพิมพ 7. บอกความหมายและสนทนาเกี่ยวกับคณุสมบัติผูสมัครงานตามที่ประกาศในสื่อส่ิงพิมพ 8. เขาใจและใชคาํเต็มของอักษรยอในโฆษณาประกาศรับสมัครงานไดถูกตอง

คุณธรรม 9. สรางลักษณะนิสัยของการมคีวามคิดริเร่ิมสรางสรรคในการฟง การพดู การอาน และการเขียนเกี่ยวกับขอมูลของสื่อส่ิงพิมพและสงเสริมการรักการอานแกผูเรียนในการคนควาสื่อและใชเทคโนโลยีที่เหมาะสม เนื้อหาสาระ 1. Language Focus: 1.1 Type of advertisement 1.2 Language of Advertisement 1.3 Information Questions (Wh-question)

2. Worksheet 2.1 ใบงานที่ 11.1 Warm up: ทําแบบฝกหดัฟงบทสนทนา Conversation 1-3แลวเลือกตอบคําถาม 3 ขอและ จับคูชนิดของ Advertisement

2.2 ใบงานที ่11.2 ทําแบบฝกหัดอาน Advertisement ตางๆ แลววเิคราะหวาเปนชนิดใด 2.3 ใบงานที่11.3 ทําแบบฝกหัดจับคูอาชีพ และลักษณะงานที่ทํา และฝกพูดบทสนทนาจากสถานการณทีก่ําหนดให 2.4 ใบงานที่11.4 ทําแบบฝกหัดวเิคราะหโฆษณาสมัครงานแลวนาํมาเขียนตอบตามหัวขอ 2.5 ใบงานที่ 11.5 ทําแบบฝกหัดเขียนโฆษณาขายรถยนต โดยใชคําศัพท สํานวนที่กําหนดใหมาเขียนใหเปนโฆษณาที่เหมาะสมถูกตอง 1. Language Focus: 1.1 Type of Advertisement

- Business opportunity - Contract and Tender - Medical and Health Care - Tour and Travel - Educations

1.2 The Language of Advertisement

No. Vocabulary Meaning 1 newspaper (noun) - a set of large fold sheets of paper containing news, articles, pictures,

advertisements, etc. printed on daily or weekly. (หนังสือพิมพ) Ex. He usually buys a newspaper before going to work.

2 come across (phrasal verb)

- to meet someone or something by chance (พบโดยบังเอิญ) Ex. Have you ever come across such a horrible person in all your life?

3 include (verb) - contain someone or something as a part (รวม) Ex. This price includes dinner, bed and breakfast.

4 accommodation (noun) - a place for someone to stay, live or work in (ที่อยู/ ทีพ่ักอาศัย) Ex. Rented accommodation

5 property (noun) - the thing or things that someone owns (ทรัพยสิน)

Ex. That’s my personal property! Leave it alone!

6 local (adjective) - connected with a particular place or area, especially the place you live in (ทองถ่ิน) Ex. Plenty of local food can be found in this supermarket.

7 look for (verb) - to try to find (คนหา) Ex. We’ve been looking for you everywhere. Where you been?

8 normally (adverb) - usually (ตามปกติ) Ex. I normally leave the house at 8 o’clock.

9 for sale ประกาศขายสนิคาตาง ๆ 10 car for sale / Auto for sale ประกาศขายรถยนต 11 wanted ประกาศแจงความจํานง เชน ตองการพนักงานขาย 12 - employment – position

vacant ประกาศรับสมัครบุคคลเขาทํางาน

13 to let ประกาศใหเชาหรืออาคารตาง ๆ 14 travel and holidays โฆษณาเกีย่วกบัการบริการทองเที่ยว 15 services โฆษณาเกีย่วกบัการบริการตาง ๆ เชน ซอมแซมบาน รถยนต 16 announcement ประกาศแจงเหตุการณหรือกําหนดการณตางๆ บางครั้งใสคําวา events 17 educational ประกาศขอมูลเกี่ยวกับการศึกษา 18 entertainment โฆษณาเกีย่วกบัการบันเทิง เชน ละคร ภาพยนตร

1.3 Information Questions (Wh-question)

• What do you do at weekend? - I usually read some magazines.

• When do you read the newspaper? - In the morning.

• Which advertisement do you find? - Job advertisement.

• How do you find it? - From the newspaper.

• Who always reads the announcement? - Ken does.

ใบความรูท่ี 11 Wrap up: Here are some classified ad group headings. Personal Wanted: - Personnel Wanted - EFL Teachers - English Teacher

- Teachers & Tutors Property Guide: - Business Opportunities - Properties for Sale - Business for Sale

- Business Services - Galleries - Business Wanted - House for Sale - Apartments or Houses for Rent - House Agents - Business Premises - Land - Housing Wanted - Real Estate Agents

Sales and Services Guide: - Pets - Article - Article wanted - Building Materials - Schools & Universities - Items for Hire

Tours and Accommodations: - Tour & Travel - Travel Agents - Hotels - Airlines Autos: - Autos for Sale - Auto Rentals - Autos Wanted

- Auto Repairs - Parts & Accessories -Boats Announcements: - Personal - Announcements - Lost & Found

- Contract & Tenders - Churches - Birthdays Where to Lunch & Dine Out: - Clubs - Restaurants - Coffee Shops Entrainment: - Bars - Pubs - Discothèques - Massage Parloirs

Abbreviations used in the property section of ‘classified ads.’ brm / b/r = bathroom bath = bathroom refridge = refrigerator 80 sqm. = 80 square meter Din rm = dining room A/C = air – conditioner 8th Fl. = 8 thFloor direct tel. = direct telephone 150 sq.w = 150 square wah 76,000 kms= 76,000 kilometers

ใบงานที่ 11.1 Warm up: Listening Exercise 1: Listen and choose the best answer. Conversation 1: On the phone. Receptionist: Classified section. Tina Suksawat speaking. May I help you? Caller: I want to place an advertisement. Receptionist: I’ll put you through to Mr. Wong. He’s our marketing manager. Caller: Thank you. Conversation 2: In an office. A: Are you doing anything this weekend? B: Well, I don’t have any plans at the moment. A: Do you play tennis or golf? B: Golf? Yes. A: Where would you suggest? B: My wife said the other day about the new golf course. She saw the ads in the classified section. I’ll phone her to find out more. A: Great Conversation 3: In the canteen. A: Top place in town...pool and snooker....guaranteed fun and happiness. B: What are you talking about Henry? A: Look at this club (the speaker shows his friend classified ads..) Napoleon Club. B: What’s special about it? A: Impressive place... I’m sure we’ll have good fun there! Questions 1. According to conversation 2, we can summarize that ...................................... a) the two speakers are old friends. b) the two speakers have been working together for long time c) the two speakers doesn’t seem to know each other well.

2. Based on the information in conversation 3, we can summarize that the first man is reading some information about..................................... a) a new swimming pool b) a new club c) a new restaurant 3. Based on the information in conversation 1, the place where the receptionist works would probably be .............................................. a) a police station b) a newspaper publishing c) a department store

Exercise 3: Listen and match the classified ads in column A. with the group heading in column B.

Answer Column A. Column B. ................. 4. English summer course a. Auto for Sale ................. 5. leasing a shop house b. Teachers & Tutors ................. 6. planning for holidays c. Pets ................ 7. teaching math d. Announcements ................. 8. selling a dog e. Tour & Travel .................. 9. announcing the death of the General Manager f. Special Courses ................. 10. buying a used car g. Obituary

ใบงานที่ 11.2 Reading Exercise 3: Match the advertisement with the topic given.

………………….1. Education ………………….2. Tour and Travel ………………….3. Medical and Health ………………….4. Food ………………….5. Business Opportunity

A B

C

D

E

ใบงานที่ 11.3 Reading and Speaking Exercise 4: Look at the list of jobs. Match the job with the definition (a – j) …………………1. Teacher a. Treats the sick. …………………2. Doctor b. Write contract, advises companies on the law. …………………3. Secretary c. Works for the government. …………………4. Accountant d. Help people to learn. …………………5. Salesman e. Usually works in a lab. …………………6. Research scientist f. Serves in restaurant. …………………7. Civil servant g. Files planes …………………8. Lawyer h. Sells products. …………………9. Pilot i. Checks financial results. …………………10. Waiter j. Assists, word-professor, makes appointments Exercise 5: Ask your partner about the activities for more details. Follow the example. Example: A: Do you read sport magazine? B: Yes, I do. A: What sport do you like? B: I like tennis. A: How often do you play it? B: Once a week.

1. Read business magazine? What? When? 2. Work on a computer? Where? 3. Entertain customer? In restaurant? How often? 4. Go on a trip? How often? 5. Take holiday? Where?

ใบงานที่ 11.4 Reading: Exercise 6: Study this advertisement and write 1 – 5 identify its parts. รูปภาพ 11.1 หนังสือพิมพ Bangkok Post Saturday, December 15, 2007

1. Place of contact……………………………………………………………………………… ……………………………………………………………………………………………….

2. Fringe benefit……………………………………………………………………………….. 3. Qualifications……………………………………………………………………………….

……………………………………………………………………………………………… ……………………………………………………………………………………………...

4. Position…………………………………………………………………………………….. 5. Office Name………………………………………………………………………………..

ใบงานที่ 11.5 Writing Exercise 7: Suppose you want to sell your car urgently. Write a short advertisement. Give the brand name and model of your car, condition, price and any other necessary details as a guide.

1. The car needs to be sold urgently 2. Brand name: BMW 3. Model: 5201 4. Color: White 5. Condition: good 6. Other details: well maintained 7. Price: B 500,500 8. Distance driven: only 6,000 kms. 9. Contact: Khun Kamon Kulwadee

10. Telephone: 0-2320-2950 Time: Office hours

Auto Sale

1.................... 2....................... 3.................. 4.....................

5...................... 6.................... 7......................

8.......................... 9.......................... 10...........................

10......................

แบบทดสอบประจําหนวยท่ี 11 Read the rental of advertisements and choose the best answer. B A D C The choice A / B /C / D / are for question 1 – 10 ............ 1. Which ads should you choose if you want rent a house which is near school? ............ 2. Which ads should you choose a house that is close downtown? .............3. Somsri is looking for a big house. .............4. Jane is looking for a furnished apartment. Which ads should she choose? .............5. Dr. Prapon wants to be close to downtown and a big house. Which ads should he do? ..............6. Joe wants a washer and dryer. Which ads should she choose? ..............7. Nancy doesn’t want to a live near downtown. Which ads should she choose? ..............8. Jack wants an apartment with parking. Which ads should she choose? ..............9. Noi has a dog. Which ads should she choose? ..............10. You have a family. Which ads should you choose?

Big 2-bdm. 1-ba house Unfurn. Stove, refrigerator

Quiet neighborhood. Near school. $ 400 mo. Rent.

$ 300 deposit. Call 0-2481-9472

Furn. 2 – bdm apt 69 A Avenue. Near downtown

No pets. Laundry. Parking. $ 350 rent. One mo’s rent deposit.

Call 0-1555-9600

Frun. 1-bdm house. 412 South St. Close to

downtown. Pets OK. All appliances.

Parking $ 300. Plus $300 deposit.

Call 0-6481 - 7320

Unfurn 1-bdm apt. 414 Spencer St. near Fleet

Elementary, stove, refrigerator

$200 rent. Plus $100 deposit.

Call 0-955-4123

Answer Exercise1 1. C 2. B 3. B Exercise 2 4. F 5. G 6. E 7. B 8. C 9. D 10. A Exercise 3 1. B 2. A 3. C 4. E 5. D Exercise 4 1. D 2. A 3. B 4. I 5. H 6. E 7. C 8. J 9. G 10. F Exercise 5

1. A: Do you read business magazine? 2. A: Do you work on a computer? B: Yes, I do. B: Yes, I do. A: What do you read? A: Where do you work? B: I read fashion. B: At my office. A: When do you read it? B: At night.

3. A: Do you entertain customer? 4. A: Do you go on a trip? B: Yes, I do. B: Yes, I do. A: What do you entertain him? A: Where do you go? B: In restaurant. B: At Sarika water fall. A: How often do you entertain him? B: Once a month. 1. A: Do you take holiday? B: Yes, I do. A: What do you take holiday? B: At Chiengmai

Exercise 6 1. Place of contact: 25TH Floor, Empire Tower 3, 195 South Sathorn Road, Yannawa Sathorn, Bangkok 10120 Thailand. (Email: [email protected]) 2. Fringe benefit: Experience in food industry 3. Qualifications: 5 years experience, good command of English, be able to drive 4. Position Purchasing Representative 5. Office Name German – Thai Chamber of Commerce

Exercise 7 1. Urgently Sale 2. BMW 3. 5201 4. White color 5. Good condition 6. well maintained 7. 500,500 B. 8. only 6,000 kms. 9. Khun Kamon Daruneet 10. Tel: 0-2320-2950 / Time: Office hours แบบทดสอบประจําหนวยท่ี 11 1. A 2. B 3. A 4. D 5. C 6. C 7. A 8. B 9. C 10. A

รายวิชา ภาษาอังกฤษเพื่อการสื่อสาร 2 Unit 12

Unit 12 Making a Phone Call

Unit 12 Making a Phone Call

สาระสําคัญ

ในปจจุบันนับเปนยุคของการสื่อสาร การติดตอมีความสะดวกอยางมาก นับเปนการติดตอส่ือสารอยางไรพรมแดน เรียกไดวาสามารถติดตอกันไดทุกที่ทุกเวลาโดยทางโทรศัพท ภาษาจึงเปนสิ่งจําเปนที่สําคัญอยางยิ่ง การใชภาษาไดยางถูกตองชัดเจนชวยใหการสื่อสารประสบความสําเร็จ

จุดประสงคการเรียนรู

ความรู 1. เขาใจความหมายของสํานวนในการพดูโทรศัพท ในสถานการณตางๆ 2. อานฟงและจับใจความจากบทสนทนาทางโทรศัพท ทักษะ

3. พูดและใชสํานวนตางๆ ในการพูดโทรศัพทในแตละสถานการณได

4. พูดบทสนทนาเกีย่วกับการติดตอทางโทรศัพทในสถานการณตาง ๆ โดยใชคําศัพท สํานวน ประโยค และไวยากรณได

คุณธรรม

5.สงเสริมสนับสนุนการมีมารยาทในการพูดและรับโทรศัพทเพื่อสรางนิสัยความสภุาพเกิดแกผูเรียนโดยการโตตอบทางโทรศัพท

เนื้อหาสาระ

1. Language Focus

1.1 Expressions as the speaker and the caller

1.2 Expressions in the negative response

1.3 Expressions in the business

1.4 Leaving a message or phone number

1.5 The present continuous tense

2. Worksheet 2.1ใบงานที่ 12.1 Warm up ทําแบบฝกหัดโดยเรียงลําดับขอความใหถูกตองของบทสนทนา ในรูปแบบของการติดตอส่ือสารทางโทรศัพท 2.2ใบงานที่ 12.2 Listening ทําแบบฝกหัดโดยฟงจับใจความและเลือกคําตอบที่ถูกตองเติมบท สนทนาใหสมบูรณ 2.3ใบงานที่ 12.3 Speaking ทําแบบฝกหัดโดยฝกพูดบทสนทนาทางโทรศัพทตามที่กําหนดใหดวยกิจกรรม pair work 2.4 ใบงานที ่ 12.4 Reading ทําแบบฝกหัดอานและนําคําที่กําหนดใหมาเขียนบทสนทนาใหสมบูรณ 2.5 ใบงานที ่ 12.5 Writing ทําแบบฝกหัดเขียนเติมบทสนทนาดวยสํานวนประโยคที่ถูกตองตามหลักไวยากรณ 2.6 ใบงานที่ 12.6 Writing ทําแบบฝกหดัโดยจับคูคําตอบของการติดตอส่ือสารทางโทรศัพทของ Speaker 1 & Speaker 2 2.7 ใบงานที่ 12.7 Writing ทําแบบฝกหัดเขียนบทสนทนาตามผังที่กําหนดให 1. Language Focus

1.1 Expressions as the speaker and the caller

Asking as the speaker Answering as the caller - Good morning, BBC Company. Could I help you? - May I speak to Mrs. Shaw, please? - Could I speak to Mrs. Malee, please? - I’d like to speak Mr. Somsak, please? - This is Elizabeth. - Is Paula there?

- Good morning. Suchada Jaidee speaking. - Who’s calling, please? - Just a moment. I’ll connect you. - Manoon speaking. - I’m on the line.

1.2 Expressions in the negative response

Asking caller to wait Negative response - Could you hold? - Hold a moment please. - Hold on please. - Just a moment please. - Just a second, I’ll get him. - Hold the line please.

- He‘s busy at the moment. - She’s occupied at the moment. - He’s on another line. - I’m sorry that number’s engaged. - I’m afraid he’s not in at the moment. - I’m sorry he’s not on another line. - I’m sorry he’s out right now. - I’m sorry he’s in a meeting on a holiday. - I’m afraid he can’t speak to you. - I’m afraid she can’t come to the phone.

1.3 Expression in the business

Start the business Asking caller to repeat - I’m with BBC Company and I need to talk with Mr. Jones about the catalog. - Do you have any idea when he is back? - Ask her to call me, please. - Please tell him that I called.

- Excuse me, what company did you say? - I’ sorry. I didn’t hear you. - I beg your pardon. - Pardon.

1.4 Leaving a message or phone number

Leaving a message Answering - Would you like to call back later? - May I take a message? - Would you like to leave a message? - Could you call back later? - What’s your number, please? - Could I have your phone number, please? - I want to leave a message.

- Yes, please. - Certainly, sir. - Yes, of course. - OK. Yes, that’s right.

Operator Charlie

Practice Conversation 1 Operator: Good evening, ABC Company. Can I help you? Charlie: I’d like to speak with Khun Nattawut, please. Operator: Please hold. I’ll put you through. Sorry, Khun Nattawut is not in. Can I take a message? Charlie: I want to inform him that there will be an urgent meeting this evening. Operator: Can I have your name and number, I’ll ask him to call you when he come back. Charlie: Thank you. My name is Udom and my number is 0 – 584 – 521 – 648. Secretary Christ Conversation 2 Christ: My name is Christ. Is Ms Susan in? Secretary: May I ask the reason of your call? Christ: Ms. Susan asked me to check the status of her order with us. Secretary: Ms. Susan is not in at the moment if you give me your name and number. I’ll ask her to call you back. Christ: Yes, I’m Christ Green. Secretary: I’ll give her the message.

Conversation 3 A: Hello? B: Hi, Susan. This is Bill. Can I talk? A: Well, I’m very busy tight now. I’m sorry. Can I call you back? B: Sure. No problem. Talk to you later. A: Thanks. Bye. B: Bye. Susan Bill

1.5 The present continuous tense is: am / is / are + v+ing I am (not) -ing I’m working. He Tom is writing a letter. She is (not) –ing She isn’t eating. It The telephone is ringing. We We’re having dinner. You are (not) –ing You’re not listening to me. They The children are doing their homework. Making appointment by phone

ใบความรูท่ี 12 Wrap up

Situation (สถานการณ)

Very Polite (สุภาพมาก)

Polite (สุภาพ)

Casual (ไมเปนทางการ)

1. ถามหาคน - May I speak to Somchai? - I’d like to talk to Somchai.

- Aree, please. - Is Somchai there?

2. สอบถามวาตองการติดตอใคร

- Who would you like to speak to? - Who are you calling for? -------------------------

3. แสดงสถานะ ------------------------------------------ - This is Aree. - This is Aree from ABC.

-------------------------

4. สอบถามสถานะของผูที่โทรศัพทมา

- May I ask who is calling, please? - May I have your name, please?

- Who’s calling please? - Who is this?

5. แสดงชื่อหนวยงาน - This is the Suksom Mansion. - ABC Company, may I help you?

-------------------------------- ------------------------

6. โปรดรอสักครู - Could you wait for just one moment, please?

- One moment, please. Wait a moment.

7. กําลังยุงอยู/สายไมวาง - He is not available at the moment. - She is not available at the moment.

-His line is busy right now. -He is on another line. - She is on the phone at the moment.

-----------------------

8. ฝากขอความ / โปรดโทรกลับ

- Could you please take a message? - Could you ask her to call me back?

- May I leave a message? - Please tell her I called.

------------------------

9. โทรกลับ ------------------------------------ - I’m sorry. I missed your call this morning. - I’m returning your call from this morning. - This is Aree, returning your call from this morning.

- I heard you called me.

ใบงานที่ 12.1 Warm up Exercise 1: Rearrange the following sentences in the logical order by putting a) to j)

Answer Sentences .................. 1. That’s right. Please tell her our meeting will be on Monday at 2:30. .................. 2. Hello. May I speak to Ms. Maria, please? .................. 3. Yes, please. This is Mrs. Anna. .................... 4. 0-2359-6871. Yes, Mrs. Anna I’ll give Ms. Maria the message. ..................... 5. I’m sorry. She’s not in. Can I take a message? ..................... 6. Good morning. A&E Industries? ...................... 7. Monday 2:30. ........................ 8. Thank you. Good – bye. ....................... 9. And would you ask her to call me this afternoon? My number is 0-2539-6871. ...................... 10. Good –bye. A: …………………………………………………………………………………………………….. B: …………………………………………………………………………………………………….. A: …………………………………………………………………………………………………….. B: …………………………………………………………………………………………………….. A: …………………………………………………………………………………………………….. B: …………………………………………………………………………………………………….. A: …………………………………………………………………………………………………….. B: …………………………………………………………………………………………………….. A: …………………………………………………………………………………………………….. B: ……………………………………………………………………………………………………..

ใบงานที่ 12.2 Listening Exercise 2: Listen to two conversations and fill in the blank with the given word. Conversation 4 hold on go ahead I’ve got as soon as could Linda: Jack is not here right now. Would you like to leave the message? Helen: This is his wife.(1.)............................ you ask him to call me back at my office. Linda: I’ll take that down (2)....................... a moment, please OK. (3)............................................ Helen: My office number is 0-245-784-631 extension 115. Linda: Yes, (4) .....................................that. 0-245-784-631 extension 115. Right? Helen: That’s right. Linda: I’ll tell him to call you (5) ................................ he gets back. Helen: Thank you very much. Conversation 5: speak up calling hear hang up no one Jill: Hi, Andrew, I’ve been (6).......................you for a long time. But the line has been busy. Andrew: Hello. Could you (7) ...................... a little, please? I can hardly hear you. Who’s calling? Jill It’ Jill! Can you (8)....................... me? Andrew: Hello, Jill. What’s wrong with the phone? Is your battery dying? Jill: The line is terrible. Let me (9).................... and call you right back. (Jill dials again.) Man: Hello? Jill: Is that Jill? Man: There’s (10).............................by that name here. I think you have the wrong number.

ใบงานที่ 12.3 Speaking

Exercise 3: Pair work. Talk with your partner according to your role.

Student A Suda: Hello. Is that 02-457-3584? Ratana: ………………………………………………………………………………… Suda: Is John there? Ratana: ………………………………………………………………………………… Suda: Yes, please tell him that the party will be on next Friday 6.00 p.m. at the Star Hotel. Ratana: ……………………………………………………………………………….. Suda: That’s right. Thank you. Ratana: ……………………………………………………………………………….. Student B Suda: ………………………………………………………………………………. Ratana: Yes, Ratana speaking Suda: ………………………………………………………………………………. Ratana: No, he is out. Would you like to leave a message? Suda: ……………………………………………………………………………….. Ratana: All right. The party will be at the Star Hotel on next Friday 6.00 p.m. Suda: ……………………………………………………………………………… Ratana: You’re welcome.

ใบงานที่ 12.4 Reading Exercise 4: Read these two conversations and fill in the blanks by using your own words. Conversation 6: Make an appointment. Speaker: Doctor Jamras’s office. How can I help you? Joo : This is Joo. I need to make an appointment. I hurt my back. Speaker: Oh, I’m sorry. How about tomorrow at 9:30? Joo : Tomorrow at 9:30? That’s fine. See you then. Speaker: Feel better! Joo : Thanks a lot. Pair work: (Problem: I hurt my foot / I have the flue) Speaker: Doctor. (1).................’s office. How can I help you? Caller : I need to make an appointment. I... (2).................................... Speaker: Oh. I’m sorry. How about. (3)...................at (4).........................? Caller: (5)........................? That’s fine. See you then. Speaker: Feel better! Caller: Thanks a lot. Conversation 7: Make a phone call. Dan : Hello? Bill : Hi, Dan. This is Bill. Can you talk? Dan : Well, I’ m fixing a door right now. I’m sorry. Can I talk you back? Bill : Sure. No problem. Talk to you later. Dan : Thanks. Bye. Pair work: Speaker: Hello? Caller : Hi, (6)...................... This is ... (7).................. Can you talk? Speaker: Well, I’m.... (8).................. right now. I’m sorry. (9)..........................................? Caller : (10)....................... Bye. Speaker: Bye.

ใบงานที่ 12.5 Writing

Exercise 5: Write positive or negative response to these conversations.

1. A: Sathita mansion. Good morning. B: --------------------------------------------------------------------------------------------------------- 2. A: Hello. Is that 0 – 3221 – 1466? B: --------------------------------------------------------------------------------------------------------- 3. A: May I speak to John, please? B: --------------------------------------------------------------------------------------------------------- 4. A: Please tell Pong that I will call back again this evening. B: --------------------------------------------------------------------------------------------------------- 5. A: Could I have your phone number, please? B: --------------------------------------------------------------------------------------------------------- 6. A: Oh, sorry. The line is busy. B: --------------------------------------------------------------------------------------------------------- 7. A: Is that John? B: Sorry. There’s no one here by that name. A: Oh, I’m sorry. I dialed the wrong number. B: ----------------------------------------------------------------------------------------------------------- 8. A: Sorry. Dr. Pitak is in a meeting. B: When should I call back? A: --------------------------------------------------------------------------------------------------------- 9. A: Extension 107, please. B: --------------------------------------------------------------------------------------------------------- 10. A: Is that 0-2465 – 5061? B: ---------------------------------------------------------------------------------------------------------

ใบงานที่ 12.6 Writing Exercise 6: What’s happening at the moment? Write the true sentences.

1. ( I / wash / my hair) I’m not washing my hair. 2. (it / rain) ……………………………………………………………………………… 3. ( I / sit / on a chair) ……………………………………………………………………………… 4. ( I / drink ) ……………………………………………………………………………… 5. (I / study / English) ……………………………………………………………………………… 6. (I / telephone) ……………………………………………………………………………… 7. (I / listen / to the radio) ……………………………………………………………………………… 8. (the sun / shine) ……………………………………………………………………………… 9. ( I / talk) ……………………………………………………………………………… 10. (I / copy / my homework) ………………………………………………………………………………

ใบงานที่ 12.7

Exercise 7 Write the conversation A: Ask to speak to…………. B: Tell him/her he is in a meeting. A: Ask when he’ll free? B: You don’ know. A: You want to leave a message. B: Ask for caller’s name and number. B: Give name and number. A: Note and repeat the message. A: Thanks him.

แบบทดสอบประจําหนวยท่ี 12 Choose the best answer. 1. Do you know what the international ......................... is for France? a) connection b) code c) number d) dial 2. I couldn’t get through to him because the line was .................................. a) busy b) interrupted c) engaged d) occupied 3. Phone me on my ....................., anytime, night or day. a) mobile b) cell c) walking phone d) handy 4. Hello. Could you ................me through to Mr. Herman, please? a) carry b) take c) bring d) put 5. We’ve got a phone in the room and an ...................... in the bedroom. a) connection b) extension c) cord d) link 6. Hello. Pete ........................ John here. a) Here’s b) I’m c) It’s d) That’s 7. If Sandy’s not here. Can I ...................... a message, please? a) leave b) give c) take d) make 8. I hate talking to ................. phones if someone is not in. a) repeat b) answer c) talking d) message 9. I can’t hear you. This is a very bad .................................. If you hang up, I’ll ring you back. a) line b) number c) wire d) phone 10. Hello. Is that Martin? Yes, .................................................. a) talking b) speaking c) I am d) he is

Answer Exercice 1 A: Good morning. A&E Industries? B: Hello. May I speak to Ms. Maria, please? A: I’m sorry, she’s not in can I take a message? B: Yes, please. This is Mrs. Anna. A: And would you ask her to call me this afternoon? My number is 0-2539-6871. B: 0-2539-6871. Yes, Mrs. Anna I’ll give Ms. Maria the message. A: That’s right. Please tell her our meeting will be on Monday at 2.30. B: Monday 2.30. A: Thank you. Good bye. B: Good bye Exercise 2 1. Could 2. hold on 3. go ahead 4. I’ve got 5. as soon as 6. calling 7. speak up 8. hear 9. hang up 10. no one Exercise 3 ประเมินผลตามเกณฑประเมินการพูดบทสนทนา Exercise 4 แนวคําตอบ 1. name 2. I hurt my foot. 3. tomorrow 4. 10.00 5. Tomorrow at 10.00 6. Name 7. Name 8. fixing a door 9. Can I take you back 10. Thanks Exercise 5 แนวคําตอบ 1. Good morning. Can I help you? 2. Sorry. You have the wrong number. 3. I’m afraid she can’t speak to you right now. Could you call back later? 4. OK. Thanks. 5. 0 – 2282 – 9436. Thank you. 6. You’re welcome. Bye. 7. I’m sorry. There is no one is here now. 8. about 10 am. 9. No, it is not. 10. I’m sorry to have bothered you. Exercise 6 แนวคําตอบ 2. It’s raining. 3. I’m sitting on a chair. 4. I’m not drinking. 5. I’m studying English. 6. I’m not telephoning. 7. I’m not listening to the radio. 8. The sun is shining. 9. I’m talking 10. I’m not copying my homework แบบทดสอบประจําหนวยท่ี 12 1. B 2. A 3. A 4. D 5. B 6. C 7. A 8. B 9. A 10. B

รายวิชา ภาษาอังกฤษเพื่อการสื่อสาร 2 Unit 13

Unit 13 Telephone Message

รูปภาพ 13.1: http:// web2.uvcs. Ca/elc/sample/ beginner/nt/wt – 33htm.

Unit 13 Telephone Messages

สาระสําคัญ ในการรับโทรศัพทแทนผูอ่ืน จะตองมีการฝกฝนทักษะการบันทึกขอความที่ผูโทรเขามาฝากขอความไว เชนการจดชื่อของผูฝากขอความ วนั เวลาที่รับโทรศัพท ขอความที่รับฝากควรใหชัดเจนและกระชับวาบุคคลนั้นไดฝากอะไรไวบาง และลงชื่อผูรับฝากขอความนั้น จุดประสงคการเรียนรู ความรู

1. ฟงและสรุปใจความของการสนทนาทางโทรศัพทได 2. เลือกใชคําศัพท สํานวน ในการฝากและรับฝากขอความทางโทรศัพท 3. พูดสอบถามขอมูลและเขียนขอมูลที่จําเปนในการรับฝากขอความทางโทรศัพท

ทักษะ 4. ฟงและระบุขอมูลที่จําเปนในการสนทนาเพื่อฝากและรับขอความทางโทรศัพทไดถูกตอง 5. เขียนขอความที่รับฝากได 6. แสดงบทบาทของการพูดโตตอบและเขยีนขอมูลเพื่อส่ือการทางโทรศัพทใดตามสถานการณ

ที่กําหนด คุณธรรม

7. เสริมสรางความพยายามในการฝกโตตอบและเขียนขอความภาษาอังกฤษในการฝากขอความทางโทรศัพท เนื้อหาสาระ 1. Language Focus

1.1 Instructions for writing messages 1.2 Telephone Messages 1.3 Be Back Soon Notes: 1.4 Object pronouns

2. Worksheet 2.1 ใบงานที ่13.1 Warm up ฝกทักษะการเขียน Telephone Messages โดยเลือกขอความเติมตามหัวขอที่กาํหนดใหใหไดใจความ 2.2 ใบงานที่ 13.2 Listening and Speaking ฝกทักษะการฟงบทสนทนาทางโทรศัพทจับใจความสําคัญ แลวเติมคําในชองวางใหสมบูรณดวยคําทกีําหนดใหและนํา Conversation ที่สมบูรณมาทําการฝกการพูดโตตอบกันเปนคูๆ 2.3 ใบงานที่ 13.3 Reading ฝกทักษะการอานทําความเขาใจบทสนทนาแลวเลือกคําตอบที่ถูกตองตามเนือ้หาของบทสนทนาที่อาน 2.4 ใบงานที่ 13.4 Writing ฝกทักษะการเขยีน โดยการวเิคราะหเนื้อหาของบทสนทนาแลวนําเอาขอมูลที่ไดมาเขียนคําตอบลงในหวัขอที่กําหนด และนําคํามาเรียงใหเปนประโยคที่ไดใจความ

2.5 ใบงานที่ 13.5 Writing: ทักษะการเขียน Telephone message จากบทสนทนาที่อาน 1. Language Focus:

1.1 Instructions for writing messages: On a clean sheet or new file include: 1.1.1 Requestor 1.1.2 What is requested? 1.1.3 Verification 1.1.4 Special Information 1.1.5 Contact information

1.2 Telephone Messages 1.2.1 Format: There is no particular format for taking telephone messages. Some

business use blank note pads for the messages. Other designs their own formats. 1.2.2 Important points: Besides the message itself. These important points

should be included in your formats. 1. Name of caller 2. Date and time of calling 3. Phone numbers of callers 4. Person who takes message Example:

Telephone Message Date: December 25 Time: 3.00 p.m. Telephone call for Mr. Gary Gellelr Caller’s name. Miss Mary Smith Caller’s number 02-547-6986 Messages The meeting on Friday is postpone to next Monday.

Taken by Usa

Example: 1.3 Be Back Soon Notes:

Give one note to the roommate. When people ask for the person, the roommate will convey the information in the note to the caller. Example: Practice Conversation Answering machine: Hi. This is Jennifer, and this is Nicole. We can’t come to the phone right now. Please leave us a message, and ....................................... Nicole: Hello? Michael: Hi. Nicole? It’s Michael. Is Jennifer there? Nicole: Oh, hi, Michael. She isn’t in. Can she call you later? Michael: Yeah, thanks. Please ask her to call me tomorrow. Nicole: Sure. Just give me the number. Michael: It’s 555 – 0367. Nicole: 555 – 0367. OK. Michael: Thanks a lot, Nicole.

I’m going down town to have lunch. Be back at 4.00. - Frieds

I’m going to the college to talk to my professor. Be back at 2.00. - Tomoko

1.4 Object pronouns

Pronouns Object pronouns I me

You your he him she her it it

we us they them

Example: - Just give me the number. - Send the news to you as soon as possible. - Please tell him to call back. - Give her a call later. – I don’t have it. - I would like to speak us now. - Can she call you later? - I saw them to use their telephone. Helpful Tip: Be sure to repeat the caller’s name, number, and message before you hang up to confirm that you have the correct information.

ใบความรูท่ี 13 Leaving a message Sometimes there may not be anyone to answer the phone and you will need to leave a message. Follow this outline to make sure that the person who should receive your message has all the information he / she needs.

1. Introduction: Hello, this is Ken or Hello. My name is Ken Beare. 2. State the time of day and your reason for calling:

- It’s ten in the morning. - I’m phoning (calling, ringing) to find out if…/ to see f…/to let you know that…./ to

tell you that….. 3. Make a request:

- Could you call (ring, telephone) me back? - Would you mind…..? 4. Leave your telephone number:

- My number is…… - You can reach me at……… - Call me at…………… 5. Finish:

- Thanks a lot, bye. – I’ll talk to you alter, bye. Here’s an example of message: Telephone: (ring…ring….ring) Hello this is Tom. I’m afraid I’m not in at the moment. Please leave a message after the beep .(beep) Ken: Hello Tom, this is Ken. It’s about noon and I’m calling to see if you would like to go to the movie on Friday. Could you call me back? You can reach me at 02-245- 6835 until five this afternoon. I’ll talk to you later, bye. - As you can see, leaving a message is pretty simple. You only need to make sure that you have stated all the most important information: your name, the time, the reason for calling, your telephone number

• Writing Tips: Telephone Message 1. Write the name of the person the caller wanted to talk to. 2. Write the name of the caller. 3. Write the details about the call. 4. Write your name.

ใบงานที่ 13.1 Warm up Exercise 1: Look at the telephone message form and fill in with the of information given. Many people who work in offices must answer the telephone for other people. This form shows what kind of information you must write in the message.

Telephone Message Date ………………………………………..………... (1) Time …………………………………..……………. (2) To ……………………………………...…………… (3) From …………………………………………………... (4) Telephone #…………………………………….……... (5)

(6) (7)

Called to see you Will call again Wants to see you Returned your call

Message: ……………………………………………………………………….(8) ……………………………………………………………………….(9) Received by: ……………………………………(10) Answer:

1. .................................................... 2..................................................... 3. .................................................... 4..................................................... 5. .................................................... 5..................................................... 7. .................................................... 7.................................................... 9. .................................................... 10..................................................... 02-547-6358 Bob Williams Janet Climes 14/09/07 Marry Telephoned please call 10.45 She wants to know what time the meeting starts.

ใบงานที่ 13.2 Listening & Speaking Exercise 2: Listen to a telephone conversation between Claire and Female. And fill with the appropriate words. number take make message May Goodbye! tell back he’s Can I help Claire: Hello, finance department. Female: Hello, (1).......................................... I speak to Adrian Hopwood, please? Claire: I’ m afraid (2)...........................n a meeting at the moment. (3)…………………..……..? Female: No I need to talk to Mr. Hopwood, I think. What time will he be out of the meeting? Claire: In about an hour. Can you call (4)……………………….. later? Female: Okay, I’ll do that. Claire: Or can I (5)……………………….. a message? Female: Actually, would you mind? Could you (6)………………… him that Jennifer Mc Andrews called and that I’m in the office all day if he could call me back? Claire: Can I take your (7)………………………… please? Female: Yes, it is 5556872. Claire: 5556872. Okay, I’ll (8)……………………… sure he gets the (9)……………………. Female: Thanks very much for your help, bye! Claire: (10)……………………………! . รูปภาพ13.2: Picture Dictionary (OXFORD)

ใบงานที่13.3 Reading Exercise 3 Read two conversations and answer the questions below. Conversation 1 Michelle: Hello, you’ve reached the marketing department. How can I help? Male: Yes can I speak to Rosallnd Willson, please? Michelle: Who’s calling please? Male: It s Ricahrd Davis here. Michelle: Certainly. Please hold and I’ll put you through. Male: Thank you. Conversation 2 Michelle: Hello, marketing. How can I help? Male: Could I speak to Jason Roberts please? Michelle: Certainly. Who shall I say is calling? Male: My name‘s Mike Andrews. Michelle: Just a second – I’ll see if he’s in. Hello, Jason. OK I’m just putting you through 1. When you telephone a company the person answering the phone may ask you a question. Which is the correct question? a) Who’ s calling please? b) Who calls? c) Who it is? d) Who called? 2. Which phrase means the same as ‘hang on a moment?’ a) Just a second b) I’ll put you on c) Go ahead d) I’m ready 3. Choose the correct word: “Please .......... and I’ll put you through. a) stop b) stay c) talk d) hold 4. What is the expression used to connect two people on the telephone? a) I’m sending you through. b) I’m putting you through. c) I’m calling you through. d) I’ m talking you through. 5. What question the person answering may ask to offer some help? a) How can I help you? b) I’m putting you through. c) Just a second. d) Who’s calling?

ใบงานที่ 13.4

Writing Exercise 5: Now that you’ve finished calling all of your friends go back to your home and retrieve the messages that your friends left for you. Use the conversation as a guide. A: Hello. B: Hello. Is John there please? A: No, he’s not here right now. B: This is (1) Mark, John’s classmate form university. A: Oh. Hi, Mark. B: Do you happen to know where he went? A: He said he was (3) going (2) downtown to do some (3) shopping. B: Do you know when he’ll be back? A: He told me he would be back by (4) 4 o’clock. Can I take a message? B: Yes. Please tell him there’s a party tonight. And could you tell him to call me when he gets in? A: Sure. Does he know how to get a hold of you? B: I’m at Harold’s Department Store right now. My number is 250 – 684 – 3234 A: I’ll right. I’ll tell him you called. B: Thanks a lot. Bye. Message while you were out: Fill in the table with the underlined information.

Name Location Activity Expected Return Time (1) (2) (3) (4) Rewrite the words into the correct sentences. 5. take / can / your/ message / I / a / please? / ................................................................................................................................................................... 6. later? / call / can / back / you .................................................................................................................................................................. 7. meeting. / afraid / I’m / she’s / in ...................................................................................................................................................................

8. message. / I’ll / sure / make / OK / gets / he / the ................................................................................................................................................................... 9. not / in / He’s / at / the / office / his / the moment, ................................................................................................................................................................... 10. I / please? / Can / Mr. June / to / Wilkinson / speak ...................................................................................................................................................................

รูปภาพ 13.3 : ภาษาอังกฤษสถานประกอบการ ฉลวย มวงพรวน หนา 72

ใบงานที่ 13.5 Writing Exercise 6 Read the telephone conversation and write the message for the person wanted on the phone. A: Amarin Hotel. Can I help you? B: I’d like to speak to Mr. John Young, please. He’s in room 548. A: I’m sorry, there’s no answer from room 548. May I take a message? A: Yes, please. My name is Suda. Please tell Mr. Young that the seminar on Friday at 9.30 a.m. is cancelled. He should come to the office on Monday at 3.00 p.m. instead. B: Right, I’ve got that. I’ll make sure he got this message. A: Thank you.

Telephone Message Date:………………………………………..Time………………………………………… For…………………………………………………………………………………………. From……………………………………………………………………………………….. Message……………………………………………………………………………………. ……………………………………………………………………………………………… Taken by……………………………………………………………….

I’m going down town to have lunch. Be back at 4.00. - Frieds

I’m going to the college to talk to my professor. Be back at 2.00. T k

Messages Please call back the number 555 – 0367 to Mr. Michale - Nicole.

แบบทดสอบประจําหนวยท่ี 13 Choose the best answer. 1. Hello. Could you .................. me through to Mr. Herman, please? a) carry b) take c) bring d) put 2. If Sandy’s not there, can I ............... a message, please? a) leave b) give c) take d) make 3. I’m sorry. Mr. Herman can’t get to the phone right now. He’s a bit ............... Can he call you back? a) help up b) stuck up c) worked up d) tied up 4. I’m afraid I can’t give you an answer today. I’ll give you a................. first thing tomorrow with our decision. a) phone b) call back c) ring d) contact 5. What is the expression used reply as your receptionist for taking a message? a) Can I take a seat? b) Can I have breakfast? c) Can I take a message? d) Can I have lunch? 6. What is the useful phrase in telephone messages? a) OK, I’ll make sure he gets the message. b) OK. I’ m sure. c) OK, it is all right. d) OK. Good bye. 7. As soon as I realized the caller was a salesman, trying to sell me.................. a) to b) with c) on d) from 8. Yes, Ms Drummond is expecting your call, if you‘ll just ............ a second, I’ll put you through. a) take b) wait c) see d) call 9. What should you say to reply the telephone? “He’s not in his office at the moment.” a) Can I use a computer, please? b) Can I take a message, please? c) Can I swim, please? d) Can I get a taxi, please? 10. What is the instruction for writing messages? a) Special information b) Personal information c) Educational information d) Healthy information

Answer Exercise 1 1. 02-547-6358 2. 10.45 3. Bob Williams 4. Janet Climes 5. 14/09/07 6. Telephoned 7. Please call 8. She wants to know 9. what time the meeting starts. 10. Marry Exercise 2: Typescript: Can I take a message? Claire: Hello, finance department. Female: Hello, May I speak to Adrian Hopwood, please? Claire: I’ m afraid he’s in a meeting at the moment. Can I help? Female: No I need to talk to Mr. Hopwood, I think. What time will he be out of the meeting? Claire: In about an hour. Can you call back later? Female: Okay, I’ll do that. Claire: Or can I take a message? Female: Actually, would you mind? Could you tell him that Jennifer Mc Andrews called and that I’m in the office all day if he could call me back? Claire: Can I take your number, please? Female: Yes, it is 5556872. Claire: 5556872. Okay, I’ll make sure he gets the message. Female: Thanks very much for your help, bye! Claire: Goodbye! Exercice 3 1. A 2. A 3. D 4. B 5. A Exercice 4 1. Mark 2. Downtown 3. Go shopping 4. 4 o’clock 5. Can I take your message, please? 6. Can you call back later? 7. I’m afraid she‘s in the meeting. 8. OK. I‘ll make sure he gets a message. 9. He’s not in his office at the moment. 10. Can I speak to Mr. June Wilkinson please? เฉลยแบบทดสอบประจําหนวย 1. A 2. A 3. D 4. C 5. C 6. A 7. C 8. B 9. B10 A

รายวิชา ภาษาอังกฤษเพื่อการสื่อสาร 2 Unit 14

Unit 14 Signs and Notices

Unit 13 Signs and Notices

สาระสําคัญ ปายและประกาศเปนเครื่องมือหรือสัญลักษณที่มีเครื่องหมายเฉพาะเปนที่เขาใจที่ยอมรับกันใน

การปฏิบัติตามของการจัดระเบียบสังคม การสื่อความหมายเปนรูปภาพ เปนขอความสั้นที่ไดขอความชัดเจน ผูเรียนควรอานและทําความเขาใจกับถอยคําสํานวนที่นยิมใชและจุดประสงคของปายและประกาศนัน้สําหรับการใหความรวมมือปฏิบัติตามไดถูกตอง จุดประสงคการเรียนรู ความรู 1. ใชคําศัพท สํานวน และไวยากรณที่เกี่ยวกับปายและประกาศ

2. อานออกเสียงอธิบายความหมาย คําศัพท สํานวน เกี่ยวกับปายและประกาศได 3. เขาใจเกีย่วกบัขอมูลเกี่ยวกบัปายและประกาศ

ทักษะ 3. ฝกอานระบขุอมูลภาษาอังกฤษที่มีอยูปายและประกาศไดถูกตอง 4. ฝกอานขอความในปายและประกาศแลวเติมขอความใหสมบูรณไดถูกตอง

5. ฝกพูดบทสนทนาแลกเปลี่ยนขอมูลเกี่ยวปายและประกาศไดอยางเขาใจ คุณธรรม

6. สรางลักษณะนิสัยของความซื่อสัตยโดยการปฏิบตัติามขอมูลที่ระบุอยูในปายและประกาศ ไดถูกตองตามกฎเกณฑที่กําหนด เนื้อหาสาระ 1. Language Focus: 1.1 Signs Giving Information

1.2 Sign giving Warning

1.3 Signs giving Prohibition 1.4 Signs stating rules or regulations 1.5 Sign giving order or command 1.6 Notices 1.7 Ways to give instructions

1.8 Ways to express warning

1.9 Order or Command: - Imperative - Prohibition

2.Worksheet 2.1 ใบงานที่ 14.1 Warm up ฝกออกเสียงคาํศัพทและบอกความหมายคาํศัพทเกี่ยวกับ ปายและ

ประกาศจากรูปภาพ 2.2 ใบงานที่ 14.2 Listening and speaking ฝกออกเสียงและพูดตามคําศพัทปายและประกาศ 2.3 ใบงานที่ 14.3 Reading ทําแบบฝกหัดอานทําความเขาใจความหมายของปายและประกาศ

และนํามา เขยีนเปนประโยคภาษาอังกฤษ 2.4ใบงานที่ 14.4 Writing ทําแบบฝกหดัเขียนขอความในปายและประกาศที่กําหนดใหตรง

กับภาพของปายและประกาศ 1. Language Focus:

1.1 Signs Giving Information: These signs are used on workplaces or factories to give information of facilities. People might need them in case of fire and injury

1.2 Sign giving Warning: The signs often found on roads indicate the road or traffic situation.

1.3 Sign giving Prohibition: Some signs order you not to do something.

1.4 Signs stating rules or regulations: These signs are also widely used in workplaces or factories as safety rules or working regulation

1.5 Sign giving order or command: Some traffic signs order you to do something.

1.6 Notices:

1.7 Ways to give instructions

Positive

verb object adverbial Wash Shake

this shirt in warm water well before use.

Negative

Do not / Never verb object adverbial Do not Never

feed expose

infants to flame.

Example: - Call for an ambulance. - Don’t pull anyone injured out of a car. - Always wear head protection.

Warning: There are different ways to express a warning. If you do not do as the warning tells you, something bad may happen or you will be in danger.

1.8 Ways to express warning You must + verb = order You should + verb = advice Example: - You must fasten a safety belt while driving a car. - You should follow the college regulations.

1.9 Order or Command - Imperative - Slow penguins crossing. - Stop for police check. - Prohibition - No smoking! - Don’t park here!

ใบงานที่13.1

Warm up Exercise 1: Look at these signs and think where you can see it. 1 2 3 4 5 6 7 8 99 9 10 1……………………………………. 2……………………………………………… 3……………………………………. 4……………………………………………… 5……………………………………. 6……………………………………………… 7……………………………………. 8……………………………………………… 9……………………………………. 10………………………………………………

ใบงานที่13.2 Listening Exercise 2: Listen to the information below and match them with the sign

1. MEN WORKING BELOW 2. ROAD NARROW 3. STOP 4. NOT FOR TEENAGER 5. PEDESTRAIN CROSSING 6. FLAMABLE 7. WEAR RESPIRATOR 8. KEEP OFF THE GRASS 9. NO TRUCK 10. METERS HIGH

A B C D

E F G H

I J K

Giving information

Giving warning

Giving prohibition

Regulation Giving order Notice

ใบงานที่13.3

Reading Exercise 3: Look at these signs which sign A B C

D E F G

H I J

1. …………….. shows you a sign for a handicap? 2. ……………..suggested that “this area you cannot enter”? 3. ……………..show you the way out? 4. ……………..suggested you to be away from it? 5. ……………..tell you not to smoke here? 6. ……………..advices you not to cross the road? 7. ……………..tell you that there’s a room for rent? 8. ……………..suggested you to use the speed limit? 9. ……………..show you the place of library? 10. ……………..tell you there is a woman in that room?

ใบงานที่13.4 Writing Exercise 4: Write these signs in Thai. A B C

D E F G

H I J

1. ……………..………………………………………………………………………….. 2. …………………………………………………………………………………………. 3. …………………………………………………………………………………………. 4. …………………………………………………………………………………………. 5. …………………………………………………………………………………………. 6. …………………………………………………………………………………………. 7. …………………………………………………………………………………………. 8. …………………………………………………………………………………………. 9. …………………………………………………………………………………………. 10. ………………………………………………………………………………………….

ใบงาน 13.5 Writing Exercise 5: Complete the sign and notice with the word in the box. And practice saying them. always careful care open except form provided to unplug clean

1. Please………………..queue. 2. Store cord in a manner………………………….. 3. ………………………..packing area. 4. Do not……………….the valve. 5. ………………….appliance when not in use. 6. ……………..use a hammer of the correct size. 7. Be……………………..not to break it. 8. Acid with……………………………… 9. No admittance……………………………..on business. 10. Please keep your room………………………………

แบบทดสอบประจําหนวยท่ี 14

Look at the sings which sign:

a. Suggests that you can not park your car. …………………….. b. Shows you a kind of beverage. …………………….. c. Shows you a sign for a handicap. …………………….. d. Advices you to careful of radiation. ……………………. e. Tells you not to turn right. ……………………. f. Shows you the pedestrian crossing. ……………………. g. Advices you of the slippery road. ……………………. h. Suggests you to be in speed limit. ……………………. i. Tells you that it can recycle. ……………………. j. Advices you to beware of dog. ……………………

1 2 3

4 5 6 7

8 9

10

Answer Exercise 1 ฝกการฟงจากผูสอนอานและผูเรียนอานตาม Exercise 2 แนวคําตอบ a) Greeting are typical in my country by Wai and say ‘Sawasdee Kha” หรือ ‘Sawas Krab” b) 1. America – Shaking hand 2.Finland - Hugging 3. England – Kissing 3. Japan – bowing slightly 5. New Zeeland – Nose to Nose Exercise 3 Conversation: At the party. 1. How do you do? 2. I don’t think we’ve met. 3. Where are you from? 4. I’m from America. 5. Nice to meet you. Exercise 3: Conversation: At the office. 1. My name is Gerald Whitman. 2. Are you Mr. Hardy? 3. Please sit down 4. Who is that? 5. Please sit down. Exercise 4: Conversation : แนวคําตอบ 1. three 2. He will go home tomorrow. 3. No. 4. Yes. 5. In an office Exercise 5: 1. Get 2. go 3. have 4. start 5. have 6. finish 7. go 8. have 9. go 10. sleep Exercise 6: 1. can’t play 2. wears 3. smoke 4. sell 5. go 6. don’t see 7. doesn’t 8. washes 9. know 10. costs . Exercise 7 Wanna: (1) ..Hi! I’m Wanna. Wanna: .(2)Please to meet you too. Wanna: .(3) Me too. My major is Electronics. Wanna: .(4) Goodbye เฉลยแบบทดสอบประจําหนวยท่ี 13

a. 7 b. 4 c. 2 d. 6 e. 9 f.10 g.1 h.5 i.3 j. 8

รายวิชา ภาษาอังกฤษเพื่อการสื่อสาร 2 Unit 15

Unit 15 Instructions

Unit 15 Instructions

สาระสําคัญ ในปจจุบนัมีจาํนวนของเครือ่งมือ อุปกรณมากมายหลายประเภททั้งชนิดที่ผลิตดวยเทคโนโลยีที่ซับซอนซึ่งผูใชจะตองมีความรูเกี่ยวกับวิธีใช คําสั่ง คําแนะนํา วิธีปฏิบัติ เปนสิ่งจาํเปนในการใชอุปกรณตางๆ ผูใชตองสอบถามหรืออานคูมือ ในการปฏิบัติตามขั้นตอนไดอยางถูกตอง และจะชวยใหใชงานไดอยางมีประสิทธิภาพ และยังชวยปองกนัอนัตรายที่อาจเกิดขึ้นได จุดประสงคการเรียนรู ความรู

1. อานและเขาใจเกีย่วกับคําศัพท สํานวน และประโยคในการใชเครื่องมือ อุปกรณ 2. บอกวิธีการและขั้นตอนการใชเครื่องมือ อุปกรณ 3. เขาใจและระบุคําแนะนําที่ควรปฏิบัติและไมควรปฏิบัติในการใชอุปกรณและเครื่องมือ ทักษะ

4 . อานและบอกความหมายของคําศัพท สํานวน ประโยคในการใชเครื่องมือและอุปกรณถูกตอง 5. สนทนาโตตอบวิธีการใชเครื่องมอื อุปกรณตามลําดับขั้นตอนการใชเครือ่งมืออยางถูกตอง 6. เขียนและพูดแนะนําขอควรปฏิบัติและไมควรปฏิบัติในการใชอุปกรณและเครื่องมือไดถูกตอง คุณธรรม

7. เกิดลักษณะนิสัยของเขาใจความสําคัญในการปฏิบัติตามขั้นตอนการใชเครื่องมือ อุปกรณได ถูกตองทําใหอายุการใชงานของเครื่องมือ อุปกรณไดนาน และกอใหเกิดความประหยัด เนื้อหาสาระ 1. Language Focus 1. Language Focus: 1.1 One-step instruction

1.2 Multi-instruction 1.3 Asking for step in doing things. 1.4 Sequence markers

2. Worksheet 1. ใบงานที่ 15.1Warm up ทําแบบฝกหัดเรียงขั้นตอนการสงe –mail ใหถูกตอง

2. ใบงานที่ 15.2 Listening ทําแบบฝกหัดฟงบทสนทนาจากเทปแลวใสหมายเลข ตามลําดับตามขอความในบทสนทนา 3. ใบงานที่ 15.3: Speaking ทําแบบฝกหัดจับคูกนัพูดบทสนทนาเลียนแบบบทสนทนาตัวอยาง โดยใชขอมูลในแตละขอที่กาํหนด 4. ใบงานที่ 15.4 Reading ทําแบบฝกหัด จับคูกริยาและคํานามใหเปนการบอกขั้นตอนการทํางาน 5. ใบงานที่ 15.5 Reading and Speaking ทําแบบฝกหัดโดยเลือกคํากรยิาใหถูกตองตามภาพที่บอกขั้นตอนการทาํงานของ printer 6. ใบงานที่ 15.6 Writing ทําแบบฝกหัดเขยีนบทสนทนาโดย สรุปใจความมาเขียนขั้นตอนการทํางานของเครื่องมือแตละชนิด 1. Language Focus: 1.1 One-step instruction

Asking for instruction Positive instruction Negative instruction - Excuse me. Can you tell me how to use this card please? - Excuse me. Could you show me how to turn on this machine?

- Sure. Insert your card here. - Yes. Press this button.

- Do not connect any other equipment into the same outlet. - Never use the machine near a humidifier.

Conversation 1 A: Excuse me. Can you help me for a minute? B: Sure. What is it? A: Can you how me how to turn this fan on? B: Yes. Pull this chain. A: Thank you very much. B: You’re welcome.

1.2 Multi-instruction

Asking for instruction Positive instruction Negative instruction - Excuse me. Can you show me how to installing the Point Head?

- First, raise the lock lever. - Then, remove the protective cap from the print head. - Next, place the print head in the holder. - Finally, lock the print head into place by lowering the lever.

- Do not touch internal parts other than the lock lever - Do not attempt to reattach the protective cap. -

Conversation 2 A: How’s your first day on the job going? B: It’s fine. Thank you. A: Do you really know how to lock the cash register? B: No, I don’t. Could you show me how? A: All right. First, take out the tray. Then, close the drawer. B: Like this? A: Okay so far? B: I’m following you. What do I do next? A: Finally, turn the key on the side. Have you got that? B: Yes, I understand. Is that all? A: Yes, that’s all. B: Thank you very much. A: You’re welcome.

1.3 Asking for step in doing things. - What do I do first? - What do I do next? - What do I do then? - What’s next? - Is that all? 1.4 Sequence markers These sequence markers will help you give order of instructions.

- First, Second, Third…….. - Firstly, Secondly, Thirdly………… - When……………,…………….. - First of all………. - After that…………… - Next the, and then……… - Finally

ใบความรูท่ี 15 The language for instructions (โครงสรางประโยคสาํหรับคําสั่ง คาํแนะนํา)

1. Imperative Verb

positive negative

- Put it on the table. - Insert the card. - Hang up the handset. - Keep the disk in a box. - Be careful.

- Don’t touch this part. - Don’t get the disk wet. - Do not step on the cord. - Don’t be late.

- ใช imperative form สําหรับคําแนะนํา - ใช do not ในการพูดอยางเปนทางการหรอืเขียน แตใช don’t ในการพดูแบบไมเปนทางการ - ใช be หรือ don’t กอนคําคณุศัพท - ใช do หนาคํากริยาในกรณทีี่เปนคําแนะนําแบบ Strong instruction - สามารถใช avoid + Ving และ never +V1 ในกรณีที่เปนปฏิเสธิ์ แทนที่ do หรือ don’t

2. Means to carry out technician instructions. ใช by + Ving หรือ by means of + Ving เชน - Enter the fax number by using numeric keypad. - Turn the power on by plugging the power cord into the wall outlet.

3. Sequence markers. คําเหลานี้จะชวยใหปฏิบัติตามคําสั่งอยางเปนขั้นตอน เชน - First, second, third……………… - Firstly, secondly, thirdly…………… - When…………………… - First of all…………………. - After that…………………… - Next then, and then………….. - Finally…………………….

ใบงานที่ 15.1 Warm up: Exercise 1 Rearrange these instructions into correct order.

How to send e-mail message ………………. A) Click “Sign out” ………………. B) Enter your ID and password to sign in. ………………. C) Print your message. ………………. D) Enter the receiver’s e-mail address. ………………. E) Click “Send” ………………. F) Click “Sing in” ……………… G) Click “Compose Mail” ……………… H) Shut down your computer.

ใบงานที่ 15.2 Listening Exercise2: Suda wants to use ATM, but she is not sure how to use it. She requests the man who is waiting next to her to show her the steps. Listen to their conversation. Put the numbers of instruction as you hear. ……………..a) Could you speak up please? ……………..b) First, you lift up the receiver. ……………..c) Insert the card in the slot here. ……………..d) To do this you need a phone card. ……………..e) Now, what’s next? ……………..f) Don’t forget to remove your card. ……………..g) Hang up the receiver and lift it again. ……………..h) Then you dial the number. ……………..i) This time you dial the number slowly.

ใบงานที่ 15.3 Speaking Exercise 3: Pair work. Practice the conversation as the example by using the situation given. A: Excuse me. Can you help me for a minute? B: Sure. What is it? A: Can you how me how to turn this fan on? B: Yes. Pull this chain. A: Thank you very much. B: You’re welcome. 1. A: turn on the machine B: flip the switch 2. A: punch in B: put in your time card like this 3. A: turn off the radio B: turn this knob to the left 4. A: turn on talking dictionary B: push this button 5. A: lock in the program B: fill the code A: …………………………………………………………………………. B: …………………………………………………………………………. A: …………………………………………………………………………. B: …………………………………………………………………………. A: …………………………………………………………………………. B: ………………………………………………………………………….

ใบงานที่ 15.4 Reading: Exercise 2 These verbs of instruction are found in computer, telephone, photocopier ATM etc. Match the verbs with the nouns.

verbs nouns 1. insert 2. send 3. place 4. press 5. select 6. withdraw 7. transfer 8. click 9. dial 10. lift up

a) the card b) the picture c) the lid d) the button e) the e – mail message f) the mouse g) the money h) the call i) the desired number j) the paper size

1. ……………………………………………………………………….. 2. ……………………………………………………………………….. 3. ……………………………………………………………………….. 4. ……………………………………………………………………….. 5. ……………………………………………………………………….. 6. ……………………………………………………………………….. 7. ……………………………………………………………………….. 8. ……………………………………………………………………….. 9. ……………………………………………………………………….. 10. ……………………………………………………………………….

ใบงานที่ 15.5 Reading and Speaking Exercise 5: Look at the instruction of installing the Print Head of the printer. Choose the right verb to fill in the blanks and practice saying it. Place Lock Raise Remove 1…………………..the lock lever. - Do not touch internal parts other than the lock

lever. 2. ………………………. the protective cap from the printer. - Do not touch the print head nozzles of electrical contacts. - Do not attempt to reattach the protective cap once it has been removed. 3. ……………………. the print head in the holder.

4. ……………… the print head into place by lowering the lever.

- Do not touch the lock lever once the print head has been installed.

ใบงานที่ 15.6 Speaking and Writing Exercise 6: Work in pairs. Take turn to ask for and give instruction. First, put the instruction in the correct order. Then complete the conversation.

1. A: Could you show me how to use the copier machine? B: Sure. First…………………………………………………………………………….. Then…………………………………………………………………………………… Finally………………………………………………………………………………… - Put down the cover - Press the START button - Place the original face down on the glass.

2. A: Could you tell me how to do a credit card sale? B: Certainly. First……………………………………………………………………… Then………………………………………………………………………………… Finally………………………………………………………………………………. - Ask the customer to sign - Place the credit card voucher and card in the machine. - Write the date and amount on the voucher.

3. A: Could you tell me how to search the job form the Internet? B: Of course. First……………………………………………………………………… Then………………………………………………………………………………… Next………………………………………………………………………………… Finally……………………………………………………………………………… - Click “SEARCH” - Select the “Search engine” - Key in the key word “job” - Press “enter” key.

4. A: Could you tell me how to apply for a scholarship? B: Yes. First…………………………………………………………………………. Then………………………………………………………………………………. Next………………………………………………………………………………. - Fill out the form. - Get an application form. - Turn in the form to the office.

5. A: Could you show me how to send the fax message? B: Certainly. First……………………………………………………………………. Then……………………………………………………………………………… Next……………………………………………………………………………… Finally…………………………………………………………………………… - Pick up the handset. - Press the START and hang up the hand set. - Dial the number - Place document face down on the feeder.

แบบทดสอบประจําหนวยท่ี 15 Underline the correct verb to complete these operating instructions.

1. You must / mustn’t copy originals that are glued or stuck together. 2. You mustn’t / need to pull out the plug when you are copying something. 3. You don’t have to / must switch the copier off before removing the covers and putting your

hand inside the machine. 4. You needn’t /mustn’t put drinks on the copier. 5. If there is a paper jam, you don’t have to / have to open the copier to remove the paper. 6. You mustn’t / needn’t sort the copies if you select the sort function. 7. You mustn’t / don’t have to store the toner in a damp or wet place. 8. You have to/ needn’t select the number of copies you want before pressing the start button. 9. You don’t have to / must cut the wire when the copier is plugged in. 10. You have to / mustn’t call a service technician when there is a paper jam.

Answer Exercise 1 1. B 2. F 3. G 4. D 5. C 6. E 7. A 8. H Exercise 2 a) 8 b) 2 c) 3 d) 1 e) 4 f) 9 g) 6 h) 5 i) 7 Exercice 3

1. A: Can you how me how to turn on the machine? B: Yes. Flip this switch.

2. A: Can you how me how to punch in? B: Yes. Put in your time card like this.

3. A: Can you how me how to turn off the radio? B: Yes. Turn this knob to the left.

4. A: Can you how me how to turn on talking dictionary? B: Yes. Push this button.

5. A: Can you how me how to lock in the program? B: Yes. Fill the code.

Exercice 4 1. Insert the card 2. Select the paper size 3. Send the e-mail message 4. Dial the code number 5. Press the button 6. Withdraw the money 7. Click the mouse 8. Lift up the lid 9. Place the picture 10. Transfer the call Exercise 5 1. Raise 2. Remove 3. Place 4. Lock

Exercise 6 1. First place the original face down on the glass. Then put down the cover. Finally press the button. 2. First place the credit card voucher and card in the machine Then write the date and amount on the voucher. Finally ask the customer to sign. 3. First select the search engine. Then click “SEARCH” Next key in the key word “job” Finally press “enter” key. 4. First get an application form. Then fill out the form. Finally turn in the form to the office. 5. First place document face down on the feeder. Then pick up the handset. Next dial the number. Finally press the START and hang up the hand set. เฉลยแบบทดสอบประจําหนวยท่ี 15 1. mustn’t 2. mustn’t 3. don’t have to 4. mustn’t 5. have to 6. needn’t 7. mustn’t 8. have to 9. Don’t have to 10. have to